You are on page 1of 288

Chapter 1

Partnership Formation

1. A partnership formed for the exercise of a profession which is duly registered is an example of:
a. Universal partnership of profits
b. Universal partnership of all present property
c. Particular partnership
d. Partnership by estoppel
Answer: (c)

2. One of the following is not a characteristic of contract of partnership.


a. Real, in that the partners must deliver their contributions in order for the partnership contract
to be perfected
b. Principal, because it can stand by itself
c. Preparatory, because it is a means by which other contracts will be entered into
d. Onerous, because the parties contribute money, property, or industry to the common fund
Answer: (a)

3. One of the following is not a requisite of a contract of partnership. Which is it?


a. There must be a valid contract
b. There must be a mutual contribution of money, property, or industry to a common fund
c. It is established for the common benefit of the partners which is to obtain profits and divide
the same among themselves
d. The articles are kept secret among members
Answer: (d)

4. The minimum capital in money or property except when immovable property or real rights
thereto are contributed, that will require the contract of partnership to be in a public instrument
and be registered with the Securities and Exchange Commission (SEC).
a. P5, 000.00
b. P10, 000.00
c. P3, 000.00
d. P30, 000.00
Answer: (c)

5. Roberts and Smith drafted a partnership agreement that lists the following assets contributed at
the partnership’s formation:
Contributed by
Roberts Smith
Cash P 20,000 P 30,000
Inventory 15,000
Building 40,000
Furniture & Equipment 15,000
The building is subject to a mortgage of P 10,000, which the partnership has assumed. The
partnership agreement also specifies that profits and losses are to be distributed evenly. What
amounts should be recorded as capital for Roberts and Smith at the formation of the partnership?
Roberts Smith
a. 35,000 85,000
b. 35,000 75,000
c. 55,000 55,000
d. 60,000 60,000

Suggested Answer: (b) 35,000 & 75,000

Roberts: 20,000 + 15,000 = P35, 000 Smith: 30,000 + 15,000 + 40,000 – 10,000 = P75,000.
The partner’s capital credit is based upon the net assets contributed by the particular partner,
thus the liabilities assumed reduced the fair market value of the building invested.

6. Using the information in No. 2, under the bonus method, what is the amount of bonus?
a. 20,000 bonus to Grey
b. 20,000 bonus to Redd
c. 40,000 bonus to Grey
d. 40,000 bonus to Redd

Suggested Answer: (b) 20,000 bonus to Redd

Contributed Capital Agreed Capital Increase (Decrease)


Grey 60,000 40,000 (20,000)
Redd 20,000 40,000 20,000
Total 80,000 80,000

The partnership agreement provides for equal initial capital. Thus under the bonus method, the
capital credit for Redd should be the same as the contribution for Grey, resulting to P20,000
bonus from Grey to Redd.

7. On May 1, 2010, the business assets of John and Paul appear below:

John Paul
Cash P 11,000 P 22,354
Accounts Receivable 234,536 567,890
Inventories 120,035 260,102
Land 603,000
Building 428,267
Furniture & Fixture 50,345 34,789
Other Assets 2,000 3,600
Total P 1, 020, 916 P 1, 317, 002

Accounts Payable P 178,940 P 243,650


Notes Payable 200,000 345,000
John, Capital 641, 976
Paul, Capital\ 728,352
Total P 1, 020, 916 P1, 317, 002

John and Paul agreed to form a partnership contributing their respective assets and equities
subject to the following adjustments:
a. Accounts receivable of P20, 000 in John’s books and P35, 000 in Paul’s are uncollectible.
b. Inventories of P5, 500 n P6, 700 are worthless in John’s and Pail’s respective books.
c. Other assets of P2, 000 and P3, 600 in John’s and Paul’s respective books are to be written
off.

The capital accounts of John and Paul, respectively, after the adjustments will be:
a. 614, 476 683, 052 c. 640, 876 712, 345
b. 615, 942 717, 894 d. 613,576 683, 350
Suggested Answer: (a) 614, 476 683, 052
John: 641, 976 – 20, 000 – 5, 500 – 2, 000 = P 614, 476
Smith: 728, 352 – 35, 000 – 6, 700 – 3, 600 = P 683, 052

8. Based on No. 4, how much assets does the partnership have?


a. 2, 317, 918
b. 2, 237, 918
c. 2, 265, 118
d. 2, 365, 218

Suggested Answer: (c) 2, 265, 118


John: 1, 020, 916 – 20, 000 – 5, 500 – 2, 000 = P 993, 416
Smith: 1, 317, 002 – 35, 000 – 6, 700 – 3, 600 = P 1, 271, 702
Total: 2, 337, 918 – 55, 000 – 12, 200 – 5, 600 = P 2, 265, 118
Chapter 2

Partnership Operations

1. The Flat and Iron partnership agreement provides for Flat to receive a 20% bonus on profits
before bonus. Remaining profits and losses are divided between Flat and Iron in the ratio of 2:3.
Which partner has a greater advantage when the partnership has a profit or when it has a loss?
PROFIT LOSS
a. Flat Iron
b. Flat Flat
c. Iron Flat
d. Iron Iron

Answer: B
Profit - bonus 20%+ Balance (2/5 x 80%)= 52%
Loss - Bonus 0 + P/L (2/5 x 100%) 40%= 40%

2. Downs, Frey and Vick formed the DFV general partnership to act as manufacturer’s
representatives. The partners agreed Downs would receive 40% of any partnership profits and
Frey and Vick would each receive 30% of such profit. It was also agreed that the partnership
would not terminate for 5 years. After the fourth year, the partners agreed to terminate the
partnership. At that time, the partners capital accounts were as follows: Downs, 20 000; Frey,
15000 and Vick 10,000. There were undistributed losses of 30 000. Vicks share of losses will be
a. 0
b. 1000
c. 9000
d. 10 000

Answer: C
30 000 x 30%= 9 000

3. Red and White formed a partnership in 2010. The partnership agreement provides for annual
salary allowances of 55 000 for Red and 45 000 for White. The partners share profits equally and
losses in a 60/40 ratio. The partnership had earnings of 80 000 for 2006 before any allowance to
partners. What amount of these earnings should be credited to each partners’ capital account?
RED WHITE
a. 40 000 40 000
b. 43 000 37 000
c. 44 000 36 000
d. 45 000 35 000

Answer: B
Red - 55 000 -12 000= 43 000
White – 45 000- 8000 = 37 000
4. Fox, Greg and Howe are partners with average capital balances during 2010 of 120 000, 60 000
and 40 000, respectively. Partners receive 10% interest on their average capital balances. After
deducting salaries of 30 000 to Fox, and 20 000 to Howe, the residual P/L is divided equally. In
2010, the partnership sustained a 33 000 loss before interest and salaries to partners. By what
amount should Fox’s capital change?
a. 7 000 increase
b. 11 000 increase
c. 35 000 decrease
d. 42 000 increase

Answer: A
Fox- 12 000 + 30 000 – 35 000 = 7 000

5. If a partnership has net income of 44 000 and partner X is to be allocated bonus of 10% of
income after the bonus. What is the amount of bonus?
a. 3 000
b. 3 300
c. 4 000
d. 4 400

Answer: C
44 000 – 40 000(44 000/110%) = 4 000

6. The partnership agreement of Donn, Eddy and Farr provides for annual distribution of P/L in the
ff. sequence:
 Donn, the managing partner, receives a bonus of 10% profit
 Each partner receives 6% interest on average capital investment
 Residual P/L is divided equally

Average capital investments were: Donn, 80 000; Eddy, 50 000; Farr, 30 000
What portion of 100 000 profit for 2010 should be allocated to Farr?
a. 28 600
b. 29 800
c. 35 133
d. 41 600

Answer: A
1 800 ( 6% on ave. cap.) + 26 800 ( balance/equally) = 28 600

7. Partners AA and BB have P/L agreement with the ff. provisions: salaries of 30 000 for AA and 45
000 for BB. Bonus to AA of 10% net income after salaries and bonus, interest of 10% on average
capital balances of 20 000 and 35 000, respectively. One third of any remaining profits will be
allocated to AA and the Balance to BB. If the net income is 102 500, how much should be
allocated to AA?
a. 44 250
b. 47 500
c. 41 000
d. 41 167

Answer: C
30 000 + 2 500 + 2 000 + 6 500 = 41 000

8. Refer to question 7, if the partnership had net income of 22 000, how much should be allocated to
AA, assuming that the provisions of the P/L agreement are ranked by order of priority starting
with salaries?
a. 13 200
b. 12 500
c. 12 000
d. 8 800

Answer: D
22 000 x 30/75

9. On October 31, 2010, Zita and Jones formed a partnership by investing cash of 300 000 and 200
000, respectively. The partners agreed to receive an annual salary allowance of 360 000 and to
give Zita a bonus of 20% of the net income after partners’ salaries, the bonus being treated as an
expense. If the profits after salaries and bonus are to be divided equally, and the profit after
salaries but before bonus of Zita is 360 000, how much is the share of Zita in the profit?
a. 100 000
b. 120 000
c. 210 000
d. 270 000

Answer: D

Salaries + bonus + balance

60 000 + 60 000 + 150 000 = 270 000


Chapter 3

Partnership Dissolution: Changes in Ownership

9. The change in relation of the partners caused by any ceasing to be associated in the carrying of
the business is known as:
a. Termination of the partnership
b. Winding up of the partnership affairs
c. Liquidation of the partnership business
d. Dissolution of the partnership
Answer: (d)

10. Three of the following will cause the automatic dissolution of a general partnership. Which one
will not?
a. When any event makes it unlawful for the business of the partnership to be carried on or for
the members to carry it on in partnership
b. Expulsion of any partner from the business bona fide in accordance with such a power
conferred by the agreement between the partners
c. A partner becomes in any way incapable of performing his part of the partnership contract
d. The insolvency of a partner or of the partnership
Answer: (c)

11. Which of the following will not cause automatic dissolution of a limited partnership?
a. Death of a general partner
b. Death of a limited partner
c. Insolvency of a general partner
d. Insanity of a general partner
Answer: (b)

12. Blue and Rubi are partners who share profits and losses in the ratio 6:4, respectively. On May 1,
2010, their respective capital accounts were as follows:
Blue P60, 000
Rubi 50, 000
On that date, Lind was admitted as a partner with one-third interest in capital, and profits for an
investment o P40, 000. The new partnership began with a total capital of P150, 000. Immediately
after Lind’s admission, Blue’s capital should be:
a. 50, 000
b. 54, 000
c. 56, 667
d. 60, 000

Suggested Answer: (b) 54, 000


Contributed Capital Agreed Capital Increase (Decrease)
Old Partners P 110, 000 P 100, 000 P (10, 000)
New Partner 40, 000 (1/3) 50, 000 10, 000
Total P 150, 000 P 150, 000

Blue’s capital before admission of Lind P 60, 000


Less: share in bonus to Lind (10, 000 x 60%) 6, 000
Blue’s capital after Lind’s admission P 54, 000

13. Fernando and Jose are partners with capital balances of P30, 000 and P70, 000, respectively.
Fernando has a 30% interest in profits and losses. All assets of the partnership are at fair market
value except equipment with book value of P300, 000 and fair market value of P320, 000. At this
time, the partnership has decided to admit Rosa and Linda as new partners. Rosa contributes cash
of 55, 000 for 20% interest in capital and 30% interest in profits and losses. Linda contributes
cash of P10, 000 and equipment with a fair value of P50, 000 for a 25% interest in capital and
35% interest in profits and losses. Linda is also bringing special expertise and client contacts into
the new partnership.

Using the bonus method, what is the amount of bonus?


a. 24, 750
b. 18, 250
c. 14, 000
d. 7, 500
Suggested Answer: (b) 18, 250

Contributed Capital Agreed Capital Increase (Decrease)


Old Partners P 100, 000 P 118, 250 P 18, 250
New Partners 115, 000 (45%) 96, 750 (18, 250)
Total P 215, 000 P 215, 000

14. Based on the information provided in No. 4, using the goodwill method, what is the amount of
goodwill traceable to the original partners?
a. 60, 000
b. 40, 000
c. 31, 250
d. 28, 750

Suggested Answer: (c) 31, 250


Contributed Capital Agreed Capital Increase (Decrease)
Old Partners P 100, 000 P 151, 250 P 51, 250
New Partners 115, 000 (45%) 123, 750 8, 750
Total P 215, 000 P 275, 000 P 60, 000
Total increase in capital P60, 000
Less: undervalued equipment (320, 000- 300, 000) 20, 000
Balance P 40, 000
Goodwill to Linda 8, 750
Goodwill to Original Partners P 31, 250

When there is a difference between the book value and fair market value of the partnership when
new partners are admitted, the goodwill method revalues assets to market value. To determine
the new capital of the partnership, contributed capital of the new partner may be divided by his
capital interest. In this case, where Linda will be provided with goodwill for bringing her
expertise and clients contact to the partnership, the capital of Rosa is used instead because it
serves as concrete basis with no goodwill involved, in determining the new capital of the
partnership. Thus, the new capital of the partnership is P275, 000 (55, 000/ 20%).
Chapter 4

Partnership Lump Sum Liquidation

1. In the liquidation of a partnership it is necessary to 1) distribute cash to the partners, 2) sell non-
cash assets, 3) allocate any gain or loss on realization to the partners, 4) pay liabilities. These
steps should be performed in the following order:
a. 2 3 4 1
b. 2 3 1 4
c. 3 2 1 4
d. 3 2 4 1

Answer: A

2. Peter and John who share P/L equally, decide to liquidate their partnership when their net assets
amounted to 260 000, and capital balances of 170 000 and 90 000, respectively.
If non cash assets were sold for amount equal to its book value, what amount of cash should Peter
and John received?
PETER JOHN
a. 130 000 130 000
b. 170 000 90 000
c. 180 000 80 000
d. 195 000 65 000

Answer: B

No gain or loss was realized

3. The following condensed balance sheet is prepared for the partnership of Smith and Jones, who
share P/L in the ratio of 60:40,

Other assets 450 000


Smith, loan 20 000
470 000

Accounts Payable 120 000


Smith, Capital 195 000
Jones, Capital 155 000
470 000

The partners decide to liquidate the partnership. If the other assets are sold for 385 000, what
amount of the available cash should be distributed to Smith?

a. 136 000
b. 156 000
c. 159 000
d. 195 000
Answer: A
Capital balance- loan – loss on realization (60:40)
195 000 – 20 000 – 39 000 = 136 000

4. On December 31, 2010, the partners of MNP partnership decided to liquidate their business.
Immediately before liquidation, the ff. condensed balance sheet was prepared:

Cash 50 000 Liabilities 375 000


Non cash assets 900 000 Nieva,loan 80 000
Perez, loan 25 000
Munoz, capital(50%) 312 500
Nieva, Capital(30%) 107 500
Perez, capital(20%) 50 000
Total 950 000 950 000

The non cash assets were sold for 400 000. Assuming Perez is the only solvent partner, what
amount of additional cash will be invested by Perez?
a. 37 143
b. 25 000
c. 5 000
d. 0

Answer: B
Capital balance + loan – loss on realization
50 000 + 25 000 – 100 000 =( 25 000 )

5. As of December 31, the books of AME partnership showed capital balances of 40 000, 25 000
and 5 000, respectively. The P/L ratio is 3:2:1. The partners decided to liquidate. They sold all
non cash assets for 37 000 cash. After settlement of all liabilities amounting to 12000, they still
have 28 000 cash left for distribution. The loss on realization of non cash assets was
a. 40 000
b. 42 000
c. 44 000
d. 45 000

Answer: B
Total capital before distribution – Cash left for distribution
70 000 – 28 000 = 42 000
6. Refer to no. 5. Assuming that any partner’s capital deficit balance is uncollectible, the share of A
in the 28 000 cash would be
a. 19 000
b. 18 000
c. 17 800
d. 40 000

Answer: C
Capital balance before liquidation – loss on realization – absorption of E
40 000 – 21 000 – 1 200 = 17 800

7. The following balance sheet is presented for the partnership ABC who share profits and losses in
the ratio of 5:3:2

Cash 120 000 Liabilities 280 000


Other assets 1080 000 A, capital 560 000
B, capital 320 000
C, capital 40 000
Total 1 200 000 1 200 000

Assume that the three partners decided to liquidate. The other assets are sold for 800 000, how
much will A receive?
a. 280 000
b. 324 000
c. 410 000
d. 412 000

Answer: C
Capital balance before liquidation – loss on realization – absorption of C
560 000 – 140 000 – 10 000 = 410 000

8. Refer to no. 8. How much B will receive?


a. 320 000
b. 236 000
c. 230 000
d. 228 000

Answer: C
Capital balance before liquidation – loss on realization – absorption of C
320 000 – 84 000 – 6 000 = 230 000

9. How much C will receive?


a. 40 000
b. 0
c. 16 000
d. 10 000

Answer: B
Capital balance before liquidation – loss on realization
40 000- 56 000= - 16 000 ( absorbed by other partners) = 0
Chapter 5

Partnership Installment Liquidation

1. Refers to the process of settling the business or affairs of the partnership after dissolution is
known as
a. Partnership formation
b. Termination
c. Partnership Liquidation
d. Incorporation
Answer: (c)

2. In accounting for the liquidation of a partnership, cash payments to partners after all non-partner
creditors’ claims have been satisfied, but before the final cash distribution, should be according to
a. The partner’s relative profit and loss sharing ratios
b. The final balances in partner’s capital accounts
c. The partner’s relative share of the gains and loss on liquidations
d. Safe payments computations
Answer: (d)

3. In partnership liquidation, the final cash distribution to the partner’s should be made in
accordance with the
a. Partner’s profit and loss sharing ratio
b. Balances of the partners’ loan and capital accounts
c. Ratio of the capital contributions by the partners
d. Ratio of capital contributions less withdrawals by the partners
Answer: (b)

4. After incurring losses resulting from very unprofitable operations, the Goh Kong Wei Partnership
decided to liquidate when the partner’s capital balances were:
Goh, Capital P80, 000
Kong, Capital 130, 000
We, Capital 96, 000
The non-cash assets were sold in installment. Available cash were distributed to partners in every
sale of non-cash assets. After the second sale of non-cash assets, the partners received the same
amount of cash in the distribution. And from the third sale of non-cash assets, cash available for
distribution amounts to P28, 000, and unsold non-cash assets has a book value of P12,500. Using
cash priority program, what amount did Wei received in the third installment of cash?
a. 11, 600
b. 8, 000
c. 5, 600
d. 2, 000

Suggested Answer: (c) 5, 600


P28, 000 x 20% = P5, 600
5. Partners Almond, Barney and Colors have capital balances of P20, 000, P50, 000 and P90, 000,
respectively. They split profits in the ratio 2:4:4, respectively. Under a safe cash distribution plan,
one of the partners will get the following total amounts in liquidation before any other partners
get anything:
a. 0
b. 15, 000
c. 40, 000
d. 180, 000

Suggested Answer: (c) 40, 000


Almond Barney Colors
Total Interest 20, 000 50, 000 90, 000
Divide by P & L 20% 40% 40%
Loss Absorption Balance 100, 000 125, 000 225, 000
Priority 1 – Colors (100, 00)
Balances 100, 000 125, 000 125, 000
Priority 2 – Colors & Barney (25, 000) (25, 000)
Balances (P & L) 100, 000 100, 000 100, 000

Since the question being asked is “one of the partners will get…before any other partners get
anything”, it is the partner under priority no. 1 (Colors). He shall receive, under priority no. 1, P40, 000
(100, 000 x 40%).

6. The ABC Partnership has assets with book value of P240, 000 and a market value of P195, 000,
outside liabilities of P70, 000, loans payable to partner Able of P20, 000 and capital balances for
partners Able, Baker, and Chapman of P70, 000, P30, 000, and P50, 000, respectively. The
partners share profits and losses equally. How would the first P100, 000 of available assets be
distributed?
a. P70, 000 to outside liabilities, P20, 000 to Able, and the balance equally among partners
b. P70, 000 to outside liabilities, and P30, 000 to Able
c. P70, 000 to outside liabilities, P25, 000 to Able, and P5, 000 to Chapman
d. P40, 000 to Able, P20, 000 to Chapman, and the balance equally among partners

Suggested Answer: (b) P70, 000 to outside liabilities, and P30, 000 to Able
Able Baker Chapman
Total Interest 90, 000 30, 000 50, 000
Divide by P & L 1/3 1/3 1/3
Loss Absorption Balance 270, 000 90, 000 150, 000
Priority 1 – Able (120, 000)
Balances 150, 000 90, 000 150, 000
Priority 2 – Able & Chapman (60, 000) (60, 000)
Balances (P & L) 90, 000 90, 000 90, 000

Payments by Priority: Able Baker Chapman


Priority 1 (120, 000 x 1/3) 40, 000
Priority 2 (60, 000 x 1/3) 20, 000 20, 000

Cash Liability Able Baker Chapman


Total 100, 000
Liability (70, 000) 70, 000
Balance 30, 000
Loan – A (20, 000) 20, 000
Balance 10, 000
Priority 1 10, 000 10, 000
Total 70, 000 30, 000

7. Given the information in No. 3, if all outside creditors and loans to partners had been paid, how
would the balance of the assets be distributed assuming Chapman had already received assets
with a value of P30, 000?
a. Each of the partners would receive P25, 000
b. Each of the partners would receive P40, 000
c. Able: P70, 000, Baker: P30, 000, Chapman: P20, 000
d. Able: P55, 000, Baker: P15, 000, Chapman: P5, 000

Suggested Answer: (d) Able: P55, 000, Baker: P15, 000, Chapman: P5, 000

Able Baker Chapman


Total Interest (excluding loan) 70, 000 30, 000 50, 000
Divide by P & L 1/3 1/3 1/3
Loss Absorption Balance 210, 000 90, 000 150, 000
Priority 1 – Able (60, 000)
Balances 150, 000 90, 000 150, 000
Priority 2 – Able & Chapman (60, 000) (60, 000)
Balances (P & L) 90, 000 90, 000 90, 000

Payments by Priority: Able Baker Chapman


Priority 1 (60, 000 x 1/3) 20, 000
Priority 2 (60, 000 x 1/3) 20, 000 20, 000

Cash Able Baker Chapman


MV of assets 195, 000
Liabilities (70, 000)
Able, Loan (20, 000)
Balance 105, 000
Priority 1 (20, 000) 20, 000
Balance 85, 000
Priority 2 (40, 000) 20, 000 20, 000
Balance 45, 000
Priority 3 (45, 000) 15, 000 15, 000 15, 000
Total 55, 000 30, 000 35, 000
Less: Asset taken by Chapman 30, 000
Balance 55, 000 15, 000 5, 000

8. The balance sheet of the partnership of Salve, Galo and Norma, who share in the profits and
losses in the ratio 5:3:2, respectively is as follows:
Assets Liabilities & Capital
Cash 30, 000 Liabilities 50, 000
Other Assets 320, 000 Salve, capital 80, 000
Galo, Capital 115, 000
Norma Capital 105, 000
Total 350, 000 Total 350, 000

The partnership is liquidated by installment. The first sale of non-cash assets with a book value of
P150, 000 realizes P100, 000. How should the remaining be distributed?
Salve Galo Norma
a. 50, 000 30, 000 20, 000
b. 40, 000 24, 000 16, 000
c. 0 31, 000 49, 000
d. 0 48, 000 32, 000

Suggested Answer: (c) 0 31, 000 49, 000


Salve Galo Norma
Capital Balances before liquidation 80, 000 115, 000 105, 000
Loss on Realization (50,000) (5:3:2) (25, 000) (15, 000) (10, 000)
Balances 55, 000 100, 000 95, 000
Less: Possible Loss (170, 000) (5:3:2) (85, 000) (51, 000) (34, 000)
Balances (30, 000) 49, 000 61, 000
Absorption of Salve (3:2) 30, 000 (18, 000) (12, 000)
Safe Payment to Partners 31, 000 49, 000
Chapter 6

Corporate Liquidation

1. If a dividend of 80% is allocable to class 7 unsecured creditors based on accounting statement of


affairs, it correctly may be concluded that
a. All unsecured claims will receive the same percentage of return.
b. All unsecured claims will be paid in full.
c. Class 1 through 6 unsecured claims will be paid in full.
d. Stockholders will receive 20% of their equity.

Answer: C

2. In the liquidation proceeding, if the proceeds on the realization of an asset exceed the lien against
that asset, the excess is assigned to
a. The holder of the lien.
b. Other lien holders whose assets will not realize a sufficient amount to cover their liens.
c. Meet the claims of the unsecured creditors.
d. The stockholders of the corporation.

Answer: C

3. If the value of pledged property is equal or more than the obligation, what is the treatment?
a. Partially secured
b. Fully secured
c. Collateralized
d. Unsecured

Answer: B

4. If the firm has more liabilities than assets, it is deemed


a. Liquidating
b. Bankrupt
c. Insolvent
d. Normal

Answer: C

5. He/she is appointed to take over the debtor’s properties in behalf of the creditor group
a. Broker
b. Judge
c. Trustee
d. Dealer

Answer: C
6. In May 2009, it was determined that it is necessary to complete the work in process of Wild West
Corp. to complete the work in process , 10 000 book value of raw materials and supplies and 10
000 conversion costs will be required. When completed, these goods will probably sell for
approx..50 000. The raw materials, which have a book value of 40 000, have an estimated total
realizable value of 20 000. What is the estimated amount that will become available for unsecured
creditors as a result of the realization of the work in process?
a. 50 000
b. 35 000
c. 30 000
d. 0

Answer: B
Estimated value upon completion 50 000
Less cost to complete:
Raw materials[10 000x(20000/40000) 5000
Conversion cost 10 000 15 000
35 000

7. The accountant of Drifting Corp. prepared a statement of affairs. Assets which there are no claims
or liens are expected to produce 700 000. Unsecured claims of all classes totaled to 1 050 000.
The ff.data claims deemed outstanding:
 Accrued salaries 15 000
 Unrecorded note for 10 000 on which 600 of interest has accrued held by Normandy Co
 A note for 30 000 secured by 40 000 receivable estimated to be 60% collectible held by
Jones Co.
 A 15 000 note on which 300 interest has accrued held by James Pty. Property with a BV
of 10 000 and MV of 18 000 is pledge to guarantee payment of principal and interest.
 Unpaid income taxes of 35 000

What is the total free assets?


a. 1 050 000
b. 700 000
c. 650 000
d. 1 000 000

Answer: B
Free Assets are assets with no claims or liens

8. What is the amount realized by partially secured creditors?


a. 10 600
b. 19 500
c. 24 900
d. 27 900
Answer: D
(Free assets- unsecured with priority) ÷ unsecured claims without priority= percent of recovery
(700 000 – 50 000 ) /1000 0000= 65 %

Realizable amount of AR( 60% x 40 000) 24 000


Add unsecured portion (30 000-24000)x 65% 3 900
27 900

ANOTA, Juanillo II
DIGNADICE, Allana
PRTC Multiple Choice Questions (Adv. Acctg.1)

CHAPTER 1: Partnership Formation

2. Which of the following is NOT a feature of a general partnership?


a. mutual agency
b. limited life
c. limited liability
d. none of these
 
3. A partner's tax basis in a partnership is comprised of which of the following items?
I. The partner's tax basis of assets contributed to the partnership.
II. The amount of the partner's liabilities assumed by the other partners.
III. The partner's share of other partners' liabilities assumed by the
partnership. 

a. I plus II minus III


b. I plus II plus III
c. I minus II plus III
d. I minus II minus III

4. Which of the following accounts could be found in the general ledger of a


partnership?

    
a. Option A
b. Option B
c. Option C
d. Option D
 
5. Which of the following accounts could be found in the PQ partnership's general
ledger?
I. Due from P
II. P, Drawing
III. Loan Payable to Q 

a. I, II
b. I, III
c. II, III
d. I, II, and III

6. Anton and Bauzon formed a partnership and agreed to divide initial capital equally,
even though Anton contributed P100,000 and Bauzon contributed P84,000 in
identifiable assets. Under the bonus method, to adjust capital accounts, Bauzon's
intangible assets should be debited for:
a. 0
b. 16,000
c. 8,000
d. 46,000

7. Roy, Sam and Tim decided to engage in a real estate venture as a partnership. Roy
invested P140,000 cash and Sam provided an office and furnishings valued at
P220,000. (There is a 60,000 note payable remaining on the furnishings to be assumed
by the partnership). Although Tim has no tangible assets to invest, both Roy and Sam
believe that Tim's expert salesmanship provides an adequate investment. The partners
agree to receive an equal capital interest in the partnership. Using the bonus method,
what is the capital balance of Tim?
a. 0
b. 50,000
c. 100,000
d. 140,000

8. Lara and Mitra formed a partnership on July 1, 2011 and invested the following
assets: P130,00 cash by Lara, and P200,000 cash and P50000 computer equipment by
Mitra. The computer equipment has a note payable amounting to P10,000, which was
assumed by the partnership. The partnership agreement provides that Lara and Mitra
will have an equal capital credit. Using the goodwill method, the amount of goodwill to
be recorded upon formation of partnership is:
a. 100,000
b. 110,000
c. 120,000
d. 140,000

9. Ana and Elsa form a new partnership. Ana invests P300,000 in cash for her 60%
interest in the capital and profits of the business. Elsa contributes land that has an
original cost of P40,000 and a fair market value of P70,000, and a building that has a
tax basis of P50,000 and a fair market value of P90,000. The building is subject to a
P40,000 mortgage that the partnership will assume. What amount of cash should Elsa
contribute?
a. 40,000
b. 80,000
c. 110,000
d. 150,000

10. Jones and Smith formed a partnership with each partner contributing the following
items:

  
Assume that for tax purposes Jones and Smith agree to share equally in the
liabilities assumed by the Jones and Smith partnership.
 
What is each partner's tax basis in the Jones and Smith partnership?

   
a. Option A
b. Option B
c. Option C
d. Option D

ANSWERS & SOLUTIONS (Chapter 1)

1. a
2. c
3. c
4. d
5. d
6. a

Zero, because under the bonus method, a transfer of capital is only required.
7. c

Roy Sam Tim

Cash P140,000 – –

Office Equipment – P220,000 –

Note payable ________ _( 60,000) ______

Net asset invested P140,000 P160,000 P        –

Agreed capitals, equally (P300,000/3) = P100,000

8. b

Lara Mitra

Cash P130,000 P200,000

Computer equipment – 50,000

Note payable ________ _( 10,000)

Net asset invested P130,000 P240,000

Goodwill (P240,000 - P130,000) = P110,000

9. b
Total Capital (P300,000/60%) P500,000
Elsa's interest ______40%
Elsa's capital P200,000
Less: Non-cash asset contributed at market value
Land P 70,000
Building 90,000
Mortgage Payable (  40,000) _120,000
Cash contribution P  80,000

10. a
Jones: (80000+300000) - 120000 + (180000/2) = 350000
Smith: (40000+200000) - 60000 + (180000/2) = 270000
CHAPTER 2: Partnership Operations

1. If the partnership agreement provides a formula for the computation of a bonus to the
partners, the bonus would be computed:
a. next to last, because the final allocation is the distribution of the profit residual.
b. before income tax allocations are made.
c. after the salary and interest allocations are made.
d. in any manner agreed to by the partners.

2. Bob and Fred form a partnership and agree to share profits in a 2 to 1 ratio. During
the first year of operation, the partnership incurs a $20,000 loss. The partners should
share the losses
a. based on their average capital balances.
b. in a 2 to 1 ratio.
c. equally.
d. based on their ending capital balances

3. Drawings
a. are advances to a partnership.
b. are loans to a partnership.
c. are a function of interest on partnership average capital.
d. are the same nature as withdrawals.

4. If partnership agreement provides for the division of profits only, losses should be
divided:
a. equally
b. using the same approach as division of profits
c. according to ratio of beginning capital
d. according to ratio of average capital

5. Allocation of an error should be based on profit and loss ratio in effect when:
a. the error occurred.
b. the error was discovered.
c. the error was corrected.
d. allocation should always be made equally.

(Use the following information for questions 6, 7 and 8.)

Albion and Blaze share profits and losses equally. Albion and Blaze receive salary
allowances of P20,000 and P30,000, respectively, and both partners receive 10%
interest on their average capital balances. Average capital balances are calculated at
the beginning of each month balance regardless of when additional capital
contributions or permanent withdrawals are made subsequently within the month.
Partners’ drawings are not used in determining the average capital balances. Total net
income for 2006 is P120,000.
Albion Blaze
January 1 capital balances P 100,000 P 120,000
Yearly drawings ($1,500 a month) 18,000 18,000
Permanent withdrawals of capital:
June 3 ( 12,000 )
May 2 ( 15,000 )
Additional investments of capital:
July 3 40,000
October 2 50,000

6. What is the weighted-average capital for Albion and Blaze in 2006?


a. 100,000 and 120,000
b. 105,333 and 126,667
c. 110,667 and 119,583
d. 126,667 and 105,333

7. If the average capital for Albion and Blaze from the above information is 112,000 and
119,000, respectively, what will be the total amount of profit allocated after the salary
and interest distributions are completed?
a. 70,000.
b. 73,100.
c. 75,000.
d. 80,000.

8. If the average capital balances for Albion and Blaze are $100,000 and $120,000,
what will the final profit allocations for Albion and Blaze in 2006?
a. 50,000 and 70,000.
b. 54,000 and 66,000.
c. 70,000 and 50,000.
d. 75,000 and 45,000.

9. The XYZ partnership provides a 10% bonus to Partner Y that is based upon
partnership income, after deduction of the bonus. If the partnership's income is 121,000,
how much is Partner Y's bonus allocation?
a. 11,000.
b. 11,450.
c. 11,650.
d. 12,100.

10. Bloom and Carnes share profits and losses in a ratio of 2:3, respectively. Bloom and
Carnes receive salary allowances of 10,000 and 20,000, also respectively, and both
partners receive 10% interest based upon the balance in their capital accounts on
January 1. Partners’ drawings are not used in determining the average capital balances.
Total net income for 2006 is 60,000. If net income after deducting the interest and salary
allocations is greater than 20,000, Carnes receives a bonus of 5% of the original
amount of net income.

Bloom Carnes

January 1 capital balances P 200,000 P 300,000

Yearly drawings (P1,500 a month) 18,000 18,000

What are the total amounts for the allocation of interest, salary, and bonus, and,
how much over-allocation is present?
a. 60,000 and 0.
b. 80,000 and 20,000.
c. 83,000 and 0.
d. 83,000 and 23,000.

ANSWERS & SOLUTIONS (Chapter 2)

1. d
2. b
3. d
4. b
5. a
6. c

Albion: [(100,000 x 6) + (88,000 x 1) + (128,000 x 5)]/12 = 110,667

Blaze: [(120,000 x 5) + (105,000 x 5) + (155,000 x 2)]/12 = 119,583

7. b Capital: (112,000 + 119,000)x(10%) = 23,100

Salary: (20,000 + 30,000) = 50,000

Total: 23,100 + 50,000 = 73,100

8. b Albion: (100,000 x 10%) + 20,000 + 24,000 = 54,000

Blaze: (120,000 x 10%) + 30,000 + 24,000 = 66,000

9. a
B = .1x(121,000 - B)
B = 12,100 - .1B
1.1B = 12,100
B = 11,000

10. b
Interest: (500,000 x 10%) = 50,000
Salary: (10,000 + 20,000) = 30,000
Bonus: Condition not met = 0

Total allocations = 80,000 and over-allocations = 80,000 - $60,000 = 20,000

CHAPTER 3: Partnership Dissolution: Changes in Ownership


 2. When a partner retires from a partnership and the retiring partner is paid more than
the capital balance in her account, which of the following explains the difference?
I. The retiring partner is receiving a bonus from the other partners.
II. The retiring partner's goodwill is being recognized. 
a. I only
b. II only
c. Either I or II
d. Neither I nor II

 (Use the following information for questions 3, 4 and 5.)

3. Refer to the above information. Which statement below is correct if a new partner
receives a bonus upon contributing assets into the partnership? 
a. B < A and D = C - A
b. B > A and D = C + A
c. A = B and A = D + C
d. B > A and C = D + A

 4. Refer to the above information. Which statement below is correct if goodwill of the
old partners is recognized upon the contribution of assets into the partnership by a new
partner? 
a. B = A and D < C + A
b. B = A and D > C + A
c. B < A and D = C + A
d. B > A and D < C + A

5. Refer to the above information. Which statement below is correct if a new partner
purchases an interest in capital directly from the old partners? 
a. C < D
b. C = D
c. C = D and B = A
d. C < D and B = A

(Use the following information for questions 6, 7 and 8.)


A summary balance sheet for the McCune, Nall, and Oakley partnership appears
below. McCune, Nall, and Oakley share profits and losses in a ratio of 2:3:5,
respectively.
Assets

Cash P 50,000

Inventory 62,500

Marketable securities 100,000

Land 50,000

Building-net 250,000

Total assets P 512,500

Equities

McCune, capital P 212,500

Nall, capital 200,000

Oakely, capital 100,000

Total equities P 512,500

The partners agree to admit Pavic for a one-fifth interest. The fair market value of
partnership land is appraised at $100,000 and the fair market value of inventory is
$87,500. The assets are to be revalued prior to the admission of Pavic and there is
$15,000 of goodwill that attaches to the old partnership .

6. By how much will the capital accounts of McCune, Nall, and Oakley increase,
respectively, due to the revaluation of the assets and the recognition of goodwill?

a. The capital accounts will increase by 25,000 each.


b. The capital accounts will increase by 30,000 each.
c. P18,000, P27,000, and P45,000.
d. P20,000, P25,000, and P30,000.

7. How much cash must Pavic invest to acquire a one-fifth interest?

a. 117,500.
b. 120,500.
c. 146,875.
d. 150,625.
8. What will the profit and loss sharing ratios be after Pavic’s investment?

a. 1:2:4:2.
b. 2:3:5:2.
c. 3:4:6:2.
d. 4:6:10:5.

9. Davis has decided to retire from the partnership of Davis, Eiser, and Foreman. The
partnership will pay Davis P200,000. Goodwill is to be recorded in the transaction as
implied by the excess payment to Davis. A summary balance sheet for the Davis, Eiser,
and Foreman partnership appears below. Davis, Eiser, and Foreman share profits and
losses in a ratio of 1:1:3, respectively.

Assets
Cash P 75,000
Inventory 82,000
Marketable securities 38,000
Land 150,000
Building-net 255,000
Total assets P 600,000

Equities
Davis, capital 160,000
Eiser, capital 140,000
Foreman, capital 300,000
Total equities P 600,000

What goodwill will be recorded?


a. 40,000.
b. 120,000.
c. 160,000.
d. 200,000.

10.  In the RST partnership, Ron's capital is 80,000, Stella's is 75,000, and Tiffany's is
50,000. They share income in a 3:2:1 ratio, respectively. Tiffany is retiring from the
partnership. Tiffany is paid 60,000, and no goodwill is recorded. What is the Ron's
capital balance after Tiffany withdraws from the partnership?
a. 74,000
b. 71,000
c. 75,000
d. 86,000

ANSWERS & SOLUTIONS (Chapter 3)


1. d
2. c
3. b
4. b
5. b
6. c

The assets will be valued upward by 90,000 which, allocated on a 2:3:5 basis,
yields 18,000 to McCune, 27,000 to Nall, and 45,000 to Oakely.

7. d
After the revaluation, the assets will be recorded at 602,500. If Pavic is admitted
for a one-fifth interest, the 602,500 represents 80% of the total implied capital. Dividing
602,500 by 80% gives a total capitalization of 753,150 for which 150,625 is required
from Pavic for a 20% interest.

8. d
Each of the original partners has given up 20% of their interest to Pavic. Their
profit and loss sharing ratios will therefore be 80% of what they were before the
admission of Pavic.

McCune 20% x 80% = 16%


Nall 30% x 80% = 24%
Oakely 50% x 80% = 40%
Pavic = 20%

Expressed as: 4:6:10:5

9. d 200,000
10. a 80000 - [(60000-50000)x3/5)] = 74,000

CHAPTER 4: Partnership Lump-Sum Liquidation

1. Which of the following procedures is acceptable when accounting for a deficit balance
in a partner’s capital account during partnership liquidation?
a. A partner with a negative capital balance must contribute personal assets to
the partnership that are sufficient to bring the capital account to
zero.
b. If a partner with a negative capital balance is personally insolvent, the negative
capital balance may be absorbed by those partners having a positive
capital balance according to the residual profit and loss sharing
ratios that apply to all the partners.
c. If a partner with a negative capital balance is personally insolvent, the
negative capital balance may be absorbed by those partners
having a positive capital balance according to the residual
profit and loss sharing ratios that apply to those partners having
positive balances.
d. All the above procedures are acceptable.

2. In partnership liquidation, how are partner salary allocations treated?


a. Salary allocations take precedence over creditor payments.
b. Salary allocations take precedence over amounts due to partners with respect
to their capital interests, but not profits.
c. Salary allocations take precedence over amounts due to partners with respect
to their capital profits, but not capital interests.
d. Salary allocations are disregarded.

3. A simple partnership liquidation requires


a. periodic payments to creditors and partners determined by a safe payments
schedule.
b. partnership assets to be converted into cash with full payment made to
all outside creditors before remaining cash is distributed to
partners in a lump sum payment.
c. only creditors to be paid in an orderly manner.
d. periodic payments to partners as cash becomes available.

5. If conditions produce a debit balance in a partner’s capital account when liquidation


losses are allocated
a. the partner receives further allocations of liquidation losses, but not gains.
b. the partner receives no further allocation of liquidation losses and gains.
c. the partner is no longer obligated to partnership creditors.
d. the partner has an obligation of personal net assets to the other partners

(Use the following information for questions 6, 7 and 8).

On June 30, 2006, the Warle, Xin, and Yates partnership had the following fiscal
year-end balance sheet:

Cash 4,000 Accounts payable 7,000


Accounts receivable 6,000 Loan from Xin 5,000
Inventory 14,000 Warle, capital(20%) 14,000
Plant assets-net 12,000 Xin, capital(30%) 10,000
Loan to Warle 6,000 Yates, capital(50%) 6,000
Total assets 42,000 Total liab./equity 42,000

The percentages shown are the residual profit and loss sharing ratios. The
partners dissolved the partnership on July 1, 2006,. and began the liquidation process.
During July the following events occurred:
* Receivables of 3,000 were collected.
* The inventory was sold for 4,000.
* All available cash was distributed on July 31, except for 2,000 that was set
aside for contingent expenses.

6. The book value of the partnership equity (i.e., total equity of the partners) on June
30, 2006 is
a. 60,000.
b. 29,000.
c. 30,000.
d. 42,000.

7. The cash available for distribution to the partners on July 31, 2006 is
a. 2,000.
b. 4,000.
c. 7,000.
d. 11,000.

8. How much cash would Xin receive from the cash that is available for distribution on
July 31?
a. 0.
b. 600.
c. 1,000.
d. 2,000.

10. O, P, & Q LLP, is beginning liquidation. It has no cash, total liabilities of 120000,
including a 20000 loan payable to P, and equal partners' capital account balances of
80000. The income-sharing ratio is 5:1:4, respectively. If a portion of the noncash
assets with a carrying amount of 280000 realizes 120000, the cash payment that P
receives is
a. 40,000.
b. 88,000
c. 106,000.
d. Some other amount.

ANSWERS & SOLUTIONS (Chapter 4)

1. c
2. d
3. b
4. a
5. d
6. b
(14,000 Warle capital + 10,000 Xin capital + 6,000 Yates capital + 5,000 Loan
from Xin - 6,000 Loan to Warle) = 29,000

7. a
(4,000 beginning balance + 3,000 cash collected + 4,000 for inventory sold -
7,000 of accounts payable - 2,000 for expenses) = 2,000

8. d

9. a

10. b
P80,000 + P20,000 – P4,000 – P8,000 = P88,000
CHAPTER 5: Partnership Installment Liquidation

1. In partnership installment liquidations, what are safe payments?


a. The amounts of distributions that can be made to the partners, after all
creditors have been paid in full.
b. The amounts of distributions that can be made to the partners with
assurance that such amounts will not have to be returned to
the partnership.
c. The amounts of distributions that can be made to the partners, after all non-
cash assets have been adjusted to fair market value.
d. All the above are examples of the safe payments concept.

2. If all partners are included in the first installment of an installment liquidation, then in
future installments
a. cash will be distributed according to the residual profit and loss sharing
ratio.
b. cash should not be distributed until all non-cash assets are converted into
cash.
c. a safe payments schedule must be prepared before each cash distribution to
avoid excessive payments to partners.
d. a cash distribution plan must be prepared so that partners will know when they
will be included in cash distributions.

3. In a schedule of assumed loss absorptions


a. the partner with lowest loss absorption is eliminated last.
b. it is necessary to have a cash distribution plan first.
c. the least vulnerable partner is eliminated first.
d. the most vulnerable partner is eliminated first.

4. Which partner is considered the most vulnerable as a result of a computation of


vulnerability rankings?
a. The partner with the lowest vulnerability ranking, who also has the
lowest loss absorption potential.
b. The partner with the lowest vulnerability ranking, who also has the highest loss
absorption potential.
c. The partner with the highest vulnerability ratio, who also has the lowest loss
absorption potential.
d. The partner with the highest vulnerability ranking, who also has the highest
loss absorption potential.
5. The rank order is for claims against a bankrupt partner of
I. Those owing to partners by way of contribution
II. Those owing to separate creditors
III. Those owing to partnership creditors
a. II first; I second and III third.
b. III first; II second and I third.
c. I first; III second and II third.
d. II first; III second and I third.

6. Hara, Ives, and Jack are in the process of liquidating their partnership. Since it may
take several months to convert the other assets into cash, the partners agree to
distribute all available cash immediately, except for 10,000 that is set aside for
contingent expenses. The balance sheet and residual profit and loss sharing
percentages are as follows:

Cash 400,000 Accounts payable 200,000


Other assets 200,000 Hara, capital (40%) 135,000
Ives, capital (30%) 216,000
Jack, capital (30%) 49,000
Total assets 600,000 Total liab./equity 600,000

How much cash should Ives receive in the first distribution?


a. 146,000.
b. 147,000.
c. 153,000.
d. 156,000

7. Jane, Khel, and Laide are in the process of liquidating their partnership. Laide has
agreed to accept the inventory, which has a fair value of $60,000, as part of her
settlement. A balance sheet and the residual profit and loss sharing percentages are as
follows:
Cash 198,000 Accounts payable 149,000
Inventory 80,000 Jane, capital (40%) 79,000
Plant assets 230,000 Khel, capital (40%) 140,000
Laide, capital (20%) 140,000

Total assets 508,000 Total liab./equity 508,000

If the partners then distribute the available cash, Laide will receive
a. 34,000.
b. 30,000
c. 29,000.
d. 23,000.

8. The year-end balance sheet and residual profit and loss sharing percentages for the
Lang, Maas, and Neal partnership on December 31, 2005, are as follows:
Cash 30,000 Accounts payable 200,000
Loan to Lang 40,000 Loan from Maas 50,000
Other assets 480,000 Lang, capital (25%) 70,000
Maas, capital (25%) 80,000
Neal, capital (50%) 150,000
Total assets 550,000 Total liab./equity 550,000
The partners agree to liquidate the business and distribute cash when it becomes
available. A cash distribution plan for the Lang, Maas, and Neal partnership will show
that cash available, after outside creditors are paid, will initially go to
a. Lang in the amount of $20,000.
b. Maas in the amount of $45,000.
c. Maas in the amount of $55,000.
d. Neal in the amount of $90,000.

9. Assume that a partnership had assets with book value of 480000 and a market value
of 390000, outside liabilities of 140000, loans payable to partner Abby of 40000, and
capital balances for partners Abby, Baby, and Cassy of 140000, 60000, and 100000.
How much would Abby receive upon liquidation assuming profits and losses are
allocated equally?
a. 110000
b. 140000
c. 150000
d. 180000

10. Assume the same situation in problem 9, how would the first 200000 of available
assets be distributed assuming profits and losses are allocated equally?
a. 140000 to outside liabilities, 40000 to Abby, and the balance equally among
partners
b. 140000 to outside liabilities, and 60000 to Abby
c. 140000 to outside liabilities, 50000 to Abby and 10000 to Cassy
d. 80000 to Abby, 40000 to Cassy, and the balance equally among partners

ANSWERS & SOLUTIONS (Chapter 5)

1. b
2. a
3. d
4. a
5. d
6. b
7. d
40% 40% 20%
Jane Khel Laide
Equities 79,000 140,000 140,000
Distribute inventory to Laide and: ( 60,000 )
recognize $20,000 loss ( 8,000 ) ( 8,000 ) ( 4,000 )
Possible losses on plant ( 92,000 ) ( 92,000 ) ( 46,000 )
Subtotal ( 21,000 ) 40,000 30,000
Eliminate Jane’s debit
balance to Khel & Laide 21,000 ( 14,000 ) ( 7,000 )
Balance 0 26,000 23,000

8. c

9. c
10. b
Liabilities should be paid first, then the balance of P60,000 should be given to
Abby since she is the one entitled to the first priority.

CHAPTER 6: Corporate Liquidation

2. A bankruptcy petition filed by a firm’s creditors is


a. a Chapter 7 petition.
b. a petition for liquidation.
c. an involuntary petition.
d. a voluntary petition.

3. A primary difference between voluntary and involuntary bankruptcy petitions is that


a. creditors file the petition in an involuntary filing.
b. trustees are not used in an voluntary filing.
c. voluntary petitions are not subject to review by the bankruptcy court.
d. the debtor corporation files the petition in an involuntary filing.

4. The first-to-last ranking order of priority of the following:


I. stockholder claims
II. unsecured priority claims
III. secured claims
IV. unsecured non-priority claims

in a Chapter 7 bankruptcy case is

a. I,II,IV, and III.


b. III,II,IV, and I.
c. III,I,IV, and II.
d. I,III,II, and IV.

5. Trustees in a bankruptcy cases have the duty to


a. nullify affiliate transactions.
b. relegate tax payments to an unsecured status.
c. call creditor meetings on liquidation proceedings.
d. provide payments to creditors and customers.

8. The following data were taken from the statement of realization and liquidation
of XYZ Corporation for the quarter ended September 30, 2008:

Assets to be realized P 330,000


Assets acquired 360,000
Assets realized 420,000
Assets not realized 150,000
Liabilities to be liquidated 540,000
Liabilities assumed 180,000
Liabilities liquidated 360,000
Liabilities not liquidated 450,000
Supplementary credits 510,000
Supplementary charges 468,000
.
The ending balances of capital stock and retained earnings are P300,000 and
P120,000, respectively. What is the net income (loss) for the period?
a. P168,000
b. P(168,000)
c. P(210,000)
d. P42,000

9. On July 1, 2008, the records of Mr. X, trustee in bankruptcy for B Corporation,


showed the following:

Cash P 57,400
Assets to be realized:
Furnitures 70,000
Buildings 301,000
Machinery 196,000
Copyright 30,800
Liabilities to be liquidated:
Accounts payable 560,000
Notes payable 280,000
Estate Deficit 184,400

During July, Mr. X sold machinery having a book value of P105,000 for P61,600
and sold the copyright for P84,000. Mr. X was paid P9,100 as trustee fee and P147,000
was distributed proportionately to the creditors.

What is the estimated deficiency?


a. (184,800)
b. (176,500)
c. (184,100)
d. (154,800)
10. A review of the assets and liabilities of G Company in bankruptcy on June 30, 2008,
discloses the ff:
(1) A mortgage payable of P118,000, is secured by building valued at P39,000 less
than its book value of P172,000.
(2) Notes payable of P57,000 is secured by furniture and equipment with a book
value of P76,000 that is 3/5 realizable.
(3) Assets other than those referred to have an estimated value of P44,000, an
amount that is 75% of its book value
(4) Liabilities other than those referred to total P91,000, which included claims with
priority of P23,000.

How much was paid to the partially secured creditors?


a. P52,340
b. P48,260
c. P49,380
d. P50,769

ANSWERS & SOLUTIONS (Chapter 6)

1. a
2. c
3. a
4. b
5. d
6. b. 76%
Solution:
BV NRV Classification
Cash 396,000 396,000 Unpledged assets
Receivables (net) 924,000 600,000 Assets pledge to Fully Secured Creditors
Receivables (net) 180,000 Unpledged assets
Inventory 231,000 132,000 Assets pledge to Partially Secured Creditors
Inventory 70,000 Unpledged assets
Prepaid Expenses 3,000 - Unpledged assets
Property and Equipme 900,000 1,050,000 Assets pledge to Partially Secured Creditors
Goodwill 120,000 - Unpledged assets
Salaries Payable 120,000 120,000 Priority Creditors
Accounts Payable 300,000 140,000 Partially Secured Creditors
Accounts Payable 160,000 Unsecured Creditors
Bonds Payable 270,000 270,000 Unsecured Creditors
Interest Payable 18,000 Unsecured Creditors
Bank Loan Payable 1,200,000 1,200,000 Partially Secured Creditors
Interest Payable 30,000 Partially Secured Creditors
Note Payable 594,000 594,000 Fully Secured Creditors
Ordinary shares 240,000
Deficit (150,000) (150,000) Squeeze
Taxes Payable 21,000 Priority Creditors
Trustees Expenses 25,000 Priority Creditors
Total - -

Assets Free Portion < Liabilities Unsecured Priority Deficiency


APFSC-FSC 6,000 < PSC-APPSC 188,000
Unpledged Assets 646,000 < Unsec. Creditors 448,000
< Priority Creditors 166,000
Total Free Assets 652,000 < 636,000 166,000 (150,000)
Squeeze
Recovery Rate = Available Assets / Unsecured Creditors
Recovery Rate = (652-166) / 636
Recovery Rate = 0.76 or 76 %

7. d. 1,324,880
Partially Secured Creditors = 100% of APPSC + ((PSC-APPSC) x Rec. Rate)
= (1050000+132000) + ((1370000-1182000) x 76%)
= 1,324,880

8. b. (168,000)
9. c. (184,100)
10. d. P50,769
(1)PARTNERSHIP FORMATION

1.1 THEORIES.
1. A partnership is a(n):
I. accounting entity.
II. taxable entity. 
A. I only
B. II only
C. Neither I nor II
D. Both I and II

2. On June 30, 2015, a partnership was formed by Mendoza and Lopez. Mendoza contributed cash.
Lopez, previously sole proprietor, contributed noncash assets including a realty subject to
mortgage which was assumed by the partnership. Lopez’s capital account at June 30, 2015
should be recorded at:
a. The fair value of the property on June 30, 2015 less the mortgage payable
b. Lopez’s carrying amount of the property on June 30, 2015
c. Lopez’s carrying amount of the property on June 30, 2015 less the mortgage payable
d. The fair value of the property on June 30, 2015

3. Two individuals who were previously sole proprietors formed a partnership. Property other than
cash which is part of the initial investment in the partnership would be recorded for financial
accounting purposes at the :
a. Proprietors’ book values or the fair value of the property at the date of the investment
whichever is higher.
b. Proprietors’ book values or the fair value of the property at the date of the investment
whichever is lower.
c. Proprietors’ book values of the property at the date of the investment
d. Fair value of the property at the date of investment

4. A unique feature of partnerships (compared with publicly owned corporations) is that:


a. Limited liability with respect to damages arising from professional services
b. Greater allowable tax deductions for retirement plans
c. Ease of formation
d. Book value

1.2 PROBLEMS.
1. On May 1, 2015, Cat and Meow formed a partnership and agreed to share profits and losses in
the ratio of 3:7, respectively. Cat contributed a parcel of land that cost her P10,000. Meow
contributed P40,000 cash. The land has a fair value of P15,000. Cat insisted that the value of
land should be P18,000. The partners agreed to value the land at P18,000. What amount should
be recorded in Cat’s capital account on formation of the new partnership?

a. P18,000 b. P17,400 c. P15,000 d. P10,000


2. On July 1, Manny and Floyd formed a partnership, agreeing to share profits and losses in the
ratio of 4:6, respectively. Manny contributed a parcel of land that cost him P25,000. Floyd
contributed P50,000 cash. The land was sold for P50,000 on July 1, four hours after formation of
the partnership. How much should be recorded in Manny’s capital account on the partnership
formation?

a. P10,000 b. P20,000 c. P25,000 d. P50,000

Use the following question for 3 & 4

On March 1, 2014, cat and Fish formed a partnership with each contributing the following
assets:

Cat Fish
Cash P30,000 P70,000
Machinery P25,000 P75,000
Building - P225,000
Furnitures and Fixtures P10,000 -

3. On March 1, 2015, the capital account of Fish would show a balance of:

a. P280,000 b. P305,000 c. 314,000 d. 370,000

4. Assuming that the partners agreed to bring their respective capital in proportion to their
respective profit and loss ratio, and using Fish capital as the base, how much cash is to be
invested by Cat?

a. P19,000 b. P30,000 c. P40,000 d. P55,000

5. On October 1, 2015, Albano and Armando formed a partnership and agreed to share profits and
losses in the ratio 3:7 respectively. Albano contributed a parcel of land that cost him P2,000,000.
Armando contributed P3,000,000 in cash. The land has a quoted price of P3,600,000 on October
1, 2015. What amount should be recorded in Albano’s capital account upon formation of the
partnership?

a. P3,600,000 b. P3,000,000 c. P3,480,000 d. P2,000,000

(2)PARTNERSHIP OPERATIONS
2.1 THEORIES.
1. The partnership agreement between Moran and Reyes stipulates that Moran is to receive a
20% bonus on profits before bonus, with the residual profit and loss to be apportioned in
the ratio of 2:3 respectively. Which partner has a greater advantage when the partnership
has profits and when it incurred a loss?
a. Profit: Reyes Loss: Moran
b. Profit: Moran Loss: Reyes
c. Profit: Reyes Loss: Reyes
d. Profit: Moran Loss: Moran
2. Partners Fojas and Gomez share profits and losses equally after each has beeen credited in
all circumstances with annual salary allowances of p30,000 and P24,000 respectively. Under
this arrangement, in which following circumstances will Fojas benefit by -6,000 more than
Gomez?

a. Only if the partnership has earnings of P54,000 or more for the year
b. Only if the partnership does not incur loss for the year
c. In all earnings or loss situations
d. Only if the partnership has earnings of at least P6,000 for the year

3. Partners Lazaro and Orlando share profits in a 2:1 ratio, respectively. Each partner receives
an annual salary allowance of P12,000. If the salaries are recorded in the accounts of the
partnership as an expense rather than treated as an allocation of profit, the total amount
allocated to each partner for salaries and net profit would be:

a. Less ofr both lazaro and orlando


b. Unchanged for both Lazaro and Orlando
c. More for lazaro and less for Orlando
d. More for orlando and less for Lazaro

4. Navarro and Paredes formed a partnership on January 2, 2015, and agreed to share income
90% and 10% respectively. Navarro contributed a capital of P50,000. Paredes contributed no
capital but has a specialized expertise and manages the firm full-time. There were no
withdrawals during the year. The partnership agreement provides for the the following:

a. Capital accounts are to be credited anually with the interest at 5% of he beginning


capital balance
b. Paredes is to be paid a salary of P2,000 a month.
c. Paredes is to receive a bonus of 20% of income calculated before deducting his
bonus, salary and interest on both capital accounts
d. Bonus, interest and Paredes’ salary are to be considered partnership expenses

5. What is the underlying purpose of the interest on capital balances component of allocating
partnership profits and losses?
a. Compensate partners who contribute economic resources to the partnership
b. Reward labor and expertise contribution
c. Reward for special responsibilities undertaken
d. None of the above

2.2. PROBLEMS.

1. ABC Partnership provided for the following distribution of profits and losses:
 First, Alberto is to receive 10% of the net profit up to P1,000,0000 and 20% on the
amount in excess thereof;
 Second, Bustos and Cancio each are to receive 5% of the remaining profit in excess of
P1,500,000 after Alberto’s share as per above; and
 The balance is to be divided equally among the partners.
For the year just ended, the partnership realized a net profit of P2,500,000 before distribution to
partners. How much is the share of Alberto in the profit of the partnership?

a. P1,000,000 b. P1,300,000 c. P1,080,000 d. P1,100,000

2. Tamayo, Banson and Vidal, a partnership formed on Jan. 1, 2015 had the following initial
investments:
 Tamayo P100,000
 Banson 150,000
 Vidal 225,000

The partnership agreement stated that the profits and losses are to be shared equally by the
partners after consideration is to be made for the following:

Salaries allowed to partners: P60,000 for Tamayo; P48,000 for Banson and P36,000 fro Vidal.
Average partners’ capital balances during the year shall be allowed 10%.

Additional Information:
 On June 30, 2012, Tamayo invested an additional P60,000.
 Vidal withdrew P70,000 from the partnership on September 30, 2012.
 Share on the remaining profit was P3,000 for each partner.

Interest on average capital balances of the partners totals:

a. P48,750 b. 53,750 c. 57,625 d. 60,625

3. Using the information in #2, the partnership net profit for 2012 before salaries, interest and
partners’ share on the remainder was:
a. 199,750 b.207,750 c. 211,625 d.
201,750
4. Using the information in #2, the total partnership capital on December 31, 2015 was:
a. 405,000 b. 666,750 c. 465,000 d.
480,000

5. Sison, Torres and Velasco are partners in a n accounting firm. Their capital account balances
at year-end were: Sison, P50,000; Torres, P110,000; Velasco, P50,000. They share profits
and losses on a 4:4:2 ratios, after the following terms.
a. Partner Velasco is to receive a bonus of 10% of net profit after bonus
b. Interest of 10% shall be paid on that portion of a partner’s capital in excess of
P100,000.
c. Salaries of P10,000 and P12,000 shall e paid to partners Sison and Velasco,
respectively.
Assuming a net profit of P44,000 for the year, the total profit share of Sison was:
a. 7,800 b. 16,800 c. 19,400 d. 19,800
(3)PARTNERSHIP DISSOLUTION

3.1 THEORIES.

1. Which of the following conditions constitutes a legal dissolution of a partnership?


a. Death of a partner
b. Retirement of a partner
c. Admission of a partner
d. All of the above

2. Ben, Inc., a partner in BenTong Partnership assigns its partnership interest to Bean, who is not
made a partner. After the assignment, Bean asserts the right to:
I. Participate in the management of BenTong
II. Ben’s share of BenTong’s profits
a. I only
b. II only
c. Both I & II
d. Neither I nor II

3. Transferable interest of a partner includes all of the following except:


a. The partner’s share of the profits and losses of the partnership
b. The right to receive distributions
c. The right to receive any liquidating distribution
d. The authority to transact any of the partnership’s operations

4. If existing partners acquires the equity of a withdrawing partner, in what manner do they divide
the equity?
a. In any manner they choose
b. Equally
c. Proportionate to their residual profit and loss ratios
d. Existing partners are not permitted to acquire the equity of a withdrawing partner

3.2 PROBLEMS.

1. Presented below is the condensed balance sheet of the partnership of HH, AA, and MM who
share profits and losses in the ratio of 6:3:1, respectively:
Cash …………………………………… P 85,000 Liabilities …………………………P 80,000
Other Assets ……………………….. 415,000 HH, capital ………………………. 252,000
AA, capital ……………………….. 126,000
________ MM, capital …………………….. 42,000
Total …………………………………..P 500,000 Total ……………………………….P 500,000
The partners agree to sell NN 20% of their respective capital and profit and loss interest for a
total payment of P90,000. The payment of NN is to be made directly to the individual partners.
The capital balances of HH, AA, and MM respectively after admission of NN are:
e. P198,000; P99,000; P33,000.
f. P201,600; P100,800; P33,600.
g. P216,000; P108,000; P36,000.
h. P255,000; P127,800; P42,600.

2. Capital balances and profit or loss sharing ratios of the partners in the BIG Entertainment Gallery
are as follows:
Britney, capital (50%) ………………………. P 140,000
Iggy, capital (30%) ……………………………… 160,000
Grabby, capital (20%) ………………………… 100,000
Total …………………………………………………P 400,000
Betty needs money and agrees to assign half of her interest in the partnership to Yessir for
P90,000 cash. Yessir does not become a partner.

What is the total capital of the BIG partnership immediately after the assignment of the interest
to Yessir?
a. P310,000 b. P200,000 c. P490,000 d. P400,000

3. The capital accounts of the partnership of NN, VV and JJ on June 1, 2015 are presented below
with their respective profit and loss ratio:
NN …………………………………. P 139,200 1/2
VV …………………………………… 208,800 1/3
JJ …………………………………….. 96,000 1/6

On June 1, 2015, LL is admitted to the partnership when LL purchased for P132,000, a


proportionate interest from NN and JJ in the net assets and profits of the partnership. As a result
of a transaction LL acquired a one-fifth interest in the net assets and profits of the firm. What is
the combined gain realized by NN and JJ upon the sale of a portion of their partnership to LL?

a. P 0 b. P43,200 c. 62,400 d. 82,000

4. PP contributed P24,000 and CC contributed P48,000 to form a partnership, and they agreed to
share profits in the ratio of their original capital contribution. During the first of operation, they
made a profit of P116,290; PP withdrew P5,050 and CC P8,000. At the start of the following
year, they agreed to admit GG in the partnership. He was to receive a one-fourth interest in the
capital and profits upon payment of P30,000 to PP and CC, whose capital accounts were to be
reduced by transfer to GG’s capital account of amounts sufficient to bring them back to their
original capital ratio.

How should the P30,000 paid be GG be divided between PP and CC?


a. PP, P9,825; CC, P20,175 c. PP, P10,000; CC, P20,000
b. PP, P15,000; CC, P15,000 d. PP, P9,300; CC, P20,700

5. CC and DD are the partners who share profits and losses in the ratio of 7:3 respectively. On
October 21, 2015, their respective capital accounts were as follows:
CC PHP 35,000.00
DD 30,000.00
Total Capital PHP 65,000.00

On that date they agreed to admit EE as a partner with one-third interest in the capital and
profits and losses, and upon his investment of P25,000. The new partnership will begin with a
total capital of P90,000. Immediately after EE’s admission, what are the capital balances of CC,
DD and EE, respectively.

a. P30,000; P30,000; P30,000 c. P31,667; P28,333; P30,000


b. P31,500; P28,500; P30,000 d. P35,000; P30,000; P25,000

(4)LUMP-SUM LIQUIDATION

4.1THEORIES.

1. The first step in the liquidation process is to


a. Convert noncash assets into cash
b. Pay partnership creditors
c. Compute any net income (loss)
d. Allocate any gains or losses to the partners

2. The following is the priority sequence in which liquidation proceeds will be distributed for a
partnership:
a. Partnership drawings, partnership liabilities, partnership loans, partnership capital
balances
b. partnership liabilities, partnership loans, partnership capital balances
c. partnership liabilities, partnership loans, partnership drawings, partnership capital
balances
d. partnership liabilities, partnership capital balances, partnership loans
3. Offsetting a partner’s loan balance against his debit capital balance is referred to as the:
a. Marshalling of assets
b. Right of offset
c. Allocation of assets
d. Liquidation of assets
4. Which item is not shown on the schedule of partnership liquidation?
a. Current cash balances
b. Property owned by the partnership
c. Liabilities still to be paid
d. Considered to be a liability of a partnership
5. Which of the following statements is correct?
I. Personal creditors have first claim on partnership assets
II. Partnership creditors have first claim on partnership assets
III. Partnership creditors have first claim on personal assets
a. I
b. II
c. III
d. Both II and III

4.2 PROBLEMS.
1. The statement of financial position of the Golf Partnership, just before liquidation, is as follows:

Cash P20,000 Liabilities P24,000


Non-cash Assets 50,000 Par, capital (50%) 20,000
Boogie, capital (30%) 16,000
_______ Birdie, capital (20%) 10,000
Total P70,000 Total P70,000

The noncash assets are sold for P10,000 net of liquidation expenses and the liabilities are paid. The
remaining cash should be distributed to the partners as follows:

Par Boogie Birdie


a. P 0 P4,000 P2,000
b. P2,000 P2,000 P2,000
c. P3,000 P1,800 P1,200
d. P15,000 P9,000 P6,000

2. The statement of financial position for the partnership of Jonas, Carlos, Tomas, whose sharesof
profits and losses are 40, 50, and 10 percent is as follows:

Cash P50,000 Liabilities P150,000


Inventory 360,000 Jonas, capital 160,000
Carlos, capital 45,000
________ Tomas, capital 55,000
Total P410,000 Total Liabilities and Equity P410,000

If the inventory is sold for P300,000, how much should Jonas receive upon the liquidation of the
partnership?
a. P48,000
b. P100,000
c. P136,000
d. P160,000

3. Nory and Oscar started a partnership some years ago and managed to operate profitability for
several years. Recently, however, they lost a substantial legal suit and incurred unexpected
losses on accounts receivable and inventories. As a result, they decided to liquidate. They sold
all the assets and only P18,000 was available to pay liabilities, which amounted to P33,000. Their
capital account balances before the state of liquidation and their profit-sharing ratios are shown
below:
Capital Account Balances Profit-sharing Ratio
Nory P23,000 60%
Oscar P13,500 40%
Nory is personally insolvent after paying the unpaid creditors, but Oscar has personal assets in
excess of P100,000. In the settlement of partners, how cash should Nory receive.

a. P0
b. P7,100
c. P1,700
d. P7,000

4. Mona and Lisa are partners with capital balances, loan balances, and profit and loss ratio as
follows:

Capital Balances Loan Balances Profit and Loss Ratio


Mona P24,500 P4,000 60%
Lisa P15,500 P3,500 40%

The partners decide to liquidate the partnership. The firms liabilities amounted to P36,000
including partners’ loan. After realization of assets, cash on hand amounts to P37,500. In the
settlement of partners, Mona and Lisa should receive:

Mona Lisa
a. P22,500 P15,000
b. P1,500 P1,000
c. P5,400 P3,600
d. P28,500 P19,000

(5)LIQUIDATION BY INSTALLMENT

5.1 THEORIES
1. The rank order is for claims against a bankrupt partner of
IV. Those owing to partners by way of contribution
V. Those owing to separate creditors
VI. Those owing to partnership creditors
a. II first; I second and III third
b. III first, II second and I third
c. I first; III second and II third
d. II first; III second and I third

2. In a partnership liquidation, the final cash distribution to the partners should be made in
accordance with the:
a. Partners’ profit and loss sharing ratio
b. Balances of the partners’ capital accounts
c. Ratio of the capital contributions by the partners
d. Ratio of capital contributions less withdrawals by the partners.

3. An advance cash distribution plan is prepared


a. Each time cash is distributed to partners in an instalment liquidation
b. Each time a partnership asset is sold in an instalment liquidation
c. To determine the order and amount of cash each partner will receive as it becomes
available for distribution
d. None of these

4. If a partner with a debt capital balance during liquidation is personally solvent, the
a. Partner must invest additional assets in the partnership
b. Partner’s debit balance will be allocated to the other partners
c. Other partners will give the partner enough cash to absorb the debit balance
d. Partnership will loan the partner enough cash to absorb the debit balance.

5.2 PROBLEMS.
1. The Statement of Financial position of the firm of RJ, SJ, and TJ just before liquidation shows the
following:
Assets P120,00

Liabilities 50,000
RJ, loan 10,000
RJ, capital 22,000
SJ, capital 30,000
TJ, capital 8,000

Total P120,000

RJ, SJ, and TJ share profits 5:3:2 respectively. Certain assets are sold for P80,000. Creditors are
paid in full, partners are paid P20,000, and cash of P10,000 is withheld pending future
developments. How much cash is to be distributed to partners?
RJ SJ TJ
a. P7,000 P13,000 -
b. P5,750 P14,250 -
c. P5,250 P14,750 -
d. P7,550 P12,450 -

2. The statement of financial position of the firm of AR, BR, CR, and DR, just prior to liquidation
shows:
AR, loan P1,000
AR, capital 5,500
BR, capital 5,150
CR, capital 6,850
DR, capital 4,500

AR, BR, CR, and DR share profits 4:3:2:1 respectively. Certain assets are sold for P6,000 and this
is distributed to partners. How much cash should CR receive?
a. P3,283
b. P0
c. P2,717
d. P6,000
*The following statement of financial position was prepared for the Tan, Lim and Wan
Partnership on March 31, 2014 and relates to items 3-5.

Assets Liabilities and Equity

Cash P 25,000 Liabilities P52,000


Other Assets 180,000 Tan, capital (40%) 40,000
Lim, capital (40%) 65,000
Wan, capital (20%) 48,000
Total Assets P205,000 Total Liabilities and Equity P205,000
3. The partnership is being liquidated by the sale of assets in instalments. The first sale of noncash
assets having a book value of P90,000 realizes P50,000.

The amount of cash each partner should receive in the first instalment is:

a. Tan, P0; Lim,P5000; Wan, P18,000


b. Tan,P12,000; Lim, P13,000; Wan, P22,000
c. Tan,P27,000; Lim, P5,000; Wan, P18,000
d. Tan, P0; Lim, P5,000; Wan, P22,000

4. If P3,000 cash is withheld for possible liquidation expenses, how much cash should Wan
receive?
a. P21,000
b. P17,000
c. P 3,000
d. P15,000

5. As a separate case, assume that each partner properly received some cash after the second sale
of assets. The cash to be distributed amount to P14,000 from the third sale of assets, and unsold
assets with P6,000 book value remain. How should the P14,000 be distributed to Tan, Lim and
Wan, respectively?
a. P5,600; P6,500; P2,800.
b. P5,000; P5,000; P4,000.
c. P - ; P11,200; P2,800.
d. P5,600; P5,600; P2,800.

(6)CORPORATE LIQUIDATION

6.1. THEORIES.

1. In a Statement of Affairs, assets pledged for partially secured creditors are:


a. Included with assets pledges for fully secured creditors
b. Offset against partially-secured liabilities
c. Included with free assets
d. Disregarded
2. The preferred sequence of listing (1) fully-secured liabilities, (2) partially-secured liabilities, (3)
unsecured liabilities with priority, and (4) unsecured liabilities without priority in the liabilities
and stockholders’ equity section of a Statement of Affairs is:
a. (1), (2), (3), and (4) c. (1), (3), (2), and (4)
b. (3), (1), (2), and (4) d. (1), (3), (4), and (2)

3. The estimated amount available for Free Assets in the Statement of Affairs for a business
enterprise undergoing bankruptcy liquidation is equal to the assets’:
a. Carrying amounts less current fair values
b. Carrying amounts plus gain or less loss on realization
c. Carrying amounts plus loss or less gain on realization
d. Current fair values less carrying amounts

4. In corporate liquidation, creditors having priority are what type of creditors?


Secured Creditors? Unsecured Creditors?
a. Yes Yes
b. Yes No
c. No Yes
d. No No

5. In reporting a company that is to be liquidated, assets are shown at:


a. Present value calculated using an appropriate effective rate
b. Net realizable value
c. Historical cost
d. Book value

6.2. PROBLEMS.

1. When Philex Company filed for liquidation with the Securities and exchange Commission, it
prepared the following balance sheet:
Current assets, net realizable value, P50,000 P 80,000
Land and Buildings, fair value, P240,000 200,000
Goodwill, fair value, P0 40,000
Total assets P 320,000

Accounts payable P 160,000


Mortgage payable, secured by land and building 200,000
Common stock 100,000
Retained earnings, deficit (140,000)
Total Eqquities P 320,000

What is the estimated deficiency to unsecured creditors?


e. P70,000 b. P90,000 c. P120,000 d. P140,000

2. What percentage of their claims are the unsecured creditors likely to get?
a. 43.75& b. 50% c. 56.25% d. 100%
*Items 3-5 are based in the following:
The following data were taken from the statement of affairs of RG Corp.:

Assets pledged for fully secured liabilities (current fair value, P75,000) P90,000
Assets pledged for partially secured liabilities (current fair value, P52,000) 74,000
Free Assets (current fair value, P40,000) 70,000
Unsecured Liabilities with priority 7,000
Fully secured liabilities 30,000
Partially secured liabilities 60,000
Unsecured liabilities without priority 112,000

3. The amount that will be paid to creditors with priority is:


a. P7,000 b. P6,000 c. P7,500 d. P6,200
4. The amount to be paid to fully secured creditors is:
a. P30,000 b. P32,000 c.P20,000 d. P35,000
5. The amount to be paid to partially secured creditors is:
a. P52,700 b. 57,200 c. 56,200 d. 57,000

Problems

Chapter 1

Partnership Formation

1. The business assets of LL and MM appear below:

LL MM
Cash P 11,000 P 22,354
Accounts Receivable 234,536 567,890
Inventories 120,035 260,102
Land 603,000 -
Building - 428,267
Furniture and Fixture 50,345 34,789
Other Assets 2,000 3,600
Total P 1,020,916 P 1,317,002

Accounts Payable P 178,940 P 243,650


Notes Payable 200,000 345,000
LL, capital 641,976 -
MM, capital - 728,352
Total P 1,020,916 P 1,317,002
LL and MM agreed to form a partnership by contributing their respective assets and equities
subject to the following adjustments:
a. Accounts Receivable of P20,000 in LL’s books and P35,000 in MM’s are uncollectible.
b. Inventories of P5,500 and P6,700 are worthless in LL’s and MM’s respective books.
c. Other assets of P2,000 and P3,600 in LL’s and MM’s respective books are to be written off.
The capital account of the partners after the adjustments will be:
a. LL: P615,942, MM: P717,894
b. LL: P614,476, MM: 683,052
c. LL: P640,876, MM: P712,345
d. LL: P640,876, MM: 683,050
2. The same information in number 2, how much total assets does the partnership have after
formation?
a. P2,265,118
b. P2,337,918
c. P2,237,918
d. P2,365,218
3. On March 1, 2015, PP and QQ decide to combine their businesses and form a partnership. Their
balance sheets on March1, before adjustments, showed the following:

PP QQ
Cash P 9,000 P 3,750
Accounts Receivable 18,500 13,500
Inventories 30,000 19,500
Furniture and Fixtures (net) 30,000 9,000
Office Equipment (net) 11,500 2,750
Prepaid Expenses 6,375 3,000
Total P 105,375 P51,500

Accounts Payable P45,750 P18,000


Capital 59,625 33,500
Total P105,375 P51,500

They agreed to have the following items recorded in their books:


1. Provide 2% allowance for doubtful accounts.
2. PP’s furniture and fixtures should be P31,000, while QQ’s office equipment is
underdepreciated by P250.
3. Rent expense incurred previously by PP was not yet recorded amounting to P1,000, while
salary expense incurred by QQ was not also recorded amounting to P800.
4. The fair market value of inventory amount to:
For PP ...............................P29,500
For QQ ...............................P21,000
Compute the net (debit) credit adjustment for PP and QQ:
PP QQ
a. P 2,870 P 2,820
b. (2,870) (2,820)
c. 870 180
d. (870) 180
5. The same information in number 4, compute the total liabilities after formation:
a. P 65,550
b. 61,950
c. 63,750
d. 63,950

Chapter 2

Partnership Operation
1. Aldub is trying to decide whether to accept a salary of P40,000 or a salary of P25,000 plus a
bonus of 10% of net income after salary and bonus as a means of allocation profit among
partners. Salaries traceable to ther other partners are estimated to be P100,000. What amount
of income would be necessary so that Aldub would consider the choices to be equal?
a. P165,000
b. 265,000
c. 290,000
d. 305,000
2. The partnership agreement of Boo, Chi and Kik provides for the year-end allocation of net
income in the following order:
 First, Boo is to recieve 10% of net income up to P200,000 and 20% over P200,000
 Second, Chi and Kik each are to receive 5% of the remaining income over P300,000
 The balance of income is to be allocated equally among the three partners

The partnership’s 2011 net income was P500,000 before any allocations to partners. What
amount should be allocated to Boo?

a. P202,000
b. 220,000
c. 206,000
d. 216,000
3. Chin, San and Soo are partners with average capital balances during 2015 of P360,000,
P180,000, and P120,000, respectively. Partners receive 10% interest on their average capital
balances. After deducting salaries of P90,000 to Chin and P60,000 to Soo the residual profit or
loss is divided equally. In 2015 the partnership sustained a P99,000 loss before interest and
salaries to partners. By what amount should Chin’s capital account change?
a. P126,000 increase
b. 105,000 decrease
c. 33,000 decrease
d. 21,000 increase
4. Star and Circle created a partnership to own and operate a store. The partnership agreement
provided that Star recieve a salary of P10,000 and Circle a salary of P5,000 to recognize their
relative time spent in operating the store. Remaining profit and losses were divided 60:40 to
Star and Circle, respectively. Income for 2015, the first year of operations, of P13,000 was
allocated P8,800 to Star and P4,200 to Circle.
On January 1, 2016, the partnership agreement was changed to reflect the fact that Circle could
no longer devote any time to the store’s operation. The new agreement allows Star a salary of
P18,000, and the remaining profits and losses are divided equally. In 2016 an error was
discovered such that the 2015 reported income was understated by P4,000. The partnership
income of P25,000 for 2016 included the P4,000 related to year 2015.
In the reported net income of P25,000 for the year 2016, Star and Circle would have:
a. Star: 0, Circle: 0
b. Star: 12,500, Circle:12,500
c. Star: 21,900, Circle: 3,100
d. Star: 17,100, Circle: 17,100

Chapter 3

Partnership Dissolution: Changes in Ownership


1. Spaghetti and Hamburger entered into a partnership on May 31, 2015, contributing cash of
P48,000 and P32,000, respectively, and agreeing to divide earnings in the ratio of their initial
investment after allowing annual salary allowances of P12,000 each. On December 31, 2015, the
Income Summary account had a credit balance of P34,000, while the drawing accounts showed
debit balances of P14,000 for Spaghetti and P10,000 for Hamburger.
At the beginning of the next year, Pizza was admitted into the firm as a new partner with a 33
1/3% interest for a capital credit equal to his cash investment of P60,000. Spaghetti and
Hamburger then effected a private cash settlement between themselves in order to make the
capital balances conform to a new profit-sharing ratio 4:2:3, respectively, with salary allowances
scrapped.
How much of the amount of the private cash settlement effected between the old partners?
a. P15,000
b. 12,000
c. 9,000
d. 5,000
2. The following are capital account balances and profit and loss ratios of the partners in Melon
Company.
Capital P&L Ratio
LL P2,250,000 2
OO 750,000 1

They agree to admit RR as a partner with a 25% in capital upon her investment of P1,000,000.
LL, OO and RR are to share profits 5:3:2, respectively.

Subsequently, TT joins the partnership by investing P1,200,000 for a 20% interest in profits and
capital, the old partners are to share profits in their original ration.

Assuming the goodwill method is used, how much is the goodwill to be recorded upon the
admission of TT?

a. P400,000
b. P600,000
c. P800,000
d. P240,000
3. The condensed balance sheet of the partnership of Vice, Anne and Vhong with the
corresponding profit and loss sharing percentage as of June 30, 2015 was as follows:

Net assets P480,000

Vice, capital P240,000


Anne, capital 144,000
Vhong, capital 96,000
P 480,000
As of said date, Vice retired from the partnership. By mutual agreement, he was paid P270,000
for his interest in the partnership. Partial goodwill or adjustment in assets was to be recorded.
After Vice’s retirement, the total net assets of the partnership was:
a. P210,000
b. 300,000
c. 240,000
d. 270,000
4. Selena Gomez, a partner in a accounting firm, decided to withdraw from the partnership,
Selena’s share of the partnership profits and losses was 20%. Upon withdrawing from the
partnership he was paidP88,000 in final settlement for his interest. The total of the partners’
capital accounts before recognition of partnership goodwill prior to Selenna’s withdrawal was
P252,000. After his withdrawal the remaining partners’ capital accounts, excluding their share of
goodwill, totalled P192,000. The total goodwill of the firm was:
a. P300,000
b. P168,000
c. P144,000
d. P192,000
5. Choco and Chips are partners who share profit and losses in the ratio of 7:3, respectively. On
October 21, 2012, their respective capital accounts were as follows:
Choco P 35,000
Chips P 30,000
P 65,000
On that date they agreed to admit Mallows as a partner with a one-third interest in the capital
and profits and losses, and upon his investment of P 25,000. The new partnership will begin with
a total capital of P 90,000. Immediately after Mallows’ admission, what are the capital balance
of Choco, Chips and Mallows, respectively?
a. P31,667; P28,333; P30,000
b. P31,500; P28,500; P30,000
c. P30,000; P30,000; P30,000
d. P35,000; P30,000; P25,000

Chapter 4

Partnership Lump-Sum Liquidation


1. Kah, Nih and Wang have capital balances of P90,000, P60,000 and P30,000, respectively. Profits
are allocated 35% to Kah, 35% to Nih, and 30% to Wang. The partnershave decided to dissolve
and liquidate the partnership. After paying ll the creditors, the amount available for distribution
is P60. Kah, Nih and Wang are personally insolvent. Under the circumstances, Wang will
a. Personally have to contribute an additional P6,000
b. Receive P18,000
c. Receive P30,000
d. Personally have to contribute an additional P36,000
2. The following account balances were available for the RFM partneship just before it entered
liquidation:

Cash P 10,000 Liabilities P 130,000


Noncash assets 300,000 R, capital 60,000
F, capital 40,000
M, capital 80,000

P 310,000 P 310,000

R, F and M share profits


and losses in a ratio of
2:4:4. Noncash assets were sold for P180,000. Liquidation expenses were P10,000. Assume that R
was personally insolvent with assets of P8,000 and liabilities of P60,000. F and M were both solvent
and able to cover deficits in their capital accounts, if any. What amount of cash could R’s personal
creditors have expected to receive from partnership assets?
a. P 34,000
b. P 0
c. P 26,000
d. P 30,000
Use the information for question 3 and 4
A local partnership was considering the possibility of liquidation since one of the partners (Dang)was
insolvent. Capital balances at that time were as follows. Profits and losses were divided on a 4:2:2:2
basis, respectively.
Dang P 60,000
Deng P 67,000
Ding P 17,000
Dong P 96,000
3. Dang’s creditor filed a P25,000 claim against the partnership assets. At that time, the
partnership held assets reported at P360,000 and liabilities of P120,000. If the assets could be
sold for P228,000, what is the minimum amount that Dang’s creditor would have recieved?
a. P -0-
b. P 2,500
c. P 36,000
d. P 38,250
4. Dang’s creditor filed a P25,000 claim against the partnership assets. At that time, the
partnership held assets reported at P360,000 and liabilities of P120,000. If the assets could be
sold for P228,000, what is the minimum amount that Dong’s creditor would have recieved?
a. P2,000
b. P-0-
c. P36,000
d. P39,250
e. P67,250
5. A, B and C is considering possible liquidation because C is insolvent. The partners have the
following capital balances: P60,000, P70,000 and P40,000, respectively and share profits and
losses 30%, 45% and 25%, respectively, the partnership has P200,000 in assets that can be sold
for P150,000. What is the minimum that C’s creditors would receive of they have filed a claim of
P50,000?
a. P -0-
b. P47,500
c. P45,000
d. P27,500
e. P50,000
Chapter 5

Partnership Installment Liquidation

1. The MiMoMu Partnership is being disolved. All liabilities have been paid and remaining assets
are being realized gradually. The equity of the partners is as follows:
Partners’ Loans to Profit and
Accounts (from) Loss Ratio
Partnership
Mi P 24,000 6,000 3
Mo 36,000 - 3
Mu 60,000 (10,000) 4
The second cash payment to any partner(s) under a program of priorities shall be made thus:
a. To Mu, P8,000
b. To Mo, P6,000
c. To Mu, P2,000
d. To Mo, P6,000 and Mu, P8,000
2. When Mickey and Minnie, partners who share earning equally. Were incapacitated in an
airplane accident, a liquidator was appointed to wind up thei business. The accounts showed
cash, P 35,000; other assets, P 110,000; liabilities, P 20,000; Mickey, capital, P 71,000; and
Minnie, capital, P 54,000. Because of highly specialized nature of the noncash assets, the
liquidator anticipated that considerable time would be required to dispose them. The expenses
of liquidating the business (advertising, rent, travel, etc) are estimated at P10,000.

How much cash can be distributed safely to each partner at this point?
a. Mickey: P5,000; and Minnie: P0
b. Mickey: P3,000; and Minnie: P0
c. Mickey: P5,000; and Minnie: P500
d. Mickey: P5,000; and Minnie: P1,000
3. A, B, and C are partners in ABC Partnership and share profits and losses 50%, 30% and 20%,
respectively. The partners have agreed to liquidate the partnership and some liquidation
expenses to be incurred. Prior to the liquidation, the partnership balance sheet reflects the
following book values:

Cash P25,200
Noncash assets 297,600
Notes Payable to CC 38,400
Other Liabilities 184,800
A, capital 72,000
B, capital deficit (12,000)
C, capital 39,600
Assuming that the actual liquidation expenses are P16,800 and that the noncash assets with a
book vaule of P240,000 are sold for P216,000.

How much cash should C receive?

a. -0-
b. P39,600
c. P46,457
d. P74,571

Chapter 6

Corporate Liquidation
1. Pink, a CPA, has prepared a statement of affairs. Assets which there are no claims or liens
are expected to produce P70,000, which must be allocated to unsecured claims of all classes
totalling P105,000. The following are some of the claims outstanding:
 Accounting fees for Pink, P1,500.
 An unrecorded note for P1,000, on which P60 of interest has accrued, held by Amy.
 A note for P3,000 secured by P4,000 receivables, estimated to be 60% collectible
held by Joy.
 A P1,500 note , on which P30 of interest has accrued, held by Joyots. Property with
a book value of P1,000 and a market value of P1,800 is pledged to guarantee
payment of principal and interest.
 Unpaid income taxes of P3,500.

Compute the estimated payment to partially secured creditors:

a. P2,490
b. P1,950
c. P2,790
d. P1,060
2. The creditors of Miswa Corporation agreed to a liquidation based on the statement of
affairs, suggested that unsecured creditors, without priority would receive approximately
P.60 on the peso. The unsecured creditors are interested in determining whether the
preliminary estimate still seems appropriate. The trustee was originally assigned noncash
assets of P1,480,000 and creditors claims as follows: fully secured, P670,000; partially
secured, P400,000; unsecured with priority, P200,000, and unsecured without proirity,
P320,000. Assets with a book value of P45,000 and unsecured liabilities(without priority) of
P35,000 were subsequently discovered. Assets with a total book value of P410,000 and
partially secured liablities of P280,000 were paid. Remaining liquidation expenses were
estimated to be P30,000.
Assume the remaining noncash assets have an estimated net realizable value as follows:
Assets traceable to fully secured creditors P 240,000
Assets traceable to partially secured creditors 110,000
Remaining assets 382,000
Determine the revised estimate of the dividend to be recieved by unsecured creditors
without priority:
a. 45.97%
b. 66.17%
c. 100%
d. Cannot be determined
3. Lugi KNB Corporation has been undergoing liquidation since January 1. As of March 31, its
condensed statement of realization and liquidation is presented below:

Assets
Assets to be realized P1,375,000
Assets acquired 750,000
Assets realized 1,200,000
Assets not realized 1,375,000
Liabilities
Liabilities liquidated P1,875,000
Liabilities not liquidated 1,700,000
Liabiities to be liquidated 2,250,000
Liabilities assumed 1,625,000
Revenues and Expenses
Supplementary Charges P3,125,000
Supplementary Credits 2,800,000

The net gain (loss) for the three-month period ending March 31 is:

a. P750,000
b. 425,000
c. (325,000)
d. 250,000
4. Using the same information on number 3, compute the ending cash balance of cash account
assuming that common stock and deficits are P1,500,000 and P500,000, respectively.
a. P1,325,000
b. 1,375,000
c. 425,000
d. 575,000
5. Beyonce, Inc.purchased a Cadillac automobile with little cash down and signed a note,
secured by the Cadillac, for 48 easy monthly payments. When company files for
bankcruptcy, the balance due on the Cadillac amounts to P6,000,000. The car has book
value of P8,000,000 and a net realizable value of P4,000,000. The unsecured creditors of
Beyonce can expect to receive 50 percent of their claims. In the liquidation, the bank that
holds the note on the Cadillac should receive:
a. P4,000
b. 3,000
c. 6,000
d. 5,000
Theories

Chapter 1

1. A unique feature of partnerships (compared with publicly owned corporation) is that


a. They do not have to follow GAAP
b. They are not governed by laws
c. Books have to maintained on the tax basis
d. They do not file income tax return

2. The advantages of the partnership form of business organization, compared to corporations


include
a. single taxation

b. ease of formation

c. mutual agency

d. limited liability

3. The drawing ledger accounts of assets from a limited liability partnership that is considered a
permanent reduction in that partner’s equity is debited to partner’s

a. drawing’s account

b. retained earnings account

c. capital account

d. loan receivable account

4. which of the statements are true when comparing corporations and partnership?

a. partnership entities provide for taxes at the same rates used by corporations

b. in theory, partnership are more able to attract capital

c. like corporations, partnerships have an infinite life

d. unlike shareholders, general partners may have liability beyond their capital balances

5. which of the following is not an advantage of a partnership over corporation?

a. ease of formation

b. unlimited liability

c. the elimination of taxes at the entity level

d. all of the above

Chapter 2

1. What is the underlying purpose of the interest on capital balances component of allocating
partnership profits and losses?

a. compensate partners who contribute economic resources to the partnership

b. Reward labor and expertise contributions

c. Reward for special responsibilities undertaken


d. none of the above
2. If partnership agreement provides a formula for the computation of a bonus to the partners, the
bonus would be computed

a. next to last, because the final allocation Is the distribution of the profit residual

b. before income tax allocation are made

c. after the salary and interest allocations are made

d. in any manner agreed to by the partners

3. Partners active in a partnership business should have their share of partnership profits based on
the following

a. a combination of salaries plus interest based on average capital balances

b. a combination of salaries and percentage of net income after salaries and any other
allocation basis

c. salaries only

d. percentage of net income after salaries is paid to inactive partners

4. Which of the following statements is true concerning the treatment of salaries in partnership
accounting?

a. partner salaries may be used to allocate profits and losses; they are not considered
expenses of the partnership

b. partner salaries are equal to the annual partner now

c. the salary of a partner is treated in the same manner as salaries of corporate employees

d. partner salaries are directly closed to the capital account

5. what is the underlying purpose of the salary component of allocating partnership profits and
losses?

a. compensate partners who contribute economic resources to the partnership

b. Reward labor and expertise contributions

c. Reward for special responsibilities undertaken


d. none of the above

Chapter 3

1. Who may acquire the ownership interest of a partner who is withdrawing from the
partnership?

a. Existing partners
b. new investor
c. the partnership
d. all of the above

2. The dissolution of a partnership occurs

a. only when the partnership sells its assets and permanently closes its books
b. only when a partner leaves the partnership
c. at the end of each year, when income is allocated to the partners
d. only when a new partner is admitted to the partnership
e. when there is any change in the individuals who make up the partnership

3. Which of the following results in dissolution of a partnership?

a. Contribution of additional assets to the partnership by any existing partner


b. Receipt of a draw by an existing partner
c. Winding up of the partnership and the distributing of remaining assets to the partners
d. Withdrawal of a partner from a partnership

4. Which of the following forms of new partner admission will not result in a change in the
partnership’s net assets?

a. purchase of an ownership interest directly from the partnership


b. purchase of an ownership interest directly from an existing partner
c. either of the above
d. neither of the above

5. Which of the following is not a criterion for recognizing a bonus to existing partners when a
new partner joins partnership?

a. Only cash assets were contributed to the partnership by the new partner
b. The existing partners desire to not recognize goodwill on the balance sheet
c. The articles of partnership indicates that the bonus method will be used to admit new
partners
d. The new partner invest more into the partnership that his/her share total partnership
capital after the investment is made

Chapter 4

1. Which of the following statements is correct?


1. Personal creditors have first claim on the partnership assets
2. Partnership creditors have the first claim on partnership assets
3. Partnership creditors have the first claim on personal assets
a. 1
b. 2
c. 3
d. Both 2 and 3
2. A partnership dissolution differs from liquidation in that
a. Payments are made to creditors before partners receive value
b. Periodic payments to partners are made when cash becomes available
c. A partner withdraws from the business and the enterprise continues to
function
d. Full payment is made to all outside creditors before remaining cash distributed
to partners in a final lump sum payment
3. The following is the priority sequence in which liquidation proceeds will be distributed for a
partnership:
a. Partnership drawings, partnership liabilities, partnership loans, partnership capital
balances
b. Partnership liabilities, partnership loans, partnership capital balances
c. Partnership liabilities, partnership loans, partnership drawings, partnership capital
balances
d. Partnership liabilities, partnership capital balances, partnership loans
4. If a partners with a debit capital balance during liquidation is personally solvent, the
a. Partner must invest additional assets in the partnership
b. Partner’s debit balance will be allocated to the other partners
c. Other partners will give the partner enough cash to absorb the debit balance
d. Partnership will loan the partner enough cash to absorb the debit balance

Chapter 5

1. Which of the following statement is correct with regard to a partnership liquidation ?


a. All creditors must be paid in full before distributions can be made to partners
b. partner capital contributions and undistributed partnership income are viewed as distinct
in the uniform partnership act
c. all creditors are equal with regard to priority of claim against partnership assets
d. loans from partners to the partnership have the same priority of claim against partnership
assets as to creditor’s claims from other entities
2. Which of the following describes a partnership installment liquidation?
a. Keeping the partnership assets and liabilities separate from the partner’s personal assets
and liabilities
b. The sale of all noncash assets and payment of liabilities before a single distribution to
partners
c. A series of interim distributions to partners while the sale of noncash assets and the
payments of liabilities is occurring
d. The combining of a partner’s capital account with loans to/from the partnership

Chapter 6

1. A corporation that is unable to pay its debts as they become due is:
a. Bankrupt
b. Overdrawn
c. Insolvent
d. Liquidating
2. The duties of the trustee includes:
a. Appointing creditors’ committees in liquidation cases
b. Approving all payments for debts incurred before the bankruptcy filing
c. Examining claims and disallowing any that are improper
d. Calling a meeting of the debtor’s creditors
3. A debtor may file which type of petition when seeking judicial protection under the bankruptcy
law?
I. Voluntary
II. Involuntary
a. I only
b. II only
c. Either I or II
d. Neither I or II
4. The statement of realization and liquidation differs from the statement of affairs because
a. The statement of realization and affairs reports estimated realizable value s rather than
actual liquidation result
b. The statement of realization and affairs is a summary of secured debt activity only
c. The statement of realization and affairs ids prepared only at final completion of the
liquidation process
d. The statement of realization and affairs reports actual liquidation results rather than
estimated realizable values

5. On a balance sheet prepared for a company during its reorganization, at what balance are
liabilities reported?

a. As current and long term


b. As monetary and nonmonetary
c. As subject compromise and not subject to compromise
d. As equity related and debt related

Jessyreen V. Mallero
Leilani M. Baclay

Chapter 1
Partnership Formation

1. Which of the following is not a characteristic of most partnership?


a. Limited liability
b. Limited life
c. Mutual agency
d. Ease of formation
Suggested answer (a) limited liability
The liability of the partners in a partnership is unlimited.

2. Which of the following is not a characteristic of the proprietary theory that influences accounting for
partnerships?
a. Partner’s salaries are viewed as a distribution of income rather than a component of net
income.
b. A partnership is not viewed as separate entity, distinct, taxable entity.
c. A partnership is characterized by limited liability.
d. Changes in the ownership structure of a partnership result in the dissolution of the
partnership.
Suggested answer (c) A partnership is characterized by limited liability

3. An advantage of the partnership as a form of business organization would be


a. Partners do not pay income taxes on their share in partnership income
b. A partnership is bound by the act of the partners
c. A partnership is created by mere agreements of the partners
d. A partnership may be terminated by the death or withdrawal of a partner
Suggested answer (c) A partnership is created by mere agreements of the partners

4. When property other than cash is invested in a partnership, at what amount should the noncash
property be credited to the contributing partner’s capital account?
a. Fair value at the date of contribution
b. Contributing partner’s original cost
c. Assessed valuation for property tax purposes
d. Contributing partner’s tax basis
Suggested answer (a)
Fair value at the date of contribution

5. Partnership capital and drawings accounts are similar to the corporate


a. Paid in capital, retained earnings, and dividends accounts
b. Retained earnings account
c. Paid in capital and retained earnings accounts
d. Preferred and common stock accounts
Suggested answer (a)
Partnership capital accounts are similar to corporate paid in capital and retained earnings; while
partnership drawing accounts are similar to corporate dividends accounts

10. During the first year of their operations, the partnership earned P325,000. Profits were distributed in
the agreed manner. Drawings were made in these amounts: Jessyreen, p50,000; Leilani, 65,000;
Shamira, P28,00.
How much are the capital balances after the first year?
a. Jessyreen, capital 750,627
Leilani, capital 735,177
Shamira, capital 372,223
b. Jessyreen, capital 728,764
Leilani, capital 713,764
Shamira, capital 361,382
c. Jessyreen, capital 757,915
Leilani, capital 742,315
Shamira, capital 375, 837
d. Jessyreen, capital 743,121
Leilani, capital 727,825
Shamira, capital 368,501

Suggested answer (b)


Jessyreen Leilani Shamira
Capital balances at P648764 P648764 P324382
40:40:20 ratio
Drawings (50000) (65000) (28000)
Share in profit (40:40:20) 130000 130000 65000
Capital balances P728764 P713764 P361382

Chapter 2
Partnership Operation
1. If the partnership agreement does not specify how income is to be allocated, profits and loss should
be allocated
a. equally
b. in proportion to the weighted average of capital invested during the period
c. equitably so that partners are compensated for the time and effort expended on behalf of
the partnership
d. in accordance with their capital contribution
Suggested answer (d)

2. Which of the ff. is not a component of the formula used to distribute income?
a. salary allocation to those partners working
b. after all other allocation, the remainder divided according to the P/L sharing ratio
c. interest on the average capital investments
d. interest on notes to partners
Suggested answer (d)

3. Which of the ff. is not considered a legitimate expense of a partnership?


a. interest paid to partners based on the amount of invested capital
b. depreciation on assets contributed to the partnership by partners
c. salaries for management hired to run the business
d. supplies used in the partners’ offices.
Suggested answer (a)

4. The fact that salaries paid to partners are not a component of partnership income is indicative of
a. a departure from generally accepted accounting principles
b. being characteristic of the entity theory
c. being characteristic of the proprietary theory
d. why partnerships are characterized by unlimited liability
Suggested answer (c)

5. A partner who contributes money or property as well as his work or industry to the capital of the
partnership is called
a. industrial partner
b. capitalist partner
c. managing partner
d. capitalist-industrial partner
Suggested answer (d)

6. The ABC partnership reports net income of P60000. If partners A, B, and C have income ratio of 50%,
30%, and 20%, respectively. What is the share of Partner C from the net income of the partnership, if he
was given a capital ratio of 25%?
a. 30000
b. 12000
c. 18000
d. 15000
Suggested answer (b)
60,000 x20% = 12,000
For question 7-8:
In the calendar year 2015, the partnership A and B realized a net profit of P240000. The capital accounts
of the partners show the ff. postings:
A, capital B, capital
Debit Credit Debit Credit
Jan. 1 P120000 P80000
May 1 P20000 P10000
July 1 P20000
Aug. 1 P10000
Oct. 1 P10000 P5000

7. If the profits are to be divided based on average capital, the share of A and B, respectively are:
a. 129600 110400
b. 144000 96000
c. 136800 103200
d. 136543 103457
Suggested answer (d)
A, Capital:
Date Balances Months Unchanged Total
Jan. 1 120,000 4 480,000
May 1 100,000 3 300,000
Aug. 1 110,000 2 220,000
Oct. 1 100,000 3 300,000
Total 12 1,300,000
Ave. Capital – A = 1,300,000/12 = 108,333
B, Capital:
Date Balances Months Unchanged Total
Jan. 1 80,000 4 320,000
May 1 70,000 2 140,000
Ju;y 1 90,000 3 270,000
Oct. 1 85,000 3 255,000
Total 12 985,000
Ave. Capital – A = 985,000/12 = 82,083
A 240,000 x (108,333/190,417) 136,543
B 240,000 x (82,083/190,417) 103,457
Total 240,000

9. During 2015, Young and Zinc maintained average capital balances in their partnership of P160000 and
P100000, respectively. The partners receive 10% interest on average capital balances, and residual profit
or loss is divided equally. Partnership profit before interest was P4000. By what amount should Zinc’s
capital account change for the year?
a. 1000 decrease
b. 2000 increase
c. 11000 decrease
d. 12000 increase
Suggested answer (a)
Young Zinc Total
10% interest on ave. capital:
(10% x 160,000) 16,000
(10% x 100,000) 10,000 26,000
Balance (equally) (11,000) (11,000) (22,000)
Total 5,000 (1,000) 4,000

10. Maxwell is trying to decide whether to accept a salary of P40000 or salary of P25000 plus a bonus of
10% of net income after salaries and bonus as a means of allocating profit among partners. Salaries
traceable to the other partners are estimated to be P100000. What amount of income would be
necessary so that Maxwell would consider choices to be equal?
a. 165000
b. 290000
c. 265000
d. 305000
Suggested answer (b)
Amount of bonus and salaries (40,000-25,000) 15,000

Net Income after bonus and salaries (15,000/10%) 150,000


Multiply by 110%
Net Income before bonus but after salaries 165,000
Add: Salaries (100,000 + 25,000) 125,000
Net Income before bonus & salaries 290,000

Chapter 3
Partnership Dissolution: Changes In Ownership

1. If a new partner acquires a partnership interest directly from the partners rather than from the
partnership itself,
a. No entry is required
b. The partnership assets should be revalued
c. The existing partner’s capital accounts should be reduced and the new partner’s account
increased
d. The partnership has undergone a quasi-reorganization
Suggested answer (c)

2. Which of the following results in dissolution of a partnership?


a. The contribution of additional assets to the partnership by an existing partner
b. The receipt of a draw by an existing partner
c. The winding up of the partnership and the distribution of remaining assets to the partners
d. The withdrawal of a partner from a partnership
Suggested answer (d)

3. When a new partner is admitted to a partnership, an original partner’s capital account may be
adjusted for
a. A proportionate share of the incoming partner’s investment
b. His or her share of previously unrecorded intangible assets traceable to the original partners
c. His share of previously unrecorded intangible assets traceable to the incoming partner
d. None of the above
Suggested answer (b)

4. If goodwill is traceable to the previous partners, it is


a. Allocated among the previous partners according to their interest on capital
b. Allocated among the previous partners only if there are no other assets to be revalued
c. Allocated among the previous partners according to their original profit and loss sharing
percentage
d. Not possible for goodwill to also be traceable to the incoming partner
Suggested answer (c)

5. The goodwill & the bonus methods are two means of adjusting for differences between the net book
value and the fair market value of partnership when new partners are admitted. Which of the ff.
statements about these methods is correct?
a. the bonus method does not revalue assets to market values
b. the bonus method revalues assets to market values
c. both methods result in the same balances in the partner capital accounts
d. both methods result in the same total value of partner capital account, but the individual
capital account vary
Suggested answer (a)
6. Blau and Rubi are partners who share profits and losses in the ratio of 6:4, respectively. On May 01,
2015, their respective capital accounts were as follows:
Blau P60,000
Rubi 50,000
On that date, Lind was admitted as a partner with one-third interest in capital, and profits for an
investment of P40,000. The new partnership began with total capital of P150,000.
Immediately after Lind’s admission, Blau’s capital should be
a. 50,000
b. 54,000
c. 56.667
d. 60,000
Suggested answer (b) 54,000
Contributed capital Agreed capital Increase (Decrease)
Old partners P110,000 P100,000 (P10,000)
New partner 40,000 50,000 (1/3) 10,000
Total P150,000 P150,000 -
Blau’s capital before admission of Lind P60,000
Less share in bonus to Lind (10000x60%) 6000
Blau’s capital after admission of Lind P 54,000

7. Partnership A has an existing capital of P70,000. Two partners currently own the partnership and split
profits 50/50. A new partner is to be admitted and will contribute net assets with a fair value of P90,000.
For no goodwill or bonus (depending on whichever method is used) to be recognized, what is the
interest in the partnership granted the new partner?
a. 33.33%
b. 50.00%
c. 56.25%
d. 75.00%

Suggested answer (c) 56.25%


Capital contributed by the new partner 90,000
Divide by total contributions (70,000+90,000) 160,000
New partner’s interest 56.25%
8. Dunn and Grey are partners with capital account balances of P60,000 and P90,000, respectively. They
agree to admit Zorn as a partner with one-third interest in capital and profits, for an investment of
P100,000, after revaluing the assets of Dunn and Grey. Goodwill to the original partners should be
a. 0
b. 33,333
c. 50,000
d. 66,667
Suggested answer (c) 50,000
Contributed capital Agreed capital Increase (Decrease)
Old partners P150,000 P200,000 P50,000
New partner 100,000 100,000 (1/3) -
Total P250,000 P300,000 P50,000
For nos. 9-10
Mitz, Marc, and Mart are partners sharing profits in the ratio of 5:3:2, respectively. As of December 31,
2014, their capital balances were P95,000 for Mitz, P80,000 for Marc, and P60,000 for Mart. On January
1, 2015, the partners admitted Vince as a new partner and according to their agreement, Vince will
contribute P80,000 in cash to the partnership and also pay P10,000 for 15% of Marc’s share. Vince will
be given a 20% share in profits, while the original partners’ share will be proportionately the same as
before. After the admission of Vince, the total capital will be P330,000 and Vince’s capital will be
P70,000.
9. The total amount of goodwill to the old partners, upon the admission of Vince would be:
a. 7,000
b. 15,000
c. 22,000
d. 37,000

Suggested answer (b) 15,000


Contributed capital Agreed capital Increase (Decrease)
Old partners P223,000 P260,000 P37,000
[235,000-(15%x80,000)
New partner
[80,000+(15%x80,000) 92,000 70,000 (22,000)
Total P315,000 P330,000 P15,000

10. The balance of Marc’s capital, after the admission of Vince would be:
a. 72,000
b. 74,600
c. 79,100
d. 81,100
Suggested answer (c) 79,100
Marc’s capital before Vince’s admission 80,000
Interest purchased by Vince (15%x80,000) (12,000)
Share in goodwill (30%x15,000) 4,500
Share in bonus (30%x22,000) 6,600
Marc’s capital after Vince’s admission 79,100

Chapter 4
Partnership Lump-Sum Liquidation
1. The ff. is the priority sequence in which liquidation proceeds will be distributed for a partnership:

A. Partnership drawings, partnership liabilities, partnership loans, partnership capital balances

B. Partnership liabilities, partnership loans, partnership capital balances

C. Partnership liabilities, partnership loans, partnership drawings, partnership capital balances

D. Partnership liabilities, partnership capital balances, partnership loans

Suggested answer (b)

2.

3. Which of the ff. statements is correct regarding a partner's capital deficiency?

A. Partners who absorb another's capital deficiency have a legal claim against the deficient

partner

B. The partner should contribute to reduce the debit balance to the extent possible
C. If contributions are not possible, the other partners with credit capital balances will be

allocated a portion of the debit balance

D. All of these statements are correct

Suggested answer (d)

4. Claims against partners' personal assets by creditors if the partnership can’t pay its debts refers to

A. Liquidation

B. Dissolution

C. Mutual agency

D. Unlimited liability

Suggested answer (d)

5. A liquidation differs from a dissolution in that in a liquidation

A. Assets may be revalued

B. The business will not continue

C. There may be an adjustment of partners' capital accounts

D. Gains and losses are distributed according to the partnership agreement

Suggested answer (b)

6. The following condensed balance sheet is presented for the partnership of Axel, Barr and Cain, who
share profits and losses at the rate of 4:3:3, respectively:
Cash P100,000
Other Assets 300,000
Total P400,000

Liabilities P150,000
Axel, Capital 40,000
Barr, capital 180,000
Cain, capital 30,000
Total P400,000
The partners agreed to dissolve the partnership after selling the other asset for P200,000. Upon
dissolution of the partnership, axel should have received
a. 0
b. 40,000
c. 60,000
d. 70,000

Suggested answer (a) Axel Barr Cain


Capital Balances before liquidation P40,000 P180,000 P30,000
Loss on realization 4:3:3 (200,000-300,000) 40,000 30,000 30,000
Capital Balances after Liquidation P 0 P150,000 P 0

7. Peter and John who share profits and losses equally, decided to liquidate their partnership when fair
net assets amounted to P260,000, and capital balances of P170,000 and P90,000, respectively.

If the noncash assets were sold for an amount equal to its book value, what is amount of cash
should Peter and John received?
Peter John
a. 130,000 130,000
b. 170,000 90,000
c. 180,000 80,000
d. 195,000 65,000

Suggested answer (b)


Given that noncash assets were sold for an amount equal to its book value, therefore,
no gain or loss was realized from the sale of noncash assets. Thus, the partners will receive an
amount equal to their respective capital balances before liquidation.

8. The following condensed balance sheet is presented to the partnership of Smith and Jones who share
profits and losses in the ratio of 60:40, respectively:
Other Assets 450,000
Smith, Jones 20,000
470,000

Accounts Payable 120,000


Smith, capital 195,000
Jones, capital 155,000
470,000
Suggested answer (a)
Smith Jones
Capital Balances 195,000 155,000
Smith, loan (Dr) (20,000) ______
Total Interest 175,000 155,000
Loss on realization (60:40)
(450,000-385,000) (39,000) (26,000)
Cash available for partners 136,000 129,000

For Items 9 & 10:


As of December 31, the books of AME Partnership showed capital balances of A =
40,000, M = 25,000 and E = 5,000. The partner’s profit and loss ratio was 3:2:1, respectively. The
partners decided to dissolve and liquidate. They sold all the non-cash assets for 37,000 cash.
After settlement of all liabilities amounting to 12,000, they still have 28,000 cash left for
distribution.

9. The loss on the realization of the non-cash assets was


a. 40,000
b. 42,000
c. 44,000
d. 45,000

Suggested answer (b)


Total capital before liquidation (40,000+25,000+5,000) 70,000
Less: Cash left for distribution 28,000
Loss on realization of the non-cash assets 42,000

10. Assuming that any partner’s capital debit balance is uncollectible, the share of A in the 28,000 cash
for distribution would be
a. 19,000
b. 18,000
c. 17,800
d. 40,000

Suggested answer (c)


A M E
Capital balances before Liquidation 40,000 25,000 5,000
Loss on Realization (21,000) (14,000) (7,000)
Balances 19000 11,000 (2,000)
Absorption of E (3:2) (1,200) (800) 2,000
Cash Payment to A & B 17,800 10,200
Chapter 5
Partnership Installment Liquidation
1. A partner’s loss absorption balance is calculated by
a. dividing the partner’s capital balance by his percentage interest in capital
b. multiplying distributable assets by the partner’s profit sharing percentage
c. dividing the partner’s total interests by his P/L sharing percentage
d. multiplying the partner’s total interests by his P/L sharing percentage
Suggested answer (a)
2.
3. In accounting for liquidation of a partnership, cash payments to partners after all outside creditor’s
claims have been satisfied. But before final cash distribution, should be according to
a. relative P/L sharing ratio
b. safe payments computations
c. the final balances in partners’ capital accounts
d. the relative share of gain or loss on liquidation
Suggested answer (b)
4. In a liquidation, the liabilities of the partnership should be paid
a. before any sales of assets
b. before the distribution of cash to partners
c. before the distribution of gains and losses on the disposal of assets
d. after a revaluation of assets
Suggested answer (b)

5. In a partnership liquidation, the assets of the partnerships shall be applied lastly to


a. those owing to outside creditors
b. those owing to the partners with respect to their share of the profits
c. those owing to the partners with respect to their capital contributions
d. those owing to inside creditors in the form of loans or advances for business expense by
the partners
Suggested answer (b)
6. After incurring losses resulting from very unprofitable operations, the Goh Kong Wei Partnership
decided to liquidate when the partners’ capital balances were:
Goh, capital (40%) 80,000
Kong, Capital (40%) 130,000
Wei, Capital (20%) 96,000
The noncash assets were sold in installment. Available cash were distributed to partners in every
sale of noncash assets. After the second sale of noncash assets, the partners received the same
amount of cash in the distribution. And from the 3 rd sale of noncash assets, cash available for
distribution amounts to 28,000 and unsold noncash assets has a book value of 12,500. Using
cash priority program, what amount did Wei received in the third installment of cash?
a. 1,600
b. 3,000
c. 5,600
d. 0

Suggested answer (c)


Since there are three partners in the firm, in the third priority of payments, the amount
to be received by Wei is 5,600 (28,000 x 20%), because at this point, their loss absorption
balances are equal.

8. Partners Almond, Barney and Colors have capital balances of 20,000, 50,000 and 90,000, respectively.
They split profits in the ratio of 2:4:4, respectively. Under a safe cash distribution plan, one of the
partners will get the following total amount in liquidation before any other partners get anything:
a. 0
b. 15,000
c. 40,000
d. 180,000

Suggested answer (c)


Almond Barney Colors
Total Interest 20,000 50,000 90,000
Divide by P&L 20% 40% 40%
Loss absorption balance100,000 125,000 125,000
Priority 1 – Colors (100,000)
Balances 100,000 125,000 125,000
Priority 2 – Colors & Barney (25,000) (25,000)
Balances (P&L) 100,000 100,000 100,000

The ABC Partnership has assets with book value of 240,000 and a market value of 195,000, outside
liabilities of 70,000, loans payable to Partner Able of 20,000, and capital balances for Partners Able,
Baker and Chapman of 70,000, 30,000 and 50,000, respectively. The partners share profits and losses
equally.

9. How would the first 100,000 of available assets be distributed?


a. 70,000 to outside liabilities, 20,000 to Able and the balance equally among partners
b. 70,000 to outside liabilities, and 30,000 to able
c. 70,000 to outside liabilities, 25,000 to Able and 5,000 to Chapman
d. 40,000 to Able, 20,000 to Chapman and the balance equally among partners
Suggested answer (b)
Able Baker Chapman
Total Interest 90,000 30,000 50,000
Divide by P&L (equally) 1/3 1/3 1/3
Loss absorption balance270,000 90,000 150,000
Priority 1 – Able 120,000
Balance 150,000 90,000 150,000
Priority 2 – Able & Chapman (60,000) (60,000)
Balance 90,000 90,000 90,000

Payments by Priority:
Priority 1 (120,000 x 1/3) 40,000
Priority 2 (60,000 x 1/3) 20,000 20,000

Cash Liability Able Baker Chapman


Total 100,000
Liability (70,000) 70,000
Balance30,000
Loan-A (20,000) 20,000
Balance10,000
Priority 1 (10,000) 10,000
Total 70,000 30,000

10. If all outside creditors and loans to partners had been paid. How would the balance of the assets be
distributed assuming Chapman had already received assets with a value of 30,000?
a. Each of the partners would receive 25,000
b. Each of the partners would receive 40,000
c. Able: 70,000, Baker: 15,000, Chapman: 20,000
d. Able: 55,000, Baker: 15,000, Chapman: 5,000

Suggested answer (d)


Able Baker Chapman
Total Interest (excluding loan) 70,000 30,000 50,000
Divide by P&L (equally) 1/3 1/3
1/3
Loss absorption balance 210,000 90,000 150,000
Priority 1 – Able (60,000)________________________
Balance 150,000 90,000 150,000
Priority 2 – Able & Chapman (60,000) (60,000)
Balance 90,000 90,000 90,000
Priority 3 – P&L

Payments by priority:
Priority 1 (60,000 x 1/3) 20,000
Priority 2 (60,000 x 1/3) 20,000 20,000

Cash Able Baker Chapman

MV of Assets 195,000
Liabilities (70,000)
Able, Loan (20,000)
Balance 105,000
Priority 1 (20,000) 20,000
Balance 85,000
Priority 2 (40,000) 20,000 20,000
Balance 45,000
Priority 3 (45,000) 15,000 15,000 15,000
Total
Less: Asset taken by Chapman 30,000
Balance 55,000 15,000 5,000

Chapter 6
Corporate Liquidation
1. Which of the ff. best illustrates the insolvency of a firm?
a. the filing of bankruptcy proceedings against the firm
b. a deficit in the firm’s retained earnings
c. the firm has more liabilities than assets
d. the firm has negative working capital
Suggested answer (c)
2. if the value of the pledged property is lesser than the obligation, what is the treatment of the liability?
a. partially secured
b. fully secured
c. collateralized
d. unsecured
Suggested answer (a)
3. The primary difference between a balance sheet and an accounting statement of affairs is that:
a. a balance sheet reflects book values, while a statement of affairs emphasizes realization
values
b. assets are arranged in a different sequence
c. liabilities are arranged in a different sequence
d. owner’s equity is not considered in the statement of affairs
Suggested answer (a)
4. An accounting statement of affairs of a corporation in financial difficulty indicates that unsecured
creditors would receive P0.40 on the peso. Which one of the ff. assets is most likely to realize the
smallest percentage of its book value?
a. accounts receivable
b. inventories
c. plant and equipment
d. goodwill
Suggested answer (d)
5. An arrangement for creditors to accept an amount less than the amount owed to them is referred to
as a
a. charge and discharge agreement
b. composition agreement
c. bankruptcy agreement
d. chandler agreement
Suggested answer (b)
6. Seco Corp. was forced into bankruptcy and is in the process of liquidating assets and paying claims.
Unsecured claims will be paid at the rate of P0.40 on the peso. Hale holds a P33,000 noninterest bearing
note receivable from Seco collateralized by an asset with a book value of P35,000 and a liquidation value
of P5,000.
The amount to be realized by Hale on this note is
a. 5,000
b. 12,000
c. 15,000
d. 17,000
Suggested answer (b) 15,000
Cash received in full as a secured creditor P5,000
Cash received as unsecured creditor 10,000
[(30,000-5,000)x0.40]
Total amount realized from the note P15,000

7. Kent Co. filed a voluntary bankruptcy petition on August 15, 2014, and the statement of affairs
reflected the ff. accounts:
Assets Book Value Estimated Current Value
Assets pledged with fully secured creditors P300,000 P370,000
Assets pledged with partially secured creditors 180,000 120,000
Free assets 420,000 320,000
P900,000 P810,000
Liabilities
Liabilities with priority P70,000
Fully secured creditors 260,000
Partially secured creditors 200,000
Unsecured creditors 540,000
P1,070,000
Assume that the assets are converted to cash at the estimated current values and the business is
liquidated. What amount of cash will be available to pay unsecured non-priority claims?
a. 240,000
b. 280,000
c. 320,000
d. 360,000
Suggested answer (d) 360,000
Estimated current value of assets pledged P370,000
with fully secured creditors
Add estimated current value of free assets 320,000
Less liabilities:
Fully secured creditors P260,000
With priority 70,000 (330,000)
Total cash available for unsecured claims P360,000

For items 8-10


In 2014, Camel Corp. was forced into bankruptcy and begun to liquidate. The ff. selected account
balances were taken from its statement of affairs:
Book Value Estimated Current Value
Assets pledged with partially P80,000 P50,000
secured creditors
Total free assets P220,000 P160,000

Book Value Estimated Current Value


Preferred claims P160,000 P0
Partially secured liabilities 75,000 25,000
Unsecured liabilities 155,000 155,000

8. What is the total amount available for payment of claims of unsecured creditors?
a. 0
b. 144,000
c. 160,000
d. 210,000
Suggested answer (b) 144,000
Estimated current value of free assets P160,000
Less preferred claims (16,000)
Total amount available to unsecured creditors P144,000

9. What is the estimated amount of liquidating dividend per peso claim (rounded to the nearest
centavo)?
a. 0.80
b. 0.88
c. 1.03
d. 1.17

Suggested answer (a) 0.80


Estimated current value of free assets P160,000
Less preferred claims (16,000)
Total amount available to unsecured creditors P144,000
Divide by total unsecured liabilities 180,000
(155,000+25,000)
Estimated liquidating dividend per peso claim P0.80
10. What is the amount of deficiency to creditors?
a. 180,000
b. 160,000
c. 144,000
d. 36,000
Suggested answer (d) 36,000
Total amount available to unsecured creditors P144,000
Less total unsecured liabilities 180,000
Deficiency to creditors P36,000
Gabiosa, Mar Sean Jan
Tribunalo, Ryan James
Chapter Questions
Theories 1-5; Problems-6-10

Chapter 1 Partnership Formation

1. A partner who is entitled to a share of the profits from a partnership is known as:
a) A salaried partner.
b) A managing partner.
c) An equity partner.
d) A limited liability partner.

Answer: An equity partner.


A partner on a fixed salary is known as a salaried partner. 

2. The maximum number of persons who are legally allowed to operate in a partnership is:
a) 2
b) 20
c) There is no legal limit
d) 100

Answer: There is no legal limit

3. Sparkle Ltd is a private limited company limited by shares. It has one director. How many
shareholders does the law require it to maintain?
a) One provided it is a different person from the director.
b) Five.
c) Two.
d) One which can be the same person as the director.

Answer: One which can be the same person as the director.


The law allows private limited companies to exist with one shareholder who is the same person
as the director.

4. Which one of the following statements about limited liability partnerships (LLPs) is incorrect?
a) An LLP has a legal personality separate from that of its members.
b) The liability of each partner in an LLP is limited.
c) Members of an LLP are taxed as partners.
d) A limited company can convert to an LLP.
Answer: A limited company can convert to an LLP.
A general partnership can convert to an LLP but a limited company cannot.

5. An organisation running a business has the following attributes: the assets belong to the
organisation, it can create a floating charge over its assets, change in membership does not
alter its existence, and members cannot transfer their interests to others. What type of
organisation is it?
a) A private limited company
b) A limited liability partnership
c) A general partnerships
d) A private limited company

Answer: A limited liability partnership.


In the question the attributes of the organisation are the same for LLPs as companies except
members of a company (private and public) can transfer their interests to others.

6. Roberts and Smith drafted a partnership agreement that lists the following assets
contributed at the partnership’s formation:
Contributed by
Roberts Smith
Cash $20,000 $30,000
Inventory -- 15,000
Building -- 40,000
Furniture & equipment 15,000 --

The building is subject to a mortgage of $10,000, which the partnership has assumed. The
partnership agreement also specifies that profits and losses are to be distributed evenly. What
amounts should be recorded as capital for Roberts and Smith at the formation of the
partnership?
Roberts Smith
a. $35,000 $85,000
b. $35,000 $75,000
c. $55,000 $55,000
d. $60,000 $60,000

Answer: (b) The requirement is to determine the amounts to be recorded as capital for Roberts
and Smith at the formation of the partnership. Unless otherwise agreed upon by the partners,
individual capital accounts should be credited for the fair market value (on the date of
contribution) of the net assets contributed by that partner. It is necessary to assume that the
amounts listed are fair market values. The amount of net assets that Roberts contributed is
$35,000 ($20,000 + $15,000). The fair market value of the net assets Smith contributed is
$75,000 ($30,000 + $15,000 + $40,000 – $10,000). The partners’ profit and loss sharing ratio
does not affect the initial recording of the capital accounts.
7. On April 30, year 1, Algee, Belger, and Ceda formed a partnership by combining their
separate business proprietorships. Algee contributed cash of $50,000. Belger contributed
property with a $36,000 carrying amount, a $40,000 original cost, and $80,000 fair value. The
partnership accepted responsibility for the $35,000 mortgage attached to the property. Ceda
contributed equipment with a $30,000 carrying amount, a $75,000 original cost, and $55,000
fair value. The partnership agreement specifies that profits and losses are to be shared equally
but is silent regarding capital contributions. Which partner has the largest April 30, year 1
capital account balance?
a. Algee.
b. Belger.
c. Ceda.
d. All capital account balances are equal.

Answer: (c) The requirement is to determine which partner has the largest capital account
balance. Use the solutions approach to solve the problem.
Algee Belger Ceda
Partner contribution 50,000 80,000 55,000
Less: Liabilities assumed
by the partnership 0 (35,000) 0
Ending capital balance $50,000 $45,000 $55,000

Each partner values his contribution to the partnership at its fair market value. The fair market
value becomes the partner’s balance in his capital account and is basis to the partnership under
generally accepted accounting principles. Any liabilities assumed by the partnership, reduces
the partners’ capital balance by the amount assumed.

8. Abel and Carr formed a partnership and agreed to divide initial capital equally, even though
Abel contributed $100,000 and Carr contributed $84,000 in identifiable assets. Under the bonus
approach to adjust the capital accounts, Carr’s unidentifiable asset should be debited for
a. $46,000
b. $16,000
c. $ 8,000
d. $0

Answer: (d) Under the bonus method, unidentifiable assets (i.e., goodwill) are not recognized.
The total resulting capital is the FV of the tangible investments of the partners. Thus, there
would be no unidentifiable assets recognized by the creation of this new partnership.

9. Ellis and Nossiter are partners sharing profits in a 30:70 ratio. The following data summarizes
2004 activity:

Partnership net income, 2004 $68,000


Ellis capital, 1/1/2004 90,000
Ellis additional investment in 2004 10,000
Ellis drawings in 2004 12,000
Nossiter capital, 1/1/2004 80,000
Nossiter drawings in 2004 20,000

What amount of net income is allocated to Nossiter’s capital account for 2004?
a. $26,600
b. $27,600
c. $34,000
d. $47,600

Answer: (d) (68,000×.7)

Chapter 2: Partnership Operations

1. The Flat and Iron partnership agreement provides for Flat to receive a 20% bonus on profits before
the bonus. Remaining profits and losses are divided between Flat and Iron in the ratio of 2:3,
respectively. Which partner has a greater advantage when the partnership has a profit or when it has
a loss?
Profit Loss
a. Flat Iron
b. Flat Flat
c. Iron Flat
d. Iron Iron

Answer: (b)
In both the case of a profit or a loss, Flat will have a greater advantage. When there is a profit,
Flat will obtain a 20% bonus on profits before the bonus, and also take 40% of the profit after the
bonus. Iron on the other hand, will only receive 60% of the profit after the bonus. In the case of a
loss, it can easily be seen that since Flat has a smaller percentage share in the loss that he has a
greater advantage.

2. Partners C and K share profits and losses equally after each has been credited in all circumstances
with annual salary allowances of $15,000 and $12,000, respectively. Under this arrangement, C will
benefit by $3,000 more than K in which of the following circumstances?

a. Only if the partnership has earnings of $27,000 or more for the year.
b. Only if the partnership does not incur a loss for the year.
c. In all earnings or loss situations.
d. Only if the partnership has earnings of at least $3,000 for the year.

Answer: (c)
Partners C and K are always credited or charged with the same amount of profit and loss, and C’s
salary is always $3,00 greater than K’s. Therefore, C will benefit $3,000 more than K in all earnings
or loss situations.

3. The Low and Rhu partnership agreement provides special compensation to Low for managing the
business. Low receives a bonus of 15% of partnership net income before salary and bonus, and also
receives a salary of $45,000. Any remaining profit or loss is to be allocated equally. During 20xx, the
partnership had a net income of $50,000 before the bonus and salary allowance. As a result of these
distributions, Rhu’s equity in the partnership would

a. Increase
b. Not change
c. Decrease the same as Low’s
d. Decrease

Answer: (d)
Low’s 15% bonus and salary exceed the partnership net income, resulting in a $2,500 “loss” to be
distributed equally to Low and Rhu.

4. In a partnership agreement, allocation of profits and losses includes salary allocation, a bonus and
interest on capital, with any remainder to be allocated by preset ratio. If the partnership incurred a
loss in the period, which of these procedures would be applied?

a. Salary allocation would not be used.


b. Loss would be allocated to partners equally.
c. Bonus criteria would not be used.
d. Interest allocation is based on beginning capital.

Answer: (c)
5. Which of the following capital balance for interest computation is less altered upon calculation of
profits and losses to partners?

a. Weighted average capital balance


b. Ending capital balance
c. Beginning capital balance
d. Invested capital

Answer: (a)

6. Red and White formed a partnership in year 1. The partnership agreement provides for annual salary
allowances of $55,000 for Red and $45,000 for White. The partners share profits equally and losses
in a 60/40 ratio. The partnership had earnings of $80,000 for year 1 before any allowance to partners.
What amount of these earnings should be credited to each partner’s capital account?

Red White
a. $40,000 $40,000
b. $43,000 $37,000
c. $44,000 $36,000
d. $45,000 $35,000

Answer: (b)
Red White Total
Profit before allowance $ 80,000
Salary allowances $55,000 $45,000 (100,000)
Loss after allowances (60/40) (12,000) (8,000) (20,000)
Earnings credited to partners $43,000 $37,000 $80,000

7. Fox, Greg, and Howe are partners with average capital balances during year 1 of $120,000, $60,000,
and $40,000, respectively. Partners receive 10% interest on their average capital balances. After
deducting salaries of $30,000 to Fox and $20,000 to Howe, the residual profit or loss is divided
equally. In year 1 the partnership sustained a $33,000 loss before interest and salaries to partners. By
what amount should Fox’s capital account change?
a. $ 7,000 increase.
b. $11,000 decrease.
c. $35,000 decrease.
d. $42,000 increase.

Answer: (a)
Fox Greg Howe
Interest allowance(10% of avg. cap.
balances) $12,000 $6,000 $4,000
Salaries 30,000 20,000
Residual* ($105,000 ÷ 3) (35,000) (35,000) (35,000)
Increase (decrease)in cap. account $ 7,000 $(29,000) $(11,000)
8. The partnership agreement of Axel, Berg & Cobb provides for the year-end allocation of net income
in the following order:
 First, Axel is to receive 10% of net income up to $100,000 and 20% over $100,000.
 Second, Berg and Cobb each are to receive 5% of the remaining income over $150,000.
 The balance of income is to be allocated equally among the three partners.

The partnership’s year 1 net income was $250,000 before any allocations to partners. What amount
should be allocated to Axel?

a. $101,000
b. $103,000
c. $108,000
d. $110,000

Answer: (c)
Step Axel Berg Cobb
1. Axel: 10% of first $100,000,
20% over $100,000
$10,000
30,000

2. Berg & Cobb: 5% of remaining


income over $150,000 [($250,000
–$10,000 – $30,000 –$150,000)
× .05]
$3,000 $3,000

3. Remaining allocated equally:


[($250,000 – $10,000 – $30,000
– $3,000 – $3,000) × 1/3]
68,000 68,000 68,000

Totals $108,000 $71,000 $71,000

9. The partnership agreement of Reid and Simm provides thatinterest at 10% per year is to be credited to each partner
on the basis of weighted-average capital balances. A summary of Simm’s capital account for the year ended
December 31, year 1, is as follows:

Balance, January 1 $140,000


Additional investment, July 1 40,000
Withdrawal, August 1 15,000
Balance, December 31 165,000

What amount of interest should be credited to Simm’s capital account for year 1?
a. $15,250
b. $15,375
c. $16,500
d. $17,250

Answer: (b)

Simm’s weighted-average capital balance for year 1 is as follows:

Capital bal. × # of months/12 = Weighted-avg.


140,000 × 6/12 = $ 70,000
180,000 × 1/12 = 15,000
165,000 × 5/12 = 68,750
$153,750

The problem states that interest of 10% per year is to be credited to each partner’s capital account, and 10% of
Simm’s weighted average capital balance of $153,750 is $15,375.
Chapter 3: Partnership Dissolution

1. In the Adel-Brick partnership, Adel and Brick had a capital ratio of 3:1 and a profit and loss ratio of
2:1, respectively. The bonus method was used to record Colter’s admittance as a new partner. What
ratio would be used to allocate, to Adel and Brick, the excess of Colter’s contribution over the
amount credited to Colter’s capital account?

a. Adel and Brick’s new relative capital ratio.


b. Adel and Brick’s new relative profit and loss ratio.
c. Adel and Brick’s old capital ratio.
d. Adel and Brick’s old profit and loss ratio.

Answer: (d)
The bonus method implies that the old partners either received a bonus from the new
partner, or they paid a bonus to the new partner. In this case, Colter, the new partner, contributed
an amount in excess of the amount credited to Colter’s capital account. Accordingly, the excess
should be treated as a bonus to Adel and Brick. This bonus should be treated as an adjustment to
the old partners’ capital accounts and should be allocated by using Adel and Brick’s old profit
and loss ratio.

2. When Mill retired from the partnership of Mill, Yale, and Lear, the final settlement of Mill’s interest
exceeded Mill’s capital balance. Under the bonus method, the excess
a. Was recorded as goodwill.
b. Was recorded as an expense.
c. Reduced the capital balances of Yale and Lear.
d. Had no effect on the capital balances of Yale and Lear.

Answer: (c)
Under the bonus method, adjustments are made only among partner’s capital accounts
(no goodwill is recorded on the partnership books). Since Mill’s partnership interest exceeded the
amount of Mill’s capital balance, the excess interest would reduce the capital balances of Yale
and Lear. Only under the goodwill method can the excess interest be recorded as goodwill. Under
no circumstances should the excess partnership interest be recorded as an expense.

3. Pat, Helma and Diane are partners with capital balances of $50,000 $30,000, and $20,000,
respectively. The partners share profits and losses equally. For an investment of $50,000 cash,
MaryAnn is to be admitted as a partner with a one-fourth interest in capital and profits. Based on this
information, the amount of MaryAnn’s investment can best be justified by which of the following?

a. MaryAnn will receive a bonus from the other partners upon her admission to the partnership.
b. Assets of the partnership were overhauled immediately prior to MaryAnn’s investment.
c. The book value of the partnership’s net assets was less than their fair value immediately prior
to MaryAnn’s investment.
d. MaryAnn is apparently bringing goodwill into the partnership and her capital account will be
credited for the appropriate amount.

Answer: (c)

Total capital after admittance of new partner $150,000


$150,000 X 1/4 = $37,500
Investment for 1/4 interest = $50,000

MaryAnn paid $50,000 for assets with a net book value of $37,500. Therefore, the book
value of the partnership’s net assets must have been less than theirs fair value at the time of her
admittance.

4. When Mill retired from the partnership of Mill, Yale and Lear, the final settlement of Mill’s interest
exceed Mill’s capital balance. Under the bonus method, the excess

a. Was recorded as goodwill.


b. Was recorded as an expense.
c. Reduced the capital balances of Yale and Lear.
d. Had no effect on the capital balances of Yale and Lear.

Answer: (c)
Under the bonus method, distributions to retiring partners in excess of their capital
balance are charged against the other partner’s capital accounts in proportion to their profit and
loss ratio.

5. Kern and Pate are partners with capital balances of $60,000 and $20,000, respectively. Profits and
losses are divided in the ratio of 60:40. Kern and Pate decided to form a new partnership with Grant,
who invested land valued at $15,000 for a 20% capital interest in the new partnership. Grant’s cost of
the land was $12,000. The partnership elected to use the bonus method to record the admission of
Grant into the partnership. Grant’s capital account should be credited for
a. $12,000
b. $15,000
c. $16,000
d. $19,000

Answer: (d)
The requirement is to determine the balance in the new partner’s capital account after
admission using the bonus method. In this case, Grant is investing land with a FV of $15,000 for
a 1/5 interest in the new total capital of $95,000. Using the bonus method, the new capital
$95,000 equals the total of the old capital plus Grant’s investment ($60,000 + $20,000 +
$15,000). Thus, a bonus of $4,000 is being credited to Grant’s capital account because his interest
(1/5 of $95,000, or $19,000) exceeds his investment ($15,000). The bonus to the new partner is
charged to the old partners’ capital accounts in their profit and loss ratios.

6. The following condensed balance sheet is presented for the partnership of Alfa and Beda, who
share profits and losses in the ratio of 60:40, respectively:

Cash $ 45,000
Other assets 625,000
Accounts payable 120,000
Beda, loan 30,000
Alfa, capital 348,000
Beda, capital 232,000

The assets and liabilities are fairly valued on the balance sheet. Alfa and Beda decide to
admit Capp as a new partner with 20% interest. No goodwill or bonus is to be recorded. What
amount should Capp contribute in cash or other assets?

a. $110,000
b. $116,000
c. $140,000
d. $145,000

Answer: (d)
If no goodwill or bonus is to be recorded, the formula to determine the necessary contribution is to
divide the old partner’s capital ($580,000) by their interest after the new partner’s admission. The result is the
total capital after admission ($580,000 ÷ 80% = $725,000). To compute the new partner’s contribution, the old
partners’ capital can be subtracted from total capital ($725,000 – $580,000 = $145,000), or total capital can be
multiplied by 20%.

(20% × $725,000 = $145,000)

7. Jay & Kay partnership’s balance sheet at December 31, year 1, reported the following:

Total assets $100,000


Total liabilities 20,000
Jay, capital 40,000
Kay, capital 40,000

On January 2, year 2, Jay and Kay dissolved their partnership and transferred all assets and liabilities to a newly
formed corporation. At the date of incorporation, the fair value of the net assets was $12,000 more than the
carrying amount on the partnership’s books, of which $7,000 was assigned to tangible assets and $5,000 was
assigned to goodwill. Jay and Kay were each issued 5,000 shares of the corporation’s $1 par value common
stock. Immediately following incorporation, additional paid-in capital in excess of par should be credited for

a. $68,000
b. $70,000
c. $77,000
d. $82,000

Answer: (d)
When a partnership incorporates, assets and liabilities must be revalued to their fair market values on
the date of incorporation. In this case, the net assets have a fair market value of $92,000 ($80,000 + $12,000) and
the amount to be credited to Additional Paid-in Capital is $82,000 ($92,000 – $10,000 par value).

8. On June 30, year 1, the balance sheet for the partnership of Coll, Maduro, and Prieto, together with
their respective profit and loss ratios, were as follows:

Assets, at cost $180,000


Coll, loan $ 9,000
Coll, capital (20%) 42,000
Maduro, capital (20%) 39,000
Prieto, capital (60%) 90,000
Total $180,000

Coll has decided to retire from the partnership. By mutual agreement, the assets are to be
adjusted to their fair value of $216,000 at June 30, year 1. It was agreed that the partnership
would pay Coll $61,200 cash for Coll’s partnership interest, including Coll’s loan which is to be
repaid in full. No goodwill is to be recorded. After Coll’s retirement, what is the balance of
Maduro’s capital account?

a. $36,450
b. $39,000
c. $45,450
d. $46,200

Answer: (c)
The requirement is to determine the balance in Maduro’s capital account after Coll’s retirement. When
a partner withdraws from a partnership a determination of the fair value of the entity must be made. Since it is
stated in the problem that the withdrawing partner is selling his interest to the partnership and that no goodwill is
to be recorded, the bonus method must be employed after restatement of assets to FV. The capital accounts after
restatement to FV would be

Coll
[$42,000 + 20%($216,000 – $180,000)] = $ 49,200

Maduro
[$39,000 + 20%($216,000 – $180,000)] = $ 46,200
Prieto
[$90,000 + 60%($216,000 – $180,000)] = $111,600

The bonus paid to Coll is the difference between the cash paid to him for his partnership interest and
the balance of that interest plus his loan balance.

Bonus = [$61,200 – ($49,200 + $9,000)] = $3,000

Maduro’s capital account would be reduced by his proportionate share of the bonus, based on the profit
and loss ratio of the remaining partners [20%/(20% + 60%) = 25%].

Maduro’s capital
[$46,200 – 25% ($3,000)] = $45,450

Chapter 4: Partnership Lump-sum Liquidation


1. Which of the following procedures is acceptable when accounting for a deficit balance in a partner’s
capital account during partnership liquidation?

a. A partner with a negative capital balance must contribute personal assets to the partnership
that are sufficient to bring the capital account to zero.
b. If a partner with a negative capital balance is personally insolvent, the negative capital
balance may be absorbed by those partners having a positive capital balance according to the
residual profit and loss sharing ratios that apply to all the partners.
c. If a partner with a negative capital balance is personally insolvent, the negative capital
balance may be absorbed by those partners having a positive capital balance according to the
residual profit and loss sharing ratios that apply to those partners having positive balances.
d. All the above procedures are acceptable.

Answer: (c)

2. Under the rule of offset, what is the proper disposition of a partnership loan that was made from a
partner who has a debit balance?

a. The loan is first paid to the debtor partner before cash payments are made to partners.
b. The loan is written off as a partnership loss if the partner does not have the cash to cover the
debit balance.
c. The loan is charged off to the capital accounts of all the partners in their profit and loss sharing
ratios.
d. The loan is charged off to the capital account of the debtor partner.

Answer: (b)

3. The rank order is for claims against a bankrupt partner of

I. Those owing to partners by way of contribution

II. Those owing to separate creditors

III. Those owing to partnership creditors


a. II first; I second and III third.
b. III first; II second and I third.
c. I first; III second and II third.
d. II first; III second and I third.

Answer: (d)

4. Which statement is correct in describing the rank order of payments as specified by the Uniform
Partnership Act?

a. Payments to partners with loans to the partnership are ranked equally with payments to
other creditors.
b. Payments to partners with loans to the partnership are ranked ahead of payments to
partners without loans to the partnership.
c. Payments to other creditors are ranked ahead of payments to partners with loans to the
partnership.
d. After payments are made to other creditors and partners with loans to the partnership,
payment can be made to partners with capital interests.

Answer: (c)

5. A partnership in liquidation has converted all assets into cash and paid all liabilities. According to the
Uniform Partnership Act, the order of payment

a. will have amounts due to partners with respect to their capital accounts take precedence over
amounts owed by partners other than for capital and profits.
b. will be according to the partners’ residual profit and loss sharing ratios.
c. will have amounts owed by partners other than for capital and profits take precedence over
amounts due to partners with respect to their capital accounts.
d. Will be by any manner that is both reasonable and rational for the partnership.

Answer: (c)

6. The following condensed balance sheet is presented for the partnership of Smith and Jones, who
share profits and losses in the ratio of 60:40, respectively:
Other assets $450,000
Smith, loan 20,000
$470,000
Accounts payable $120,000
Smith, capital 195,000
Jones, capital 155,000
$470,000

The partners have decided to liquidate the partnership. If the other assets are sold for $385,000,
what amount of the available cash should be distributed to Smith?

a. $136,000
b. $156,000
c. $159,000
d. $195,000

Answer: (a)
Smith Jones Total
Capital (credit) $195,000 $155,000 $350,000
Loan (debit) (20,000) (20,000)
Net balances $175,000 $155,000 $330,000
Loss on sale of other assets
(450 – 385) 39,000 26,000 (65,000)
Cash available for partners $136,000 $129,000 $265,000

7. The following condensed balance sheet is presented for the partnership of Alfa and Beda, who
share profits and losses in the ratio of 60:40, respectively:
Cash $ 45,000
Other assets 625,000
Accounts payable 120,000
Beda, loan 30,000
Alfa, capital 348,000
Beda, capital 232,000

Alfa and Beda decide to liquidate the partnership. If the other assets are sold for $500,000, what
amount of the available cash should be distributed to Alfa?

a. $255,000
b. $273,000
c. $327,000
d. $348,000
Answer: (b)
To determine the amount of cash distributed during liquidation, the solutions approach is
to prepare an abbreviated statement of partnership liquidation. In a partnership liquidation, cash is
distributed based on the capital balances of the partners after adjusting them for any income (loss)
to the date of liquidation and any loans or advances between the partners and the partnership. The
abbreviated statement follows:

Alfa Beda Total


Beg. capital
balance $348,000 $232,000 $580,000
Adj. for loans (30,000) (30,000)
Adj. for loss on sale
of assets* (60-40) (75,000) (50,000) (125,000)
Adj. capital balance
* ($500,000 – $625,000) $273,000 $152,000 $425,000

Note that the total cash available also equals $425,000.

Beginning cash $ 45,000


Proceeds from sale 500,000
Payment of AP (120,000)
$425,000

8. Jade, Kahl, and Lane are in the process of liquidating their partnership. Lane has agreed to accept
the inventory, which has a fair value of $60,000, as part of her settlement. A balance sheet and the
residual profit and loss sharing percentages are as follows:

Cash $198,000 Accounts payable $149,000


Inventory 80,000 Jade, Capital 79,000
Plant Assets 230,000 Kahl, Capital 140,000
Total Assets $508,000 Total Liabilities and Equity $508,000

If the partners then distribute the available cash, Lane will receive

a. $23,000
b. $29,000
c. $30,000
d. $34,000
Answer: (a)

Jade Kahl Lane

Equities $ 79,000 $140,000 $140,000

Distribute inventory to Lane and: (60,000)


recognize $20,000 loss (8,000) (8,000) (4,000)

Possible losses on plant (92,000) (92,000) (46,000)

Subtotal $(21,000) $40,000 $30,000

Eliminate Jade’s debit balance to Kahl & Lane 21,000 (14,000) (7,000)

Balance $0 $26,000 $23,000

Use the following information for questions 9 and 10.

On June 30, 2015, the Warle, Xin, and Yates partnership had the following fiscal year-end
balance sheet:

Cash $ 4,000 Accounts payable $ 7,000

Accounts receivable 6,000 Loan from Xin 5,000

Inventory 14,000 Warle, capital(20%) 14,000

Plant assets-net 12,000 Xin, capital(30%) 10,000

Loan to Warle 6,000 Yates, capital(50%) 6,000

Total assets $ 42,000 Total liab./equity $ 42,000

The percentages shown are the residual profit and loss sharing ratios. The partners dissolved the
partnership on July 1, 2015 and began the liquidation process. During July the following events occurred:
 Receivables of $3,000 were collected.
 The inventory was sold for $4,000.
 All available cash was distributed on
 July 31, except for $2,000 that was set aside for contingent expenses.

9. The book value of the partnership equity (i.e., total equity of the partners) on June 30, 2015 is
a. $60,000.

b. $29,000.
c. $30,000.

d. $42,000.

Answer: (b)

$14,000 (Warle capital) + $10,000 (Xin capital) +$6,000 (Yates capital) + $5,000 (Loan from Xin) -
$6,000 (Loan to Warle)

10. The cash available for distribution to the partners on July 31, 2006 is

a. $ 2,000.

b. $ 4,000.

c. $ 7,000.

d. $11,000.

Answer: (a)

$4,000 (beginning balance) + $3,000 (cash collected) + $4,000 (for inventory sold) - $7,000 (of accounts
payable) - $2,000 (for expenses)

Chapter 5 Installment Liquidation


1. Under the rule of offset, what is the proper disposition of a partnership loan that was made
from a partner who has a debit balance?
a. The loan is first paid to the debtor partner before cash payments are made to
partners.
b. The loan is written off as a partnership loss if the partner does not have the cash to
cover the debit balance.
c. The loan is charged off to the capital accounts of all the partners in their profit and
loss sharing ratios.
d. The loan is charged off to the capital account of the debtor partner.

Answer: (b) The loan is written off as a partnership loss if the partner does not have the cash to
cover the debit balance.

2. In partnership liquidations, what are safe payments?

a. The amounts of distributions that can be made to the partners, after all creditors have
been paid in full.
b. The amounts of distributions that can be made to the partners with assurance that
such amounts will not have to be returned to the partnership.
c. The amounts of distributions that can be made to the partners, after all non-cash
assets have been adjusted to fair market value.
d. All the above are examples of the safe payments concept.

Answer: (b) The amounts of distributions that can be made to the partners with assurance that
such amounts will not have to be returned to the partnership.

3. If all partners are included in the first installment of an installment liquidation, then in future
installments
a. cash will be distributed according to the residual profit and loss sharing ratio.
b. cash should not be distributed until all non-cash assets are converted into cash.
c. a safe payments schedule must be prepared before each cash distribution to avoid
excessive payments to partners.
d. a cash distribution plan must be prepared so that partners will know when they will
be included in cash distributions.

Answer: a cash will be distributed according to the residual profit and loss sharing ratio.

5. Which partner is considered the most vulnerable as a result of a computation of vulnerability


rankings?
a. The partner with the lowest vulnerability ranking, who also has the lowest loss
absorption potential.
b. The partner with the lowest vulnerability ranking, who also has the highest loss
absorption potential.
c. The partner with the highest vulnerability ratio, who also has the lowest loss
absorption potential.
d. The partner with the highest vulnerability ranking, who also has the highest loss
absorption potential.

Answer: (a) The partner with the lowest vulnerability ranking, who also has the lowest loss
absorption potential.

6. After realization of a portion of the noncash assets of Saul, Tapp & Uris LLP, which is being
liquidated, the capital account balances were Saul, $35,000; Tapp, $40,000; and Uris, $43,000.
Cash of $42,000 and other assets with a carrying amount of $78,000 were on hand. Creditors’
claims totaled $2,000. The partners shared net income and losses equally. The cash that may be
paid to Uris at this time is:

A) $43,000
B) $17,000
C) $14,000
D) $13,333
E) Some other amount

Answer: (b) ($43,000 - $26,000 = $17,000)

8. Which partner is most vulnerable to losses?


a. Luis
b. Mac
c. Nel
d. Oma

Answer: (c)
Loss absorption potential:

Profit Loss
Partners’ and Loss Absorption Vulnerability
Equity Ratio Potential Ranking
Luis $ 20,000  40% $ 50,000 4
Mac 9,000  25% $ 36,000 2
Nel 7,000  25% $ 28,000 1
Oma 4,000  10% $ 40,000 3

9. Which among the partners has the first priority of payment after all creditors is paid.
a. Luis
b. Nel
c. Oma
d. Mac

Answer: (a)

Cash distribution plan:


First $44,000 pays the priority creditors;
Next $4,000 goes to Luis;
Next $2,000 goes $1,600 to Luis, and $400 to Oma;
Next $6,000 goes $3,200 to Luis, $2,000 to Mac, and $800 to Oma;
Remainder goes 40% to Luis, 25% to Mac, 25% to Nel, and 10% to Oma.

10. A cash distribution plan for the Upton, Valenta, and Walker partnership was as follows:

Priority
Creditors Upton Valenta Walker
First $100,000 100%
Next $180,000 44% 10% 46%
Next $270,000 2/9 1/9 2/3
Remainder 11% 44% 45%

If $700,000 of cash was distributed by the partnership, how much was received respectively by
the priority creditors, Upton, Valenta, and Walker?
a. 100,000; 155,700; 115,000; 333,000
b. 100,000; 157,500; 114,000; 300,300
c. 100,000; 155,700; 114,000; 330,000
d. 100,000; 155,700; 114,000; 330,300

Answer: (d)

Priority
Creditors Upton Valenta Walker
First $100,000 $ 100,000
Next $180,000 $79,200 $18,000 $82,800
Next $270,000 60,000 30,000 180,000
Last $150,000 16,500 66,000 67,500
Total $700,000 $ 100,000 $155,700 $114,000 $330,300
Chapter 6 Corporate Liquidation
1. Bankruptcy reorganizations are used by management to:
a. forestall the inevitable liquidation in all cases.
b. provide time to turn the business around.
c. allow the courts time to set up an administrative structure.
d. All of the above.
e. None of the above.

Answer: b

2. The difference between liquidation and reorganization is:


a. reorganization terminates all operations of the firm and liquidation only terminates
non-profitable operations.
b. liquidation terminates only profitable operations and reorganization terminates only
non-profitable operations.
c. liquidation terminates all operations and reorganization maintains the option of the
firm going concern.
d. liquidation only deals with current assets and reorganization only consolidates debt.
e. None of the above.

Answer: c

3. What is the absolute priority rule of the following claims once a corporation is determined to
be bankrupt?
a. administrative expenses, wages claims, government tax claims, debtholder and then
equityholder claims
b. administrative expenses, wages claims, government tax claims, equityholder and then
debtholder claims
c. wage claims, administrative expenses, debtholder claims, government tax claims and
equityholder claims
d. wage claims, administrative expenses, debtholder claims, equityholder claims and
government tax claims
e. None of the above.

Answer: a

4. Equityholders may prefer a formal bankruptcy filing because:


a. the firm can issue debtor in possession debt.
b. they can delay pre-bankruptcy interest payments.
c. the lack of information about the length and magnitude of the cash flow problem
favors
equityholders.
d. All of the above.
e. None of the above.

Answer: d

5. The net payoff to creditors in formal bankruptcy may be low in present value terms because:
a. the financial structure may be complicated with several groups and types of creditors.
b. indirect costs of bankruptcy may have been costly in lost revenues and poor
maintenance.
c. administrative costs are high and increase with the complexity and length of time in
the formal bankruptcy process.
d. All of the above.
e. None of the above.

Answer: d

6. Brook Corporation has filed for bankruptcy. Of the following debts Brook owes, indicate their
priorities from the highest to the lowest.
I. Federal taxes unpaid for the previous year.
II. Wages of $3,000 owed to employees.
III. Balance of $5,000 owed to a creditor that had a security
interest. This creditor got paid fully by selling off the collateral except for this $5,000
deficiency.
a. I, II, III.
b. I, III, II.
c. II, I, III.
d. III, I, II.

Answer: (c) Of those listed, wages of the bankrupt’s employees receive the highest priority for
up to a specified formula. Federal taxes have a low priority but are ahead of general creditors.
Any deficiency for secured creditors after the collateral is sold is paid along with the general
creditors.

7. On August 1, Hall filed a voluntary petition under Chapter 7 of the Federal Bankruptcy Code.
Hall’s assets are sufficient to pay general creditors 40% of their claims. The following
transactions occurred before the filing:
• On May 15, Hall gave a mortgage on Hall’s home to National Bank to secure payment
of a loan National had given Hall two years earlier. When the loan was made, Hall’s twin
was a National employee.
• On June 1, Hall purchased a boat from Olsen for $10,000 cash.
• On July 1, Hall paid off an outstanding credit card balance of $500. The original debt
had been $2,500.

The National mortgage was


a. Preferential, because National would be considered an
insider.
b. Preferential, because the mortgage was given to secure
an antecedent debt.
c. Not preferential, because Hall is presumed insolvent
when the mortgage was given.
d. Not preferential, because the mortgage was a security
interest.

Answer (b) Under Chapter 7 of the Federal Bankruptcy Code, the trustee may set aside
preferential transfers made to a creditor within ninety days prior to the filing of the petition for
bankruptcy. Preferential transfers are those made for antecedent debts that allow the creditor
to receive more than s/he would have under the bankruptcy law. All of these conditions were
met for the National mortgage. Answer (a) is incorrect because National would not be
considered an insider. Even though Hall’s twin was a National employee, he was not an officer,
director, or controlling stockholder of National. Furthermore, the preferential transfer was not
made to him personally but to National Bank. Answer (c) is incorrect because to set aside a
preferential transfer, the debtor must have made the transfer while he was insolvent in the
bankruptcy sense. Therefore, if Hall was presumed insolvent when the mortgage was given, the
trustee is able to set aside the preferential transfer. Note that insolvency is irrelevant to
whether a transfer is preferential or nonpreferential. Answer (d) is incorrect because when Hall
gave National Bank the mortgage to secure payment of the two-year-old loan, this was a
preferential transfer because it attempted to give National Bank more priority than it would
have had as a general unsecured creditor.

For Problems 8-10


Dart Inc., a closely held corporation, was petitioned involuntarily into bankruptcy under
the liquidation provisions of Chapter 7 of the Federal Bankruptcy Code. Dart contested the
petition. Dart has not been paying its business debts as they became due, has defaulted on its
mortgage loan payments, and owes back taxes to the IRS. The total cash value of Dart’s
bankruptcy estate after the sale of all assets and payment of administration expenses is
$100,000.

Dart has the following creditors:


• Fracon Bank is owed $75,000 principal and accrued interest on a mortgage loan secured by
Dart’s real property. The property was valued at and sold, in bankruptcy, for $70,000.
• The IRS has a $12,000 recorded judgment for unpaid corporate income tax.
• JOG Office Supplies has an unsecured claim of $3,000 that was timely filed.
• Nanstar Electric Co. has an unsecured claim of $1,200 that was not timely filed.
• Decoy Publications has a claim of $14,000, of which $2,000 is secured by Dart’s inventory that
was valued and sold, in bankruptcy, for $2,000. The claim was timely filed.

8. What dollar amount would Nanstar Electric Co. receive?


a. $0
b. $ 800
c. $1,000
d. $1,200

Answer: (a) The bankruptcy estate contains $100,000 after the sale of all assets and payment of
administration expenses. The secured debt of $70,000 to Fracon Bank and the secured debt of
$2,000 to Decoy Publications are satisfied first. (This actually takes place as a higher priority
over the administrative expenses.) Therefore, after paying this $72,000 there is $28,000 left.
The $12,000 of unpaid income tax has the next highest priority of those listed. This leaves
$16,000 for the general creditors who filed on time. There are three of these, that is, Fracon
who is owed $5,000 in excess of what the sale of the property brought, JOG who is owed
$3,000, and Decoy who is still owed $12,000 in excess of the security interest. These three
creditors together are owed $20,000 ($5,000 + $3,000 + $12,000). Since this is more than the
$16,000 left, these 3 general creditors’ debts are prorated. The last priority of unsecured
claimants who filed late get nothing. Therefore, Nanstar Electric gets $0.

9. What total dollar amount would Fracon Bank receive on its secured and unsecured claims?
a. $70,000
b. $72,000
c. $74,000
d. $75,000

Answer: (c) The bankruptcy estate contains $100,000 after the sale of all assets and payment of
administration expenses. The secured debt of $70,000 to Fracon Bank and the secured debt of
$2,000 to Decoy Publications are satisfied first. (This actually takes place as a higher priority
over the administrative expenses.) Therefore, after paying this $72,000 there is $28,000 left.
The $12,000 of unpaid income tax has the next highest priority of those listed. This leaves
$16,000 for the general creditors who filed on time. There are three of these, that is, Fracon
who is owed $5,000 in excess of what the sale of the property brought, JOG who is owed
$3,000, and Decoy who is still owed $12,000 in excess of the security interest. These three
creditors together are owed $20,000 ($5,000 + $3,000 + $12,000). Since this is more than the
$16,000 left, these three general creditors’ debts are prorated. Fracon Bank gets money from
both the unsecured and secured claims. From the unsecured claim, Fracon receives a prorated
share or
16,000 × $5,000 = $4,000
$5,000 + $3,000 + $12,000

Add this prorated share of $4,000 to the $70,000 Fracon received from the sold property to
arrive at $74,000.

10. What dollar amount would the IRS receive?


a. $0
b. $ 8,000
c. $10,000
d. $12,000

Answer: (d) The bankruptcy estate contains $100,000 after the sale of all assets and payment of
administration expenses. The secured debt of $70,000 to Fracon Bank and the secured debt of
$2,000 to Decoy Publications are satisfied first. (This actually takes place as a higher priority
over the administrative expenses.) Therefore, after paying this $72,000 there is $28,000 left.
The $12,000 of unpaid income tax has the next highest priority of those listed.

Prepared By: Jaia Shanti M. Mabad

Rachel Anne R. Nacario

(Guererro)

Chapter 1 Partnership Formation

1.On July 1,1997, Monuz and Pardo form a partnership, agreeing to share profits and losses in the ratio
of 4:6,respectively. Monuz contributed a parcel of land that cost him P25,000. Pardo contributed
P50,000 cash. The land was sold for P50,000 on July 1,1997 four hours after formation of the
partnership. How much should be recorded in Munoz capital account on formation of the partnership?

a) P10,000
b) P20,000
c) P25,000
d) P50,000

2.Moonbits partnership had a net income of P8,000.00 for the month ended September 30,1997.

Sunshine purchased an interest in the Moonbits partnership of Liz and Dick by paying Liz P 32,000.00 for
half of her capital and half of her 50 percent profit sharing interest on October 1,1997. At this time Liz
capital balance was P24,000.00 and Dick capital balance was P56,000.00. Liz should receive a debit to
her capital account of:

a) P 12,000.00
b) P 20,000.00
c) P 16,000.00
d) P 26,667.00

3.On March 1,1997, Santos and Pablo formed a partnership with each contributing the following assets:

Santos Pablo
Cash P 30,000 P 70,000
Machinery and Equipment 25,000 75,000
Building -0- 225,000
Furniture & Fixtures 10,000 -0-

The building is subject to a mortgage loan of P80,000, which is to be assumed by the partnership. The
partnership agreement provides that Santos and Pablo share profits and losses 30% and 70%,
respectively. On March 1,1997 the balance in Pablo’s capital account should be:

a) P 290,000.00
b) P 305,000.00
c) P 314,000.00
d) P 370,000.00

4. The business assets of John and Paul appear below:

John Paul
Cash P 11,000 P 22,354
Accounts Receivable 234,536 567,890
Inventories 120,035 260,102
Land 603,000
Building 428,267
Furniture & Fixtures 50,435 34,789
Other Assets 2,000 3,600
Total P 1,020,916 P 1,317,002

Accounts Payable 178,940 243,650


Notes Payable 200,000 345,000
John, Capital 641,976
Paul, Capital 728,352
Total P 1,020,916 P 1,317,002

John and Paul agreed to form a partnership contributing their respective assets and equities subject to
the following adjustments:

a. Accounts receivable of P 20,000 in John’s books and P 35,000 in Paul’s are uncollectible.
b. Inventories of P 5,500 and P 6,700 are worthless in John’s and Paul’s respective books.
c. Other assets of P 2,000 and P 3,600 in John’s and Paul’s respective books are to be written off.

The capital account of the partners after the adjustment will be:

a) John’s P 614,476
Paul’s P 683,052
b) John’s P 615,942
Paul’s P 717,894
c) John’s P 649,876
Paul’s P 712,345
d) John’s P 613,576
Paul’s P 683,350

5. The following is the condensed balance sheet of the partnership Jo, Li and Bi who share profits and
losses in the ratio of 4:3:3.

Cash P 180,000 Accounts Payable P 420,000


Other Assets 1,660,000 Bi, Loan 60,000
Jo, receivable 40,000 Jo, Capital 620,000
Li, Capital 400,000
Bi, Capital 380,000

Total P1,880,000 Total P1,880,000

Assume that the assets and liabilities are fairly valued on the balance sheet and the partnership decides
to admit Mac as a new partner, with a 20% interest. No goodwill or bonus is to be recorded. How much
Mac contributes to cash or other assets?

a) P 350,000
b) P 280,000
c) P 355,000
d) P 284,000

Solution and Explanation

1. D. The requirement is Munoz’ capital account balance upon formulation of the partnership. As in the
case with all entities, investment in the capital of a partnership should be measured at the fair market
value of the assets contributed. In this case, the FMV of the land would be measured at the fair market
value by its sales price on the date of sale (P50,000) which is also the date of the partnership formation.
Recording the land of Munoz’ cost would result in the partners sharing the gain from the sale in
accordance with their profit and loss ratio. This is not equitable since the gain accrued while the land
was held by Monuz.

2. A. Under the admission by purchase only the transfer of the capital purchase by the selling partner
(Liz) to the buying partner (Sunshine) is recorded. Therefore 50% of the capital of Liz (P24,000) or P
12,000 is to be debited to her capital account.

3. A. P 290,000.00

Assets contributed by Pablo P 370,000


Less: Mortgage assumed by partnership (80,000)
Capital balance of Pablo P 290,000

Note that the profit and loss sharing ratio is irrelevant to the solution of this problem.

4. A. John’s P 614,476

Paul’s P 683,052

John Paul
Capital balance before adjustments P641,976 P 728,352
Adjustments:
Uncollectible accounts (20,000) (35,000)
Inventories Written Off (5,500) 6,700
Other Assets written off (2,000) (3,600)
Capital balances after adjustments P 614,476 P 683,052

5. A. P 350,000

Total agreed capital of the new partnership ( 1,400,000 ÷ 80% ) P 1,750,000

Total contributed capital of the old partners ( 1,400,000)

Mac’s contribution P 350,000

Chapter 2 Partnership Operation

Problem

1.On January 1,1997 Zeep and Beep have capital balances of P 20,000.00 and P 16,000.00 respectively.
On July 1,1997 Zeep invest an additional P 4,000.00 and Beep withdraws P 1,600.00. Profits and losses
are divided as follows: Beep is the managing partner and as such shall receive P 16,000.00 salary and
Zeep shall receive P 7,200.00; both partners shall receive interest of 10% on their beginning capital
balances to offset whatever difference in capital investments they have and any remainder shall be
divided equally.

Income of the Zeep-Beep partnership for the year 1997 is P 9,600.00. Zeep’s share in the net income is:

a. P 9,200.00
b. P 880.00
c. P 4,800.00
d. P 600.00

2. Mr. Zoom and his very close friend Mr. Boom formed a partnership on January 1,1997 with Zoom
contributing P 16,000 cash and Boom contributing equipment with a book value of P 6,400.00 and a fair
value of P 8,000.00. During 1997 Boom made additional investments of P 1,600.00 on April 1 and P
1,600.00 on June 1, and on September 1, he withdrew P 4,000.00. Zoom had no additional investment
nor withdrawals during the year. The average capital balance at the end of 1997 for Boom is:

a. P 9,600.00
b. P 8,000.00
c. P 8,800.00
d. P 7,200.00

3. The partnership agreement of Eve and Fred provides that interest at 10% per year is to be credited to
each partner on the basis of weighted-average capital balances. A summary of Fred’s capital account for
the year ended 31 December 1997 is as follows:

Balance, 1 January P 280,000.00


Additional investment, July 1 80,000
Withdrawal, 1 August (30,000.00)
Balance, 31 December 330,000.00

The amount of interest that should be credited to Fred’s capital account for 1997 is

a. P 30,750
b. P 30,500
c. P 34,500
d. P 33,000

4. The partners, A and B, share profits 3:2. However, A is to receive a yearly bonus of 20% of the profits,
in addition to his profit share. The partnership made a net income for the year of P 24,000 before the
bonus. Assuming A’s bonus is computed on profit after deducting said bonus, how much profit share will
B receive?

a. P 15,200
b. P 9,600
c. P 8,000
d. P 9,000

5. A, B, and C are partner in the accounting firm. Their capital account balances at year-end were:
A,P90,000; B,P11,000; C,P50,000. They share profits and losses in a 4:4:2 ratio, after the following
special terms:

1) Partner C is to receive a bonus of 10% of the net income after bonus.


2) Interest of 10% shall be paid on that portion of a partner’s capital in excess of P100,000.
3) Salaries of P10,000 and P12,000 shall be paid to partners A and C, respectively.

Assuming a net income of P 44,000 for the year, the total profit share of partner C would be:

a. P 7,800
b. P 16,800
c. P 19,400
d. P 19,800

Solutions and Explanations

1.D. P 600

Zeep Beep Total


Salaries P 7,200 P 16,000 P 23,200
Interests 2,000 1,600 3,600
Balance, equally (8,600) (8,600) (17,200)
Total P 600 P 9,000 P 9,600

2.C. P 8,800

Date Capital Balances Months Unchanged Peso Months


January P 8,000 3 P 24,000
April 9,600 2 19,200
June 11,200 3 33,600
September 7,200 4 28,800
12 P105,600
Average Capital (105,600÷ 12) = P 8,800

3.A. P 30,750

Date Capital Balances Months Unchanged Peso Months


January P 280,000 6 P 1,680,000
July 360,000 1 360,000
August 330,000 5 1,650,000
P 3,690,000
Average Capital (P 3,690,000 ÷ 12) P 307,500
Interest (P307,500 x 10%) P 30,750

4.C. P 8,000

A B Total
Bonus to A (P24,000 ÷ 120%) x 20% P 4,000 P 4,000
Balance, 3:2 12,000 8,000 P 20,000
Total P 16,000 P 8,000 P24,000

5. C. P 19,400

A B C Total
Bonus, [44,000-(44,000/110%)] - - P 4,000 P 4,000
Interest (10,000 x 10%) - P 1,000 - 1,000
Salaries 10,000 - 12,000 22,000
Balance, 4:4:2 6,800 6,800 3,400 17,000
Total P 16,800 P 7,800 P 19,400 P 44,000

Chapter 3 Partnership Dissolution


1.Cen, Deng and Lala are partners with capital balances on 31 December 1997 of P300,000,P300,000
and P200,000 respectively. Profits are shared equally. Lala wishes to withdraw and it is agreed that she
is to take certain furniture and fixtures with second hand value of P 50,000 and note for the balance of
her interest. The furniture and fixtures are carried in the books at P 65,000. Brand new, the furniture
and fixtures may cost P 80,000. Lala’s acquisition of the second-hand furniture will result to:

a. Reduction in capital of P 15,000 each for Cen and Deng.


b. Reduction in capital of P 10,000 for Lala.
c. Reduction in capital of P 5,000 each for Cen, Deng and Lala.
d. Reduction in capital of P 7,500 each for Cen and Deng.

2.Jaime Dizon, a partner in an accounting firm, decided to withdraw from the partnership. Dizon’s share
of the partnership profits and losses was 20%. Upon withdrawing from the partnership he was paid
P74,000 in final settlement for his interest. The total of the partners’ capital accounts before recognition
of partnership goodwill prior to Dizon’s withdrawal was P210,000. After his withdrawal the remaining
partner’s capital accounts, excluding their share of goodwill, totaled P 160,000. The totaled agreed upon
goodwill of the firm was:

a. P 120,000
b. P 140,000
c. P 160,000
d. P 250,000

3.Cina,Doy and Eli shared profit and losses based on 5:3:2. Eli was allowed to withdraw from the
partnership on 31 December 1997 with P 600,000 cash as full settlement. The condensed balance sheet
of the partnership as of that date was as follows:

Assets
Due from Eli P 250,000
Goodwill 2,000,000
Other Assets 4,750,000
Total Assets P7,000,000

Liabilities and Capital


Liabilities P 2,000,000
Due to Doy 750,000
Cina, Capital 1,750,000
Doy, capital 1,500,000
Eli,Capital 1,000,000
Total liabilities and capital P 7,000,0000

Using the goodwill method, the new capital balances of the remaining partners after Eli’s withdrawal
are:
a. Cina,P 1,843,750 and Doy,P 1,556,250
b. Cina,P 1,375,000 and Doy,P 1,275,000
c. Cina,P 2,000,000 and Doy,P 1,650,000
d. Cina,P 1,750,000 and Doy,P 1,500,000

4.The condensed balance sheet of the partnership of Edong, Frredo and Godo with corresponding profit
and loss sharing percentage as of June 30,1997 was as follows:

Net Assets P400,000


Edong,Capital (50%) P200,000
Fredo,Capital (30%) 120,000
Godo,Capital (20%) 80,000
P 400,000
As of said date, Edong retired from the partnership. By mutual agreement, he was paid P225,000 for his
interest in the partnership.The total implied goodwill was to be recorded. After Edong’s retirement, the
total net assets of the partnership was:

a. P 250,000
b. P 175,000
c. P 200,000
d. P 225,000

5.Pastor, Ramon and Sendong were partners with capital balances as of January 1,1997, of
P100,000,P150,000 and P200,000 respectively, sharing profit and losses on a 5:3:2 ratio.

On July 1,1997 Pastor withdraw from the partnership. Partners agreed that at the time of withdrawal,
certain inventories had to be revalued at P 70,000 from its cost of P 50,000. For the six month period
ending June 30,1997,the partnership generated a net income of P140,000. Further partners agreed to
pay Pastor P195,000 for his interest and that the remaining partner’s capital accounts, would be
adjusted for whatever goodwill the settlement would generate. The payment to Pastor included a
goodwill of:

a. P 15,000
b. P 25,000
c. P 50,000
d. P42,500

Solutions and Explanations


1.C. Reduction in capital of P 5,000 each for Cen, Deng and Lala.

Book value of the furniture and fixtures P 65,000


Second hand value (50,000)
Loss to be shared by Cen, Deng, Lala, equally P15,000

2. A. P 120,000

Partnership capital before withdrawal by Dizon P 210,000


Less: Partnership capital after withdrawal (excluding goodwill) (160,000)
Book value of Dizon's interest P 50,000
Price paid Dizon for 20% interest 74,000
Less: Book value of interest (50,000)
Implied goodwill on 20% interest P 24,000
Implied goodwill on entire firm (P 24,000 ÷ 20%) P 120,000

3. B

Cina Coy Eli


Capital balances before withdrawal of
Eli P 1,750,000 P 1,500,000 P 1,000,000
Due from Eli (250,000)
Goodwill written off (2,000,000) (1,000,000) (600,000) (400,000)
Capital balances before settlement 750,000 900,000 350,000
Settlement with Eli (600,000)
Total goodwill, 1,250,000
(250,000 ÷ 20%) 625,000 375,000 250,000
New capital balances after withdrawal P 1,375,000 P 1,275,000 P -0-

4. D. P 225,000

Edong's capital P 200,000


Settlement 225,000
Goodwill (50%) P 25,000
Net assets before settlement with Edong P 400,000
Settlement (225,000)
Total Implied goodwill (P25,000 ÷ 50%) 50,000
Net assets after Edong's retirement P 225,000
5. A.

Pastor's capital before his retirement P 100,000


Undervaluation of inventory (P 20,000 x 50%) 10,000
Profit share (P140,000 x 50%) 70,000
Pastor's interest P 180,000
Settlement 195,000
Goodwill P 15,000
Chapter 4 Partnership Liquidation

1.Gilbert, Joseph and Li are partners with capital balance of P350,000, P250,000 and P350,000 and
sharing profits 30%, 20% and 50% respectively. Partners agree to dissolve the business and upon
liquidation, all of the partnership assets are sold and sufficient cash is realized to pay all the claims
except one for P 50,000. Li is personally insolvent, but the other two partners are able to meet any
indebtedness to the firm. On the remaining claim against the partnership, Gilbert is to absorb.

a. P 40,000
b. P 15,000
c. P 30,000
d. P 25,000

2.After operating for five years, the books of the partnership of Joe and Letty showed the following
balances:

Net assets P 130,000


Joe, Capital 85,000
Letty, Capital 45,000

If liquidation takes place at this point and the net assets are realized at book value, the partners are
entitled to:

a. Joe to receive P 90,000 & Letty to receive P 40,000


b. Joe to receive P 97,500 & Letty to receive P 32,500
c. Joe to receive P 65,000 & Letty to receive P 65,000
d. Joe to receive P 85,000 & Letty to receive P 45,000

3. Partners Beth, John and Star who shared profit and losses based on 4:4:2 decided to liquidate. All
assets of the partnership were liquidated.

The condensed balance sheet just prior to liquidation follows:

Cash P 100,000 Liabilities P 140,000


Other Assets 400,000 Beth,loan 10,000
Beth, Capital 45,000
John, Capital 105,000
  Star, Capital 200,000
Total P 500,000 Total P 500,000

Other assets were sold for P 247,500 realizing a loss of P 152,500. Parties agreed to fully terminate the
partnership’s business thus, necessitating distribution of cash to partners and in the event of capital
deficiency, contribution of additional cash. The three partners were all solvent and could answer any
capital deficiency. The realization of assets, distribution of loss and payment of liabilities resulted to the
following partner’s loan and capital accounts balances prior to final cash settlement:

Beth,Loan Beth,Capital John,Capital Star,Capital


a. P 10,000 P 10,000 P 50,000 P 165,000
b. 10,000 (16,000) 44,000 169,500
c. 10,000 15,000 55,000 165,000
d. 10,000 45,000 105,000 200,000

4. Bach, Johann and Straus were partners sharing profits and losses based on 4:4:2 decide to liquidate.
All assets of the partnership were liquidated. The condensed balance sheet just prior to liquidation
follows:

Assets Liabilities & Capital


Cash P 100,000 Liabilities P 140,000
Other Assets 400,000 Beth,loan 10,000
Beth, Capital 45,000
John, Capital 105,000
  Star, Capital 200,000
Total P 500,000 Total P 500,000

Other assets were sold for P 247,500 realizing a loss of P 152,500. Parties agreed to fully terminate the
partnership’s business thus, necessitating distribution of cash to partners and in the event of capital
deficiency, contribution of additional cash. The three partners were all solvent and could answer any
capital deficiency.

Name the partner and give the corresponding additional cash he had to invest due to his net capital
deficiency to finally settle the liquidation of the partnership.

a. Bach, P 16,000
b. Johann, P 44,000
c. Bach, P 6,000
d. Straus, P 30,500

5.Silverio, Domingo, Reyes and Pastor are partners, sharing earnings in the ratio of 3/21, 4/21, 6/21 and
8/21, respectively. The balances of their capital accounts on December 31,1997 are as follows:

Silverio……………………………………………… P 1,000

Domingo …………………………………………. 25,000

Reyes ……………………………………………… 25,000


Pastor …………………………………………….. 9,000

The partners decide to liquidate, and they accordingly convert the non-cash assets into P 23,200 of cash.
After paying the liabilities amounting to P 3,000, they have P 22,200 to divide. Assume that a debit
balance of any partner’s capital is uncollectible.

The share of Silverio in the loss upon conversion of the non-cash assets into cash was:

a. P 4,972
b. P 5,257
c. P 5,400
d. P 5,200

Solutions and Explanations

1.A. P 40,000

Total Gilbert Joseph Li


*Capital balances before realization P 950,000 P 350,000 P 250,000 P 350,000
Loss on realization (squeeze) (1,000,000) (300,000) (200,000) (500,000)
Capital balances after realization (50,000) 50,000 50,000 (150,000)
Absorption of Li's deficiency - (90,000) (60,000) 150,000
Additional Investment (50,000) (40,000) (10,000) -

*The total capital realization should be equal to the unpaid liabilities.

2. D. Joe to receive P 85,000 & Letty to receive P 45,000

Since no gain or loss on realization is to be distributed to the partner, the partners are entitled to receive
cash equal to their capital balances.

3. B. Beth,Loan P 10,000; Beth,Capital (P16,000); John,Capital P 44,000; Star,Capital P 169,500

Capital
Beth,Loan Beth John Star
Balance before realization P 10,000 P 45,000 P 105,000 P 200,000
Loss on realization,P152,500,4:4:2 -  (61,000) (61,000) (30,500)
Balances P 10,000 (P 16,000) P 44,000 P 169,500
4. C. Bach, P 6,000

Bach Johann Straus


Capital balances P 45,000 P 105,000 P 200,000
Loan balance 10,000 - -
Total interest 55,000 105,000 200,000
Loss on realization P 152,500,4:4:2 (61,000) (61,000) (30500)
Balances (6,000) (44,000) (169,500)
Additional Investment by Bach 6,000 - -
Payment to partner P -0- P 44,000 P 169,500

5. C. P 5,400

Book value of non-cash asset (Sch. 1) P 61,000


Cash Realization (23,200)
Loss on realization 37,800
Silverio's P/L ratio 3/21
Silverio's share P 5,400

Schedule 1:
Payment to partners P 22,200
Add back liabilities paid 3,000
Cash balance after realization 25,200
Less: Cash realized from sale of assets 23,200
Cash balance before realization 2,000
Total assets (P60,000+P3,000) 63,000
Book value of non-cash assets P 61,000
Chapter 5 Installment Liquidation

Problems

1. Kay and Loy, partners who share profits and losses equally decided to liquidate their partnership
business in installment. The statement of financial position showed Cash,P35,000; Liabilities,P20,000;
Kay capital,P71,000; and Loy capital,P54,000. Anticipated liquidation expenses amounts to P10,000.

How much cash can be distributed safely to each partner at this point?

Kay Loy
a. P 5,000 P -0-
b. P 5,000 P 500
c. P 3,000 P -0-
d. P 5,000 P 1,000

2. The partnership of Javier, Karim, and Laurel share profits and losses in the ratio of 5:3:2, respectively.
The partners voted to dissolve the partnership when its assets, liabilities, and capital were as follows:

Assets Liabilities and Capital

Cash P40,000 Liabilities P60,000


Other Assets 210,000 Javier,Capital 48,000
Karim,Capital 72,000
Laurel,Capital 70,000

P250,00
Total P250,000 Total 0

The partnership will be liquidated over a prolonged period of time. As cash is available it will be
distributed to the partners. The first sale of non-cash assets having a book value of P120,000 realized
P90,000. How much cash should be distributed to each partner after this sale?

a. Javier, P0; Karim, P28,800; Laurel, P41,200


b. Javier, P0; Karim, P30,000; Laurel, P40,000
c. Javier, P35,000; Karim, P21,000; Laurel, P14,000
d. Javier, P45,000; Karim, P27,000; Laurel, P18,000

3. The condensed statement of financial position of Alex, Jay and John partnership as of March 31,2011
follows:
Cash P28,000
Other assets 265,000
Total P293,000

Liabilities P48,000
Alex,Capital 95,000
Jay,Capital 80,000
John,Capital 70,000
Total Unsecured Liabilities P293,000

Income and loss ratio is 50:25:25 respectively. The partners voted to dissolve the partnership and
liquidate by selling assets in installments. P70,000 was realized on the first cash sale of other assets
which has a book value of P150,000. After settlement with creditors, all cash available was distributed to
partners. How much cash was received by John?

a. P10,500
b. P32,500
c. P21,250
d. P20,000

4.The following statement of financial position is for the partnership of D, E and F:

Cash P20,000   Liabilities P50,000


Other assets 180,000 D,Capital (40%) 37,000
E,Capital (40%) 65,000
F,Capital (20%) 48,000

P200,00
Total P200,000   Total 0

Figures shown parenthetically reflect agreed profit and loss sharing percentages.

If the firm as shown on the original balance sheet, is dissolved and liquidated by selling assets in
installments, the first sale of non-cash assets having a book value of P90,000 realizes P50,000, and cash
of P17,000 after settlement with creditors is distributed; the respective partners would receive ( to the
nearest peso).

a. D, P8,000; E, P 8,000; F, P 4,000


b. D, P6,667; E, P 6,667; F, P 6,666
c. D, P 0; E, P13,333; F, P 6,667
d. D, P 0; E, P 1,000; F,P16,000
5. L, M, N and O partners to a law firm share profits 5:3:1:1 respectively. Partner’s accounts prior to
liquidation were as follows:

  Advances (Dr) Loans (Cr) Capitals (Cr)


L - P5,000 P40,000
M - 10,000 30,000
N P 4,500 - 15,000
O 2,500 -  25,000

At this point, cash of P18,000 is available for distribution to the partners. How much of the P18,000 cash
should be distributed to each partner?

L M N O
a. P -0- P 18,000 P -0- P -0-
b P -0- P -0- P -0- P 18,000
c. P -0- P 6,625 P -0- P 11,375
d. P 9,000 P 5,400 P 1,800 P 1,600
Solutions and Explanations

1.A. Kaye P 5,000 ; Loy, P -0-

Kay Loy
Capital balances before cash distribution P71,000 P54,000
Possible loss, [(P20,000 + 125,000)-P35,000]
P110,000 + 10,000 = P 120,000 (60,000) (60,000)
Balances P 11,000 P (6,000)
Additional loss to Kay (6,000) 6,000
Payment to Partner Kay P5,000 P-0-

2. A. Javier, P0; Karim, P28,800; Laurel, P41,200

Javier Karim Laurel


Balances P 48,000 P72,000 P 70,000
Loss on sale of assets,P30,000 (15,000) (9,000) (6,000)
Balances 33,000 63,000 64,000
Possible loss,P90,000 (45,000) (27,000) (18,000)
Balances (12,000) 36,000 46,000
Absorption of Javier's deficit 12,000 (7,200) (4,800)
Cash Distribution -0- P 28,800 P 41,200

3. D. P20,000

Alex Jay John


Capital before realization P95,000 P80,000 P70,000
Loss on realization,P80,000 (40,000) (20,000) (20,000)
Capital before cash distribution 55,000 60,000 50,000
Possible Loss, P 115,000 (57,500) (28,750) (28,750)
Balances (2,500) 31,250 21,250
Additional loss to Jay & John, 25:25 2,500 (1,250) (1,250)
Payment to Partners P-0- P 30,000 P 20,000

4. D. D, P – 0-; E, P 1,000; F,P 16,000

D E F
Capital balances before realization P 37,000 P 65,000 P 48,000
Loss on realization (16,000) (16,000) (8,000)
Balances 21,000 49,000 40,000
Possible loss:
Other Assets (P180,000-90,000) P90,000
Cash Withheld: (P20,000+50,000-
50,000-17,000 3,000
Total P93,000 (37,200) (37,200) (18,600)
Balances (P16,200) P11,800 P21,400
Additional loss to E & F, 4:2 16,200 (10,800) (5,400)
Payment to partners P-0- P1,000 P16,000

5. C. L, P-0-; M, P6,625; N,P-0-; O, P 11,375

L M N O
Capital balances P40,000 P30,000 P15,000 P25,000
Loan balances 5,000 10,000 - -
Advances --- --- (4,500) (2,500)
Total Interest P45,000 P40,000 P10,500 P22,500
Possible loss (P118,000-18,000) (50,000) (30,000) (10,000) (10,000)
Balances (5,000) 10,000 500 12,500
Additional loss to MNO, 3:1:1 5,000 (3,000) (1,000) (1,000)
Balances - 7,000 (500) 11,500
Additional loss to MO, 3:1 - (375) 500 (125)
Payment to M and O P-0- P6,625 P-0- P 11,375
Chapter 6 Corporate Liquidation

Problems

1. Sayap Company signed a note payable to its bank for P2000000.Accrued interest on the note on
February 29,2000 amounts to P50,000.The note is secured by inventory with a book value of
P2,300,000.The inventory is sold for P1,600,000 and unsecured creditors receive 30% of their claims.
What amount should the bank receive in settlement of the note and interest?

a. P2,050,000

b. P2,000,000

c. P1,705,000

d. P1,600,000

2. The following are the data presented by Ilocos Company:

Assets at book value P1,000,000


Assets at net realizable value 750,000
Liabilities at book value:
Fully Secured Mortgage 400,000
Unsecured accounts and notes payable 450,000
Unrecorded Liabilities:
Interest on bank notes 2,500
Estimated administrative expenses 40,000

The Statement of Affairs at this time should include an estimated deficiency to unsecured creditors of:

a. P350,000
b. P310,000
c. P142,500
d. P100,000

3. The Statement of Affairs for the Failed Company contained the following relevant information:

Assets pledged wit fully secured creditors P1,000,000


Assets pledged with partially secured creditors 500,000
Free Assets 600,000
Liabilities with Priority 100,000
Fully Secured Liabilities 800,000
Partially Secured Liabilities 750,000
Unsecured Liabilities 900,000
All assets are stated at net realizable values. The unsecured creditors should receive what percentage of
their claims in liquidation? (Round to two decimal places.)

a. 60.87%
b. 64%
c. 74.29%
d. 82.35%

4. Luck Company has filed for liquidation. The following data is available:

Free assets at net realizable value P 100,000


Liabilities per book 160,000
Unrecorded liabilities:
Liquidation expenses 6,000
Unpaid wages with priority claim 10,000

What percentage of their claims should the unsecured creditors receive in liquidation?

a. 62.5%
b. 56.82%
c. 55.29%
d. 52.5%

5. The Abu Company in liquidation provided the following data:

Assets at book value P 100,000


Assets at net realizable value 75,000
Liabilities at book value 85,000
Unrecorded Liabilities: Interest on bank notes 250
Liquidation expenses 4,000

The journal entry to record the assets and liabilities should include estate deficit of:

a. P14,250
b. P14,000
c. P10,250
d. P10,000
Solutions and Explanations

1.C. P1,705,000

P1,600,00
Proceeds for inventory 0
Unsecured Note balance and interest at 30%
(P300,000+P50,000 x 30%) 105,000
P1,705,00
Payment to bank in settlement of note and interest 0

2.C.P142,500

Total Assets at realizable value P750,000


Fully secured liabilities (400,000)
Priority claim: Estimated administrative expenses (40,000)
Estimated amount available 310,000
Unsecured Claims (P450,000+P2,500) (452,500)
Estimated deficiency to unsecured creditors (P 142,500)

3.A.P1,150,000

Assets available for unsecured creditors:


Fully secured assets P1,000,000
Liabilities thereon (800,000)
Available for unsecured creditors 200,000
Free assets 600,000
Priority liabilities 100,000 700,000
Unsecured liabilities 900,000
Partially secured liabilities 750,000
Assets pledged thereon (500,000) 250,000
Total Unsecured Liabilities 1,150,000

Percentage to be paid: P700,000 / P1,150,000 = 60.87%

4.D.52.5%

Free assets P100,000


Less: Priority claims
Liquidation expenses P6,000
Unpaid wages 10,000 (P16,000)
Net free assets P84,000
Divided by unsecured liabilities 160,000
Recovery percentage 52.50%

5.C.P10,250

Assets at realizable value P75,000


Less: Liabilities:
Per books P85,000
Unrecorded 250 (85,250)
Estate deficit (P 10,250)
PARTNERSHIP FORMATION

1. A partnership is a (an):

I. Accounting entity

II. Taxable entity

a. I only

b. II only

c. Neither I or II

d. Both I and II

2. Which of the following accounts can be found in the MN partnerships' general ledger?

I. Receivable from M

II. M drawing

III. M loan

a. I only

b. I and II

c. I, II and III

d. II and III

3. Which of the following statements about partnership accounts is true?

a. Two accounts are generally maintained for each partner.

b. The drawing account is credited with the partner's withdrawals of cash or other assets during the
period.

c. Answer (a) is correct but (b) is false

d. Answers (a), (b) and (c) are all correct.


4. The partners' drawing accounts are used:

a. To record the partner's salaries

b. To reduce the partner's capital account balances at the end of the period.

c. In the same manner as the partners' loan accounts.

d. To record the partner' share of net income or loss for an accounting period.

5. A partnership is formed by two individuals who were previously sole proprietors. Property other than
cash that is part of the initial investment in the partnership is recorded for financial accounting purposes
at the:

a. Proprietor's book value or the fair value of the property at the date of the investment, whichever is
higher.

b. Proprietor's book values or the fair value of the property at the date of the investment, whichever is
lower.

c. Proprietor's book values of the property at the date of investment.

d. Fair value of the property at the date of investment.

PARTNERSHIP OPERATIONS

1. Which of the following is an expense of a partnership?

a. Interest on partners' capital account balances.

b. Interest on loans from partners to the partnership.

c. Both a and b.

d. Neither a nor b.

2. A partner's withdrawal of assets from a partnership that is considered a permanent reduction in that
partner's equity is debited to the partner's:
a. Drawing account

b. Retained earnings account

c. Capital account

d. Loan receivable account

3. The allocation of an error should be based on the profit and loss ratio in effect when:

a. The error was made

b. The error was corrected

c. The error was discovered

d. The allocation should always be made equally

4. If the partnership agreement provides for the division of losses only, profits should be divided:

a. Equally

b. According to beginning capital ratio

c. According to ending capital ratio

d. According to average capital ratio

5. If there is no provision for division of profits but not losses in the partnership agreement, it is
concluded that:

a. Losses should not be divided to the capital accounts, but matched against future earnings.

b. Losses should be divided using the same approach as division of profits.

c. Losses should be divided equally

d. Losses should be allocated according to the ratio of capital account balances.

PARTNERSHIP DISSOLUTION
1. Which of the following conditions constitutes a legal dissolution of a partnership?

a. Death of a partner

b. Retirement of a partner

c. Admission of a partner

d. All of the above

2. When admitting a new partner into an existing partnership, any allocation of goodwill to the old
partners is based on:

a. The profit and loss ratio

b. An equal distribution among the partners

c. The fair values of the assets each partner has contributed to the partnership

d. The relative capital balances of the partners

3. According to the text, the recognition of goodwill in the accounting records of a partnership may be
appropriate for:

a. The admission of a new partner for a cash investment

b. The retirement of an existing partner

c. Either of the foregoing situations

d. Neither of the foregoing

4. When the investment of a new partner exceeds the new partners' initial capital balance and goodwill
is not recorded, who will receive the bonus?

a. The new partner

b. The old partners in their old profit and loss ratio

c. The old partners in their new profit and loss ratio

d. The old and new partners in their new profit and loss ratio
5. Under what circumstances will the bonus and goodwill methods produce different results in the
future?

a. If the new partners' percentage interest in profits and losses is the same as his initial fractional
interest in the partnership capital.

b. If the new percentage interests in profits and losses of the old partners is in the same relative
proportion as their old percentage interests.

c. If any partner retires in the near future

d. If the partnership realizes losses, rather than profits

PARTNERSHIP LIQUIDATION : LUMP-SUM

1. When is a partnership legally insolvent?

a. When the partnership assets are insufficient to meet partnership liabilities

b. When the partnership assets are insufficient to meet the partnership liabilities and at least one
partner is personally insolvent

c. When all the partners are personally insolvent

d. When the assets of the partnership plus the assets of all the partners are insufficient to meet the
partnership plus the individual partners' liabilities

2. In which order are partnership assets distributed to partners under the Partnership Law?

a. Capital balances, loans, profits

b. Loans, profits, capital balances

c. Loans, capital balances, profits

d. Profits, capital balances, loans

3. What is the rule of offset?

a. Receivables from partners should offset against their debit capital balances before they receive any
cash distributions.
b. Loans to partners should offset against their debit capital balances before they receive any cash
distributions.

c. Loans from partners should offset against their credit capital balances before the receive any cash
distributions.

d. Loans from partners should offset against their debit capital balances before they receive any cash
distributions.

4. If a partnership is liquidated, how is the final allocation of business assets made to the partners?

a. Equally

b. According to the profit and loss ratio

c. According to the final capital account balances

d. According to the initial investment made by each of the partners

5. If Juan, a partner with a loan receivable from a liquidating partnership, receives less cash than the
amount of the loan during the liquidation, the payment is recorded with a debit to:

a. Loan Receivable from Juan

b. Juan Capital

c. Juan drawing

d. Loan Payable to Juan

PARTNERSHIP LIQUIDATION: INSTALLMENT

1. In the installment liquidation of a partnership, each installment of cash is distributed:

a. In the partners' profit and loss ratio

b. In the ratio of partner's capital account balances

c. As agreed to by the partners

d. As if no more cash would be forthcoming


2. In the cash distribution plan which partner gets the first cash distribution?

a. The partner with the largest loan balance

b. The partner with the largest loss absorption balance

c. The partner with the largest capital balance

d. The partner with the largest profit and loss ratio

4. In calculating the safe payment, you assume:

a. Parynership liabilities have been paid

b. No liquidation expenses will be paid

c. All non-cash assets are worthless

d. Cash on hand can fully be distributed

5. In the preparation of schedule of safe payment to partners, cash withheld foe future liquidation
expenses and unrecorded liabilities that may be discoveres is trwated as:

a. Operating expenses

b. Liabilities

c. Loss on realization

d. Possible loss

CORPORATE LIQUIDATION

1. The number of classes of creditors in a corporate liquidation is:

a. Two

b. Three
c. Four

d. Five

2. A category of assets that typically has zero in the free assets column of a statement of affairs is:

a. Factory supplies inventory

b. Tools

c. Short-term prepayments

d. None of the above

3. Insolvency in corporate liquidation means:

a. Book value of assets is greater than liabilities

b. Fair value of assets is less than liabilities

c. Inability to meet financial obligations as they come due

d. Liabilities are greater than book value of assets

4. In the reporting of a corporate liquidation, assets are shown at:

a. Present Value calculated using an appropriate effective rate

b. Net realizable value

c. Historical cost

d. Book value

5. Which of the following is not a liability that has priority in liquidation?

a. Administrative expenses incurred in the liquidation

b. Salary payable owed to employeed

c. Payroll taxes due to the government


d. Advertising expense incurred before the company became insolvent

Partnership formation

1. Partnership capital and drawing accounts are similar to the corporate


A. Paid-in capital, retained earnings, and dividend accounts
B. Retained earnings account.
C. Paid-in capital and retained earnings accounts.
D. Preferred and common stock accounts
2. For individuals who were previously sole proprietors form a partnership. Each partner
contributes inventory and equipment for use by the partnership. What basis should the
partnership use to record the contributed assets?
A. Inventory at the lower of FIFO cost or market.
B. Inventory at the lower of weighted-average cost or market.
C. Equipment at each proprietor’s carrying amount.
D. Equipment at fair value.
3. Meca and Came formed a partnership on January 1,2015 with each contributing the
following assets:
Meca Came
Cash P30,000 P70,000
Machinery 25,000 75,000
Inventory 10,000 -
Building 225,000
The building is subject to a mortgage loan of P90,000 which is to be assumed by the
partnership. On January 1,2015, the capital account of Came would show a balance of:
A. P280,000 B. P305,000 C. P314,000 D. P370,000
4. The business assets of LL and MM appears below:
LL MM
Cash P11,000 P22,354
Accounts Receivable 234,536 567,890
Inventories 120,035 260,102
Land 603,000 ----------
Building ---------- 428,267
Furniture and Fixtures 50,345 34,789
Other Assets 2,000 3,600
P1,020,916 P1,317,002
Accounts Payable P178,940 P 243,650
Notes Payable 200,000 345,000
LL,Capital 641,976 -------------
MM, Capital ---------- 728,352
P1,020,916 P1,317,002
LL and MM agreed to form a partnership contributing their respective assets and equities
subject to the following adjustments:
a. Accounts receivable of P20,000 in LL’s books and P35,000 in MM’s are
uncollectible.
b. Inventories of P5,500 and P6,700 are worthless in LL’s and MM’s respective books.
c. Other assets of P2,000 and P3,600 in LL’s and MM’s respective books are to be
written off.
The capital account of the partners after the adjustments will be:
a. LL, P615,942; MM, P717,894 c. LL, P640,876; MM, P683,050
b. LL, P640,876; MM, P712,345 d. LL, P614,476; MM, P683,052

5. Langley invests his delivery van in a computer repair partnership with McCurdy. What
amount should the van be credited to Langley’s partnership capital?
A. The tax basis.
B. The fair value at the date of contribution.
C. Langley’s original cost.
D. The assessed valuation for property tax purposes.
6. On April 30, 1993, Algee, Belger, and Ceda formed a partnership by combining their
separate business proprietorships. Algee contributed cash of $50,000, Belger contributed
property with a $36,000 carrying amount, a $40,000 original cost, and $80,000 fair value.
The partnership accepted responsibility for the $35,000 mortgage attached to the
property. Ceda contributed equipment with a $30,000 carrying amount, a $75,000
original cost, and $55,000 fair value. The partnership agreement specifies that profits and
losses are to be shared equally but is silent regarding capital contributions. Which
partner has the largest April 30, 1993, capital account balance?
A. Algee. c. Ceda.
B. Belger. d. All capital account balance are equal.

7. Jamby and Minam just formed a partnership. Jamby contributed cash of P2,205,000 and
office equipment that cost P945,000. The equipment had been used in her sole
proprietorship and had been 70% depreciated, the appraised value of the equipment is
P630,000. Jamby also contributed a note payable of P210,000 to be assumed by the
partnership. Jamby is to have 60% interest in the partnership. Miriam contributed only
P1,575,000 merchandise inventory at fair market value. Assume the use of bonus method,
the partners’ capital must be in conformity with their profit and loss ratio upon formation.
In the formation of a partnership, which of the following is true?
A. The agreed capital of Jamby upon formation is P2,625,000
B. The total agreed capital of the partnership is P4,375,000
C. The capital of Miriam will increase by P105,000 as a result of the transfer of capital
D. There is either an investment or withdrawal of asset under the bonus method
8. Alley and Barvey established a partnership on December 1, 20x4. They agreed that Alley
will contribute cash of P20,000; Land of P15,000 and Building of P50,000. Alley’s
accounts payable of P10,000 is to be assumed by the partnership. Barvey will contribute
cash of P30,000 and furniture and fixtures of P25,000.
Assume that each partner initially should have an equal interest in partnership capital
with no contribution of intangible asset (bonus method). How much are the capital
balances of each partner?
a. P85,000 for Alley and P55,000 for Barvey
b. P65,000 for Alley and P65,000 for Barvey.
c. P75,000 for Alley and P55,000 for Barvey
d. P75,000 for Alley and P75,000 for Barvey.

9. The partnership agreement is an express contract among the partners (the owners of the
business). Such an agreement generally does not include
a. A limitation on a partner’s liability to creditors.
b. The rights and duties of the partners.
c. The allocation of income between the partners.
d. The rights and duties of the partners in the event of partnership dissolution.
Partnership operation

1. The XYZ partnership provides a 10% bonus to Partner Y that is based upon partnership
income, after deduction of the bonus. If the partnership's income is P121,000, how
much is Partner Y's bonus allocation?
a. P11,000.
b. P11,450.
c. P11,650.
d. P12,100.

2. If the partnership agreement provides a formula for the computation of a bonus to the
partners, the bonus would be computed
A. next to last, because the final allocation is the distribution of the profit residual.
B. before income tax allocations are made.
C. after the salary and interest allocations are made.
D. in any manner agreed to by the partners.

3. The income statement of Vita Plus Partnership for the year ended December 31, 2007
appear below:

Vita Plus Partnership

Income Statement

For the year ended December 31, 2007

Sales P300,000

Less: Cost of Goods Sold 190,000

Gross Profit P110,000

Less: Operating Expenses 30,000


Net Income P 80,000

Additional Information:

1. Melon and Dalandan began the year with capital balances of P40,800 and P112,000,
respectively.

2. On April 1, Melon invested an additional P15,000 into the partnership and on August 1,
Dalandan invested an additional P20,000 into the partnership.

3. Throughout 2007, each partner withdrew P400 per week in anticipation of partnership net
income. The partners agreed that these withdrawals are not to be included in the computation
of average capital balances for purposes of income distribution.

Melon and Dalandan have agreed to distribute partnership net income according to the
following plan:

MELON DALANDAN

1. Interest on average capital balances 6% 6%

2. Bonus of net income before the bonus but after interest

on average capital balances 10%

3. Salaries P25,000 P30,000

4. Residual (if positive) 70% 30%

5. Residual (if negative) 50% 50%

The share of Melon and Dalandan on the net income, respectively is:
a. P40,473 and P39,527 c. P40,342 and P39,658

b. P40,282 and P39,718 d. P38,935 and P41,065


4. The partnership agreement for the partnership of Brisbane and Ric provided for salary
allowances of P450,000 to Brisbane and P350,000 to Ric, and the residual profit was
allocated equally. During 2007, Brisbane and Ric each withdraw cash equal to 80 percent
of their salary allowances. If during 2008, the partnership had profit in excess of
P1,000,000 without regard to salary allowances and withdrawals, Brisbane’s equity in the
partnership would
a. Increase more than Ric’s c. Increase the same as Ric’s
b. Decrease more than Ric’s d. Decrease the same as Ric’s

5. Jinky is trying to decide whether to accept a bonus of 162,500 or a salary of P97,500 plus
a bonus of 10% of net income after salaries and bonus as a means of allocating profit
among the partners. Salaries traceable to the other partners are estimated to be P450,000.
What amount of income would be necessary so that Jinky would consider the choices to
be equal?
a. P1,100,000 b. P1,197,500 c. P650,000 d. P1,262,500

6. During 1994, Young and Zinc maintained average capital balances in their partnership of
$160,000 and $100,000 respectively. The partners receive 10% interest on average
capital balances, and residual profit or loss is divided equally. Partnership profit before
interest was $4,000. By what amount should Zinc’s capital account change for the year?
A. $1,000 decrease.
B. $2,000 increase.
C. $11,000 decrease.
D. $12,000 increase.
7. Partners A and B have a profit and loss agreement with the following provisions: salaries
of 30,000 and 45,000 for A and B, respectively; a bonus to A of 12% of net income after
salaries and bonus; and interest of 10% on average capital balances of 50,000 and 65,000
for A and B, respectively. One-fourth of any remaining profits are allocated to A and the
balance to B. If the partnership had net income of 108,600, how much should be allocated
to Partner A?
A. 43,225
B. 43,816
C. 47,850
D. 65,375

8. Raymond, Jane, and Venus are partners and share profits and losses as follows: Salaries
of P20,000 to Raymond; P15,000 to Jane; and none to Venus. If net income exceeds
salaries, then a bonus is allocated to Raymond. The bonus is 5 % of net income after
deducting salaries and the bonus. Residual profits or residual losses are allocated 10 % to
Raymond; 20 % to Jane, and 70 % to Venus. After the allocation was recorded and the
books were closed, the partners discovered an error and that correction of the error would
reduce the net income from P70,000 to P30,000. The error involved understated
depreciation expense.
How much is the necessary adjustment to Raymond’s capital?

A. Increase by P25,000
B. Increase by P19,500
C. Decrease by P5,500
D. Decrease by P7,667
Partnership dissolution

1. Perez, Que and Ramos are partners sharing earnings in the ratio of 5:3:2, respectively. As of
December 31, 2008, their capital balance showed P95,000 for Perez, P80,000 for Que, and
P60,000 for Ramos.
On January 1,2009 the partnership admitted Santos as a new partner and according to the
agreement, Santos will invest P80,000 in cash to the partnership and will also purchase 15%
of Que’s interest for P10,000. Santos will share 20% in the earnings while the ratio of the
original partners will remain proportionately the as before Santos’ admission. After Santos’
admission, the total capital of the partnership will be P330,000 while Santos’ capital account
will be P70,000.

What is the balance of Que’s capital account after the admission of Santos?

P81,100 b. P79,100 c. P74,600 d. P72,600


2. Perez, Reyes and Suarez were partners with capital balances as of January 1, 2009 of
P100,000, P150,000 and P200,000 respectively. They share profits on a 5:3:2 ratio.

On July 1, 2009, Perez withdraw from the partnership. For the six month period ending June
30, 2009, the partnership generated a net income P140,000. Partners agreed that at the time
of withdrawal, certain inventory had to be revalued at P70,000 from its cost of P50,000.
Further, partners agreed to pay Perez P195,000 for his interest.

What are the capital balances of Reyes and Suarez after Perez’s retirement?
Reyes Suarez

a. P217,000 P238,000
b. P189,000 P226,000
c. P177,000 P218,000
d. P187,500 P226,000

3. A, B and C have capital balances of P112,000, P130,000 and P58,000, respectively, and
share profits in the ratio 3:2:1. D invest cash in the partnership for a one-fourth interest.
Assume D receives a one-fourth interest in the assets of the partnership and D is credited
with P20,000 of the bonus from the old partners that is recognized upon D’s admission.
How much cash D invest?
a. P73,333 * b. P100,000 c. P93,333 d. P80,000

4. The goodwill and bonus methods are two means of adjusting for differences between the net
book value and the fair value of partnerships when new partners are admitted. Which of the
following statement about these methods is correct?
a. The bonus method does not revalue assets to market values.

b. The bonus method revalues assets to market values.

c. Both methods result in the same balances in partner capital accounts.

d.Both methods result in the same total value of partner capital accounts, but the individual

capital accounts vary.

5. The total of the partners’ capital accounts was P110,000 before the recognition of partnership
goodwill in preparation for the withdrawal of a partner whose income and loss sharing ratio
is 2/10. He was paid P28,000 by the firm in final settlement for his interest. The remaining
partners’ capital accounts, excluding their share of the goodwill, totalled P90,000 after his
withdrawal. Compute the total goodwill of the firm agreed upon.
a. P20,000 b. P48,000 c. P8,000 d. P40,000 *
6. Partner’s Rachel, Cecil, and Arlene share profits and losses 5:3:2, respectively, and their
balance sheet on October 31, 2007 follows:
Cash P 240,000 Accounts Payable P 600,000
Other Assets 2,160,000 Rachel, Capital 444,000
Cecil, Capital 780,000
Arlene, Capital 576,000

P 2,400,000 P 2,400,000

The assets and liabilities are recorded at their current fair value. Lark is to be admitted as a new
partner with a 1/5 interest in capital and earnings. Rachel was credited a bonus of P15,000. How
much should Lark contribute?

a. P456,000 b. P450,000 c. P480,000 d. P487,500


7. Under the bonus method, when a new partner is admitted to the partnership, the total capital
of the new partnership is equal to:
A. the book value of the previous partnership plus the fair market value of the consideration
paid to the existing partnership by the incoming partner
B. the book value of the previous partnership plus any necessary asset write ups from book
value to market value plus the fair market value of the consideration paid to the existing
partnership by the incoming partner
C. the book value of the previous partnership minus any asset write downs from book to market
value plus the fair market value of the consideration paid to the existing partnership by the
incoming partner
D. the fair market value of the new partnership as implied by the value of the incoming partner's
consideration in exchange for an ownership percentage in the new partnership

8. The following balance sheet is presented for the partnership of A, B, and C who share profits
and losses in the ratio of 5:3:2 respectively
Cash 60,000

Other assets 540,000

Liabilities 140,000

A, capital 280,000

B, capital 160,000

C, capital 20,000

Assume that the assets and liabilities are fairly valued on the balance sheet and the partnership
decided to admit D as a new partner with a one-fifth interest. No goodwill or bonus is to be
recorded how much should D contributed in cash or other assets?

A. P120,000 B. P115,000 C. P92,000 D. P73,600

9. A, B and C have capital balances of P112,000, P130,000 and P58,000, respectively, and
share profits in the ratio 3:2:1. D invest cash in the partnership for a one-fourth interest.
Assume D receives a one-fourth interest in the assets of the partnership and B is credited with
P15,000 of the bonus from D, how much cash D invest?
a. P115,000 b. P105,000 c. P160,000 * d. P120,000

10. Jaime Dizon, a partner in an accounting firm, decided to withdraw from the partnership.
Dizon’s share of the partnership profits and losses was 20%. Upon withdrawing from the
partnership, he was paid P74,000 in final settlement for his interest. The total of the partners’
capital accounts before recognition of partnership goodwill prior to Dizon’s withdrawal was
P210,000. After his withdrawal, the remaining partners’ capital accounts, excluding their
share of goodwill, totalled P160,000. The implied goodwill of the firm was:
a. P120,000

b. P140,000

c. P160,000

d. P250,000
Partnership liquidation (lump sum)

1. R, S and T decided to dissolve the partnership on November 30, 2015. Their capital
balances and profit ratios follow: R, capital- P50,000 (40%); S, Capital- P60,000 (30%);
and T, capital- P20,000 (30%). The net income from January 1 to November 30,2015 is
P54,000. Also, on this date, cash and liabilities are P40,000 and P90,000 respectively.
On November 30,2015, the book value of the non-cash assets amounted to:
A. P234,000 B. P243,000 C. P274,000 D. P314,000

2. On September 21, 2008, Tina, Donna and Gina formed a partnership investing cash of
P189,000, P170,100 and P52,920, respectively. The partners share profits 3:2:2 and on
October 17, 2008, they have cash of P12,600, and other assets of P598,500; liabilities are
P322,560. On this date they decided to go out of business and sell all the assets for
P378,000. Gina has personal assets of P18,900 that may, if necessary, be used to meet
partnership obligations. How much should be distributed to Donna upon liquidation of
the partnership?
a. P25,704 b. P61,236 c. P0 d. P50,400

3. Sy, Ty, Uy, and Vet are partners sharing profits in the ratio of 3/21, 4/21, 6/21, 8/21.
Their capital balances on December 31,2009 are Sy 500, Ty 12,500, Uy 12500 and Vet
4,500.The partners decided to liquidate their firm and they accordingly covert the non
cash assets into 11600 cash. After paying liabilities of 1500, they have 11,100 to divide.

How much is the gain (loss) on realization?


a. (112500 B. 18900 C. (18400) D. (18900)

4. Partner T is personally insolvent, owing $400,000. Personal assets will only bring
$150,000 when liquidated. At the same time, T has a credit capital balance in the
partnership of $85,000. The capital amounts of the other partners total a (credit) balance
of $200,000. Under the doctrine of marshaling of assets, the personal creditors of T can
collect up to __________.

A. $150,000
B. $235,000
C. $400,000
D. $435,000
5. Partners Able, Baker, and Chapman have the following personal assets, personal
liabilities, and partnership capital balances:

Able Baker Chapman

Personal assets 30,000 80,000 60,000

Personal liabilities 25,000 50,000 72,000

Capital balances 50,000 (32,000) 70,000

Assume profits and losses are allocated equally.

After applying the doctrine of marshaling of assets, the capital balances for Able, Baker, and
Chapman, respectively, would be

a. 50,000, (2,000), and 58,000.

b. 48,000, 0, and 58,000.

c. 49,000, 0, and 57,000.

d. 34,000, 0, and 54,000.

6. The following balance sheet is presented for the partnership of D, E and F who share
profits and losses in the ratio of 5:3:2, respectively:
Cash P 60,000 Liabilities P140,000
Other assets 540,000 D, Capital 280,000
E, Capital 160,000
F, Capital 20,000
Total P600,000 Total P600,000
The partners decided to liquidate the partnership. If the other assets are sold for
P400,000, how should the available cash be distributed to each partner, respectively?
(a) P280,000, P160,000, P20,000 (c) P206,000, P114,000, P 0
(b) P210,000, P118,000, P8,000 (d) P205,000, P115,000, P 0
7. A, B, and C partnership began the process of liquidation with the following balance
sheet:
Cash P16,000
Non-cash assets 434,000
Liabilities 150,000
A, capital 80,000 cr.
B, Capital 90,000 cr.
C, Capital 130,000 cr.
A, B, and C share profits and losses in the ratio of 3:2:5. Liquidation expenses are
expected to be 12,000. After the liquidation expenses of P12,000 had been paid and the
non-cash assets sold, C had a deficit of P8,000. For what amount were the non-cash
assets sold?
A. P170,000 B. P264,000 C. P158,000 D. P146,000
8. J, C, P are partners with a profit and loss ratio of 4:3:3. The partnership was liquidated
and, prior to the liquidation process, the partnership balance sheet was as follows:
Cash P60,000 J, Capital P216,000
Other assets 540,000 C, Capital 240,000
P, capital 144,000
After the partnership was liquidated and the cash was distributed, C received P96,000 in
cash in full settlement of his interest. The liquidation loss must have been
A. P360,000 B. P144,000 C. P504,000 D. P480,000
9. A, B and C have capital balances of P90,000,P60,000 and P30,000, respectively. Profits
are allocated to 35% to A, 35% to B, and 30% to C. the partners have decided to dissolve
and liquidate the partnership. After paying all creditors , the amount available for
distribution is P60,000. A, B and C are all personally solvent. Under the circumstances ,
C will
A. Receive P18,000
B. Receive P30,000
C. Personally have to contribute an additional P6,000
D. Personally have to contribute an additional P36,000
10. A local partnership has cash of P5,000 and a building P80,000. All liabilities have been
paid and the partners are all insolvent. The partners’ capital account are as follows H
P40,000, L P30,000 and W P15,000. The partners share profits and losses 4:4:2. If the
building is sold for P50,000, how much cash will H receive in the final settlement?
A. P9,000 B. P15,000 C. P28,000 D. P25,000
Partnership liquidation (installment)

11. Partners A, B, and C have capital balances of P20,000, P50,000 and P 90,000, respectively,
just before liquidation. They split profits in the ratio of 2:4:4, respectively.
Under a safe cash distribution, one of the partners will get the following total amount in
liquidation before any other partners get anything.
A. P0 B. P15,000 C. P40,000 D. P180,000

12. Partners Sasha, Haly and Jane share profits and losses in the ratio of 5:3:2. At the end of a
very unprofitable year, they decided to liquidate the firm. The partners’ capital account
balances at this time are as follows:
Sasha P123,300 Jane P44,000

Haly 139,440

The liabilities accumulate to P168,000, including a loan of P56,000 from Sasha. The cash
balance is P33,600. All the partners are personally solvent. The partners plan to sell the
assets in installment. If Haly received P20,160 from the first distribution of cash, how much
did Sasha receive at that time?

a. P11,200 b. P4,480 c. P6,720 d. P0

13. The Walker, Wilson, and Winston Partnership is being liquidated. All liabilities have been
paid. The balance of assets on hand is being realized gradually. The following are details of
partners’ accounts:
Capital Account Drawing Account Loans to P/L
Balances Balances Partnership Ratio

Walker P200,000 P15,000 Cr. P150,000 5

Wilson 250,000 20,000 Dr. - 2

Winston 100,000 30,000 Cr. 50,000 3

If you are to rank the partners from the most vulnerable to the least vulnerable, the ranking will
be as follows:

a. Walker, Wilson, and Winston, respectively.

c. Winston, Wilson and Walker, respectively

b. Wilson, Walker, and Winston, respectively.

d. Winston, Walker and Wilson, respectively. *

14. Partners A, B and C share profits and losses in the ratio of 5:3:2. At the end of a very
unprofitable year, they decided to liquidate the firm. The partner’s capital account balances at
this time are as follows: A, P616,000; B, P697,200; C, P420,000. The liabilities accumulate
to P840,000, including a loan of P280,000 from A. The cash balance is P168,000. All the
partners are personally solvent. The partners plan to sell the assets in instalment.
If B received P100,800 from the first distribution of cash, how much did C receive at that
time?

a.P56,000
b.P33,600
c.P22,400
d.P61,600
15. Partners A, B, C, and D have been operating ABCD Partnership for ten years. Due to a
significant reduction in the demand for their product over recent years, the partners have
agreed to liquidate the partnership. At the time of liquidation, balance sheet accounts
consisted of cash, P103,500; noncash assets, P300,000; liabilities to outsiders, P60,000;
capital credit balances for partners A, B, and C, P90,000, P150,000, and P120,000,
respectively; and a debit capital balance for partner D of P16,500. Partners share equally in
income and loss. It is estimated that the administrative cost of liquidation will total P4,500.
While preparing for liquidation, an unrecorded liability of P7,500 was discovered. Assuming
the available cash of P103,500 was distributed, how much must be the share of partner B?
a. P31,500 b. P30,750 * c. P65,167 d. none

16. The Walker, Wilson, and Winston Partnership is being liquidated. All liabilities have been
paid. The balance of assets on hand is being realized gradually. The following are details of
partners’ accounts:
Capital Account Drawing Account Loans to P/L
Balances Balances Partnership Ratio

Walker P200,000 P15,000 Cr. P150,000 5

Wilson 250,000 20,000 Dr. - 2

Winston 100,000 30,000 Cr. 50,000 3

If partner Walker receives P150,000, how much partner Wilson receives?

a. P144,000 * b. P51,000 c. P86,000 d. PP129,000

17. Assume that a partnership had assets with a book value of $240,000 and a market value of
$195,000, outside liabilities of $70,000, loans payable to partner Able of $20,000, and capital
balances for partners Able, Baker, and Chapman of $70,000, $30,000, and $50,000. How
would the first $100,000 of available assets be distributed assuming profits and losses are
allocated equally?
a. $70,000 to outside liabilities, $20,000 to Able, and the balance equally among the partners

b. $70,000 to outside liabilities and $30,000 to Able

c. $70,000 to outside liabilities, $25,000 to Able, and $5,000 to Chapman

d. $40,000 to Able, $20,000 to Chapman, and the balance equally among the partners

18. Assume that a partnership had assets with a book value of $240,000 and a market value of
$195,000, outside liabilities of $70,000, loans payable to partner Able of $20,000, and capital
balances for partners Able, Baker, and Chapman of $70,000, $30,000, and $50,000. If all
outside creditors and loans to partners had been paid, how would the balance of the assets be
distributed assuming that Chapman had already received assets with a value of $30,000
assuming profits and losses are allocated equally?
a. Each of the partners would receive $25,000.

b. Each of the partners would receive $40,000.

c. Able: $70,000, Baker: $30,000, Chapman: $20,000

d. Able: $55,000, Baker: $15,000, Chapman: $5,000

19. ALP partnership of Alan, Lemuel, and Paul is beginning liquidation. It has no cash, total
liabilities of P60,000 including a P10,000 loan payable to Lemuel, and equal partners’ capital
balances of P40,000. The income-sharing ratio is 5:1:4, respectively. If a portion of the non-
cash assets with a carrying amount of P140,000 realizes P120,000, the cash payment that
Lemuel receives
A. P20,000 B. P44,000 C. P53,000 D. some other amount
20. J, S and H, partners, share net income and losses in the ratio 5:3:2. The partners decided to
liquidate the partnership. Their balance sheet prior to liquidation is:
Cash P 40,000 Liabilities P 60,000
Other assets 210,000 J, Loan 8,000
J, Capital 40,000
S, Capital 72,000
H, Capital 70,000
Total P250,000 Total P250,000
The partnership is to be liquidated by installment. The first sale of non-cash assets with a
carrying amount of P120,000 realized P90,000. Liquidation expenses paid amounted to
P2,000. How much cash should be distributed to each partner, respectively?
(a) P 0, P35,400, P45,600 (c) P 0, P 9,600, P28,400
(b) P32,000, P62,400, P63,600 (d) P 0, P27,600, P40,400

Corporate liquidation

21. The following amounts were taken from the statement of affairs for Bagsak Company:
Unsecured liabilities without priority P90,000
Stockholders' equity 36,000
Loss on realization of assets 45,000

Estimated administrative expenses that have not been


entered in the accounting records 4,500
Unsecured liabilities with priority 10,000

The estimated payment for the unsecured liabilities without priority will be

A. P76,500 C. P81,000
B. P77,850 D. P90,000

22. The following amounts were taken from the statement of affairs for Bagsak Company

Unsecured liabilities without priority P90,000


Stockholders' equity 36,000
Loss on realization of assets 45,000

Estimated administrative expenses that have not been


entered in the accounting records 4,500
Unsecured liabilities with priority 10,000
How much is the net realizable value of the total assets?
A. P36,000 B. P 136,00 C. P 91,000 D. P100,000
23. following were taken from the statement of affairs of Nodebt Company
Assets pledged with fully secured creditors P71,000
Assets pledged with partially secured creditors 12,500
Free assets 11,000
Preferred creditors 3,000
Fully secured creditors 69,000
Partially secured creditors 20,000
Unsecured creditors without priority 18,000
The estimated deficiency to unsecured creditors is
A. P5,000 B. P12,500 C. P15,500 D. P14,500

24. The following data were taken from the statement of affairs of Hopeless Company
Assets pledged with fully secured creditors P 218,750
Assets pledged with partially secured creditors 157,500
Free assets 152,250
Fully secured liabilities 175,000
Partially secured liabilities 192,500
Unsecured liabilities without priority 157,500
Unsecured liabilities with priority 22,750
Which of the following estimated payments to types of liabilities is correct?
A. Fully secured liabilities P218,750
B. Partially secured liabilities 192,500
C. Unsecured liabilities with priority 20,475
D. Unsecured liabilities without priority 141,750
25. A review of the assets and liabilities of G Company in bankruptcy on June 30,
2008,discloses the following:
a. A mortgage payable of P118,000, is secured by building valued at P39,000 less
than its book value of P172,000.
b. Notes payable of P57,000 is secured by furniture and equipment with a book
value of P76,000 that is 3/5 realizable.
c. Assets other than those referred to have an estimated value of P44,000, an amount
that is 75% of its book value
d. Liabilities other than those referred to total P91,000, which included claims with
priority of P23,000.
How much was paid to the partially secured creditors?
A. P52,340 B. P48,260 C. P49,380 D. P50,769

26. The following data are provided by the Troubled Company:


Assets at the book value P150,000

Assets at net realizable value 105,000

Liabilities at book value:

Fully secured mortgage 60,000

Unsecured accounts and notes payable 70,000

Unrecorded liabilities:

Interest on bank notes 500

Estimated cost of administering estates 6,000

The court has appointed a trustee to liquidate the company:

.The journal entry made by the trustee to record the assets and liabilities should include an estate
deficit of:

a.P31,500 c. P25,500

b. 31,000 d. 25,000

27. Using the same information above, the statement of affairs prepared by the trustee at this
time should include an estimated deficiency to unsecured creditors of:
a. P45,000 c.P31,500

b. P 39,000 d.P 25,000

28. The accountant of Holy company under liquidated provided the following data:
Assets at book value P100,000

Assets at net realizable value 75,000

Liabilities at book value:

Fully secured mortgage payable 40,000

Unsecured accounts and notes payable 45,000

Unrecorded Liabilities:

Interest on bank notes 250

Administrative Expenses 4,000

A trustee is appointed to liquidate the company.

Using the data above, what is the estimated deficiency to unsecured creditors?

a. P35,000
b. P31,000
c. P14,250
d. P10,000
29. Lakeside Bank holds a 100,000 note secured by a building owned by Fly-By-Night
Manufacturing, which has filed for bankruptcy. If the property has a book value of 120,000
and a fair market value of 90,000, what is the best way to describe the note held by Second
City Bank? The bank has a(n)
A. secured claim of 100,000.

B.unsecured claim of 100,000.

C. secured claim of 90,000 and an unsecured claim of 10,000.

D. secured claim of 100,000 and an unsecured claim of 20,000.


Prepared by:

Dejarme, Johannah Rose M.

Polancos, Jean Claire P

.
Mindanao State University
General Santos City

ADVANCE ACCOUNTING1
MCQ‘s PROBLEMS AND THEORIES

Source : Mr Antonio Jaramillo Dayag, CPA, MBA


Prepared By: Ms. Angel D.
Pates
and Angelli
Lamique

Chapter 1
Partnership Formation
THEORIES
1. The partnership form of business is:
a. An economic entity.
b. A separate legal entity, just as a corporation is a legal entity.
c. A taxable entity.
d. A fiscal entity.
2. A distinct and major advantage of the professional corporation form of organization in
comparison with the partnership form of organization is:
a. Limited liability with respect to damages arising from professional services.
b. Greater allowable tax deductions for retirement plans.
c. Ease of formation
d. Book value
e. Historical cost
3. Which of the following is not a characteristic of a partnership?
a. The partnership itself pays no income taxes.
b. It is easy to form a partnership.
c. Any partner can be held personally liable for all debts of the business.
d. A partnership requires written Articles of Partnership.
e. Each partner has the power to obligate the partnership for liabilities.
4. The advantages of the partnership form of business organization, compared to
corporations, include
a. Single taxation
b. Ease of raising capital
c. Mutual Agency
d. Limited Liability
e. Difficulty of formation

5. Which of the following is NOT a characteristic of the proprietary theory that influences
accounting for partnerships?
a. Partners’ salaries are viewed as a distribution of income rather than a
component of net income.
b. A partnership is not viewed as a separate, distinct, taxable entity.
c. A partnership is characterized by limited liability.
d. Changes in the ownership structure of a partnership result in the dissolution of
the partnership.

PROBLEMS
6. Albert, Claude, and Jamie form a partnership by contributing P25,000, P70,000, and
P80,000, respectively. In addition, the partners agree that Albert should receive P20,000
of goodwill because of his special skills relevant to this business. What amount of capital
will exist for Claude when the partnership is formed?
a. P60,000
b. P65,000
c. P70,000
d. Some other amount
7. Bill and Ken enter into a partnership agreement in which Bill is to have a 60% interest in
capital and profits and Ken is to have a 40% interest in capital and profits. Bill
contributes the following:

Cost Fair value


Land P10,000 P20,000
Building P100,000 P60,000
Equipment P20,000 P15,000
There is a P30,000 mortgage on the building that the partnership agrees to assume. Ken
contributes P50,000 cash to the partnership. Bill and Ken agree that Ken’s capital
account should equal Ken’s P50,000 cash contribution and that goodwill (revaluation of
asset) should be recorded. Goodwill (revaluation of asset) should be recorded in the
amount of:
a. P10,000
b. P15,000
c. P16,667
d. P20,000
8. Paul, Jeremy, and Juan are forming a partnership. Juan contributes a building having an
historical cost, accumulated depreciation, and market value of P290,000, P100,000, and
P400,000, respectively. The building is initially recorded on the partnership’s books at
Juan’s book value (P190,000). Two years later the building is sold for a P270,000 gain.
What portion of the profit or loss should be allocated to Juan?
a. P20,000
b. P90,000
c. P210,000
d. P230,000

9. Jones and Smith formed a partnership with each partner contributing the following
items:

Jones Smith
Cash P80,000 P40,000
Building-Cost to 300,000
Jones
-Fair Value 400,000
Inventory- Cost 200,000
Smith
-Fair Value 280,000
Mortgage Payable 120,000
Account Payable 60,000

Assume that for tax purposes Jones and Smith agree to share equally in the liabilities
assumed by the Jones and Smith partnership. What is the balance in each partner’s
capital account for financial accounting purposes?
a. Jones- P350,000 and Smith- P270,000
b. Jones- P260,000 and Smith- P180,000
c. Jones- P360,000 and Smith- P260,000
d. Jones- P500,000 and Smith- P300,000
10. On July 1, ML and PP formed a partnership, agreeing to share profits and losses in the
ratio of 4:6, respectively. ML contributed a parcel of land that cost her P25,000. PP
contributed P50,000 cash. The land was sold for P50,000 on July 1, four hours after
formation of the partnership. How much should be recorded in ML’s capital account on
the partnership formation?
a. P10,000
b. P20,000
c. P25,000
d. P50,000

CHAPTER 2
Partnership Operation
THEORIES
1. Partnership drawings are
a. Always maintained in a separate account from the partner’s capital account.
b. Equal to partners’ salaries.
c. Usually maintained in a separate draw account with any excess draws being
debited directly to the capital account.
d. Not discussed in the specific contract provisions of the partnership.

2. Which of the following would be least likely to be used as a means of allocating profits
among partners who are active in the management of the partnership?
a. Salaries
b. Bonus as a percentage of net income before the bonus
c. Bonus as a percentage of sales in excess of a targeted amount
d. Interest on average capital balances
3. A partnership agreement calls for allocation of profits and losses by salary allocations, a
bonus allocation, interest on capital, with any remainder to be allocated by preset
ratios. If a partnership has a loss to allocate, generally which of the following procedures
would be applied?
a. Any loss would be allocated equally to all partners.
b. Any salary allocation criteria would not be used.
c. The bonus criteria would not be used.
d. The loss would be allocated using the profit and loss ratios, only.
4. Which of the following statement is true concerning the treatment of salaries in
partnership accounting?
a. Partner salaries may be used to allocate profits and losses; they are not
considered expenses of the partnership
b. Partner salaries are equal to the annual partner draw.
c. The salary of a partner is treated in the same manner as salaries of corporate
employees.
d. Partner salaries are directly closed to the capital account.
5. Which of the following could be used as a basis to allocate profits among partners who
are active in the management of the partnership?
1) Allocation of salaries
2) The number of years with the partnership
3) The amount of time each partner works
4) The average capital invested
a. 1 and 2
b. 1 and 3
c. 1, 2 and 3
d. 1, 3 and 4
e. 1, 2, 3, and 4

PROBLEMS
6. Partners A and B have a profit and loss agreement with the following provisions: salaries
of P30,000 and P45,000 for A and B, respectively; a bonus to A of 12% of net income
after salaries and bonus; and interest of 10% on average capital balances of P50,000 and
P65,000 for A and B, respectively. One-fourth of any remaining profits are allocated to A
and the balance to B. If the partnership had net income of P108,600. How much should
be allocated to Partner A?
a. P43,225
b. P43,816
c. P47,850
d. P65,375
7. X, Y, and Z, a partnership formed on January 1, 20x4 had the following initial
investment:

X P100,000
Y 150,000
Z 225,000
The partnership agreement states that the profits and losses are to be shared equally by
the partners after consideration is made for the following:
 Salaries allowed to partners: P60,000 for X, P48,000 for Y, and P36,000 for Z.
 Average partners’ capital balances during the year shall be allowed 10%.

Additional information:

 On June 30, 20x4, X invested an additional P60,000.


 ZZ withdrew P70,000 from the partnership on September 30, 20x4.
 Share the remaining partnership profit was P5,000 for each partner.

Partnership net profit on December 31, 20x4 before salaries, interests and partner’s
share on the remainder was:

a. P199,750
b. P207,750
c. P211,625
d. P222,750

8. A partnership begins its first year of operations with the following capital balances:

Winston, Capital P110,000


Durham, Capital 80,000
Salem, Capital 110,000

According to the articles of partnership, all profits will be assigned as follows:


 Winston will be awarded an annual salary of P20,000 with P10,000 assigned to
Salem.
 The partners will be attributed interest equal to 10 percent of the capital balance
as of the first day of the year.
 The remainder will be assigned on a 5:2:3 basis, respectively.
 Each partner is allowed to withdraw up to P10,000 per year.

Assume that the net loss for the first year of operations is P20,000 and that net
income for the subsequent year is P40,000. Assume also that each partner
withdraws the maximum amount from the business each period. What is the
balance in Winston’s capital account at the end of the second year?

a. P102,600
b. P104,400
c. P108,600
d. P109,200
9. Maxwell is trying to decide whether to accept a salary of P60,000 or a salary of P25,000
plus a bonus of 20% of net income after the bonus as a means of allocating profit among
the partners. What amount of income would be necessary so that Maxwell would
consider the choices to be equal?
a. P35,000
b. P85,000
c. P140,000
d. P210,000

10. Garcia and Henson formed a partnership on January 2, 20x4, and agreed to share profits
90%, 10%, respectively. Garcia contributed capital of P25,000. Henson contributed no
capital withdrawals during the year. The partnership agreement provides for the
following:
Capital accounts are to be credited annually with interest at 5% of beginning capital.
 Henson is to be paid salary of P1,000 a month.
 Henson is to receive a bonus of 20% of income calculated before deducting his
salary and interest on both capital accounts.
The partnership 20x4 income statement follows:

Revenues P96,450
Expenses(including salary, interest, and bonus) 49,700
Net Income P46,750

CHAPTER 3
Partnership Dissolution
THEORIES
1. Transferable interest of a partner includes all of the following except:
a. The partner’s share of the profits and losses of the partnership
b. The right to receive distributions
c. The right to receive any liquidating distribution
d. The authority to transact any of the partnership’s business operations
2. Which of the following must occur for a new partner to enter the partnership by acquiring an
ownership interest directly from an existing partner?
a. Existing partners must know the amount the new partner is paying for the ownership
interest
b. The new partner must acquire all of the current partner’s ownership interest
c. Existing partners must approve the admission of the new partner into the partnership
d. The new partner must live in the same state as the other partners
3. Which of the following is not a criterion for recognizing a bonus to existing partners when a new
partner joins the partnership?
a. Only cash assets were contributed to the partnership by the new partner
b. The existing partners desire to not recognize goodwill on the balance sheet
c. The articles of partnership indicate that the bonus method will be used to admit new
partners
d. The new partner invests more into the partnership that his/her share of total
partnership capital after the investment is made
4. Which of the following statements is false with regard to the goodwill recognized for a new
partner entering a partnership?
a. The new partner’s capital account balance will exceed the amount invested
b. The existing partners’ capital accounts will remain unchanged
c. The amount invested by the new partner will be less than his/her proportion of the
partnership’s book value before goodwill is recognized
d. The three partners will have equal capital account balances when the transaction is
completed
5. When a new partner is admitted into a partnership and the old partners’ goodwill is recognized,
the goodwill is allocated to:
I. All the partners in their profit-and-loss sharing ratio.
II. The old partners in their profit and loss sharing ratio.
a. I only
b. II only
c. Either I or II
d. Neither I nor II

PROBLEMS
6. William desires to purchase a one-fourth capital and profit and loss interest in the partnership of
Eli, George, and Dick. The three partners agree to sell William one-fourth of their respective
capital and profit and loss interests in exchange for a total payment of P40,000. The capital
accounts and the respective percentage interests in profits and losses immediately before the
sale to William are as follows:

Eli Capital 60% P80,000


George Capital 30% 40,000
Dick Capital 10% 20,000
P140,000
All other assets and liabilities are fairly valued, and implied goodwill is to be recorded prior to
the acquisition by William. Immediately after William’s acquisition, what should be the capital
balances of Eli, George, and Dick, respectively?
a. P60,000, P30,000, P15,000
b. P69,000, P34,500, P16,500
c. P77,000, P38,500, P19,500
d. P92,000, P46,000, P22,000
7. A partnership has the following capital balances:

Partners Capital Balance


William (40% of gains and losses) P220,000
Jennings (40%) 160,000
Bryan (20%) 110,000

Darrow invests P270,000 in cash for a 30% ownership interest. The money goes to the original
partners. Goodwill is to be recorded. How much goodwill should be recognized, and what is
Darrow’s beginning capital balance?
a. P410,000 and P270,000
b. P140,000 and P270,000
c. P140,000 and P189,000
d. P410,000 and P189,000
8. A partnership has the following capital balances:

Partners Capital Balance


Allen P60,000
Burns 30,000
Castello 90,000

Profits and losses are split as follows: Allen (20%), Burns (30%), and Costello (50%). Costello
wants to leave the partnership and is paid P100,000 from the business based on provisions in
the articles of partnership. If the partnership uses the bonus method, what is the balance of
Burn’s capital account after Costello withdraws?
a. P24,000
b. P27,000
c. P33,000
d. P36,000
9. Assuming that a partnership currently consists of two partners, Adams and Brown, with
respective capital interests of P60,000 and P40,000. Adams and Brown share income and losses
in the ratio of 6:4. Both partners agree to the admission of a new partner. Assume that Call was
admitted as a new partner by acquiring 30% capital interest and profit and loss ratio also at 30%
in the partnership by paying the partners P36,000. Given the choice between book value
approach (or bonus method) and revaluation of assets (or goodwill method), Call will:
a. Prefer book value (or bonus) method due to Call’s gain of P6,000.
b. Prefer book value (or bonus) method due to Call’s gain of P12,000.
c. Prefer revaluation (or goodwill) method due to Call’s gain of P6,000.
d. Be indifferent for the revaluation (goodwill) and the book value (or bonus) methods are
the same.
10. Partners Art and Tony, who share equally in profits and losses, have the following balance sheet
as of December 31, 20x4:

Cash P120,000 Accounts Payable P172,000


Receivable 100,000 Accumulated Depreciation 8,000
Inventory 140,000 Art, capital 140,000
Equipment 80,000 Tony, capital 120,000
Total P440,000 Total P440,000

They agreed to incorporate their partnership, with the new corporation absorbing the net assets
after the following adjustments: provision of allowances for bad debts of P10,000; statement of
the inventory as its current fair value of P160,000; and, recognition of further depreciation on
the equipment of P3,000. The corporation’s capital stock is to have a par value of P100, and the
partners are to be issued corresponding total shares equivalent to their adjusted capital
balances. The total par value of the shares of capital stock that were issued to partners Art and
Tony was:
a. P260,000
b. P267,000
c. P273,000
d. P280,000

Chapter 4
Partnership Lump-sum Liquidation
THEORIES
1. If cash payment to partners of a limited partnership in liquidation are delayed until all
non cash assets have been realized, any cash remaining after all partnership creditors
have been paid is distributed :
a. According to the liquidator’s best judgment
b. In the ratio for sharing the net income and losses
c. In the amount equal to the partners’ loan and capital balances
d. In some other answer
2. Which of the following statements is correct regarding a partner’s debit capital
balances?
a. The partners should make contribution to reduce the debit balance too
whatever extent possible.
b. If contribution are not possible, the other partners with credit capital balances
will be allocated of a portion of the debit balance based on their proportionate
profit-and-loss-sharing ratio
c. Partners who absorb another’s debit capital balance have a legal claim against
the deficit partner.
d. All of these statements are correct.
3. If condition produce a debit balance in a partner’s capital account when liquidation
losses are allocated
a. The partner receives further allocation of liquidation losses, but not gains.
b. The partner receives no further allocation of liquidation losses and gains.
c. The partner is no longer obligated to partnership creditors.
d. The partner has an obligation of personal net assets to the other partners.
4. Which of the following procedures is acceptable when accounting for a deficit balance in
a partner’s capital account during partnership liquidation?
a. A partner with a negative capital balance must contribute personal assets to the
partnerships that are sufficient to bring the capital account to zero.
b. If a partner with a negative capital balance is personally insolvent, the negative
capital balance may be absorb by those partners having a positive capital
balance according to the residual profit and loss sharing ratios that applies to all
partners.
c. If a partner with a negative capital balance is personally insolvent, the negative
capital balance may be absorb by those partners having a positive capital
balance according to the residual profit and loss sharing ratios apply to those
having positive balances.
d. All of the above procedures are acceptable.
5. In partnership liquidation, how are partner salaries are allocated?
a. Salary allocations take precedence over creditor payments
b. Salary allocations take precedence over amount due to partners with respect to
their capital interests, but not profits.
c. Salary allocations take precedence over amounts due to partners with respect to
their capital profits, but not capital interest
d. Salary allocations are disregarded

PROBLEMS
6. Larry, Marsha and Natalie are partners in a company that is being liquidated. They share
profit and loss 55%, 20% and 25%, respectively. When the liquidation begins they have
capital account balances of P108,000, P62,000, and P56,000, respectively. The
partnership just sold equipment with a historical cost and accumulate depreciation of
P25,000 and P18,000, respectively for P10,000. What is the balance in Marsha’s capital
account after the transaction is completed?
a. 62,000
b. 61,400
c. 62,600
d. 65,000
7. As of December 31, 2015, the books of Ton Partnership showed capital account
balances of: T P40,000; O P25,000; N P5,000. The partnership profit and loss ratio was
3:2:1, respectively. The partners decided to liquidate and they sold all their noncash
asset for P37,000. After settlement of all liabilities amounting P12,000, they still have
cash of P28,000 left for distribution. Assuming that any capital debit balance is
uncollectible, the share of T in the distribution of the P28,000 cash would be:
a. 17,800
b. 18,000
c. 19,000
d. 17,000
8. After all noncash asset have been converted into cash in the liquidation of AA and JJ
partnership, the ledger contains the following account balances:

Debit Credit
Cash ………………………………………………………………………… 34,000
Accounts Payable……………………………………………………. 25,000
Loan Payable to AA…………………………………………………. 9,000
AA, Capital………………………………………………………………. 8,000
JJ, Capital…………………………………………………………………. 8,000

Available cash should be distributed: P25,000 to accounts payable and;

a. P9,000 loan payable to AA


b. P4,500 each to AA and JJ
c. P1,000 to AA and P8,000 to JJ
d. P8,000 to AA and P1,000 to JJ

9. Jar, Ram, and Millo, who divides profit and losses 50%, 20%, and 30%, respectively, have
the following October 31, 2015 account balances:

Jar, Drawing (Dr)………………………………………………….. P12,000


Millo, Drawing (Cr)……………………………………………… 4,800
Accounts Receivable—Jar………………………………….. 7,200
Loans Payable –Ram……………………………………………. 14,400
Jam, Capital…………………………………………………………. 59,400
Ram, Capital…………………………………………………………. 44,400
Millo, capital………………………………………………………… 39,000

The partnership’s assets are P211,200 ( including cash of P64,200). The partnership is
liquidated and Millo receives P33,000 in final settlement. How much is the total loss on
realization?

a. P10,800
b. 31,200
c. 54,000
d. 64,200
10. After operating for five years, the books of the partnership of Bo and By showed the
following balances:

Net assets………………………………………………………… P169,000


Bo, capital………………………………………………………… 110,500
By, capital…………………………………………………………. 58,500

If liquidation takes place at this point and the net assets are realized at book value the
partners are entitled to:

a. Bo to receive P115,000 and By t received P52,000


b. Bo to receive P126,750 and By to receive P42,250
c. Bo to receive P84,500 and By to receive P84,500
d. Bo to receive P110,500 and By to receive P58,500

Chapter 5
Partnership Installment Liquidation
THEORIES
1. Which of the following is correct with regard to partnership liquidation?
a. All creditors must be paid in full before the distributions can be made to
partners.
b. Partner capital contributions and undistributed income are viewed as distinct in
the Uniform Partnership Act.
c. All creditors are equal with regard to priority of claim against partnership assets.
d. Loans from partners to partnership have the same priority claim against
partnership assets as to creditor claims from the other assets.
2. In accounting for partnership liquidation, cash payment to partners after all creditor’s
claim have been satisfied, but before the final cash distribution, should be according to:
a. The partners’ relative profit and loss sharing ratios.
b. The final balances in partner capital accounts.
c. The partners’ relative share of the gain or loss on liquidation.
d. Safe payment s computation
3. In a partnership liquidation, the final cash distribution to the partners should be made in
accordance with the:
a. Partners’ profit and sharing ratio
b. Balances of the partners’ capital accounts.
c. Ratio of the capital contributions by the partners
d. Ratio of capital contributions less withdrawals by the partners.

4. The partnership of Angel, Leigh and Rose was insolvent and will be unable to pay
P30,000 in liabilities currently due. What recourse was available to the partnership’s
creditors?
a. They must present an equal claims to the three partners as individual
b. They must try to obtain a payment from the partner with the largest capital
account balances.
c. They cannot seek remunerations from the partners as individuals.
d. They may seek remuneration from any of the partner they choose.
e. They must present their claims to the three partners in the order of the partner’s
capital account balances.
5. In partnership liquidations, what are safe payments?
a. The amounts of distribution that can be made to the partners, after all creditors
have been paid in full.
b. The amounts of distributions that can be made to the partners with assurance
that such amount will not have to be returned to the partnership.
c. The amount of distribution that can be made to the partners, after all noncash
assets have been adjusted to fair market value.
d. All of the above are examples of the safe payments concept.

PROBLEMS
6. When Mikki and Mylene, partners who share earnings equally, were incapacitated in an
airplane accident, a liquidator was appointed to wind up their business. The accounts
showed cash, P35,000; other assets, P110,000; Liabilities, P20,000; Mikki, Capital,
P71,000; Mylene, Capital, P54,000. Because of highly specialized nature of the noncash
assets, the liquidator anticipated that considerable time would be required to dispose
them. The expenses of liquidating the business( advertising , rent, travel , etc) are
estimated at P10,000.

How much cash can be distributed safely to each partner at this point?

a. P5,000 to Mikki; and P0 to Mylene


b. P5,000 to Mikki; andP500 to Mylene
c. P3,000 to Mikki; and P0 to Mylene
d. P5,000 to Mikki; and P1,000 to Mylene
7. PP, QQ , and RR, partners to a firm, have capital balances of P11,200, P13,000, and
P5,800, respectively, and share profits in the ratio of 4:2:1. Prepare a schedule showing
how available cash will be given to the partners as it becomes available. Who among the
partners shall be paid first with an available cash of P1,400?
a. QQ
b. No one
c. RR
d. PP
8. Scott, Joe, and Ed are liquidating their partnership. At the date liquidation begins Scott,
Joe and Ed have capital account balances of P162,000, P192,500, and P215,000,
respectively and the partners share profits and losses 40%, 35%, and 25%, respectively.
In addition, the partnership has a P36,000 Notes Payable to Scott and a P20,000 Notes
Receivable from Ed. When the liquidation begins, what is the loss absorption power with
respect to Joe?
a. P192,500
b. 67,375
c. P550,000
d. 770,000
9. The balance sheet of the Queen, Reed, and Stac Partnership at the end of its fiscal year
on October 31, 2015 are as follows:

ASSETS LIABILITIES AND EQUITY


P P50,00
Cash 15,000 Liabilities 0
Receivables -net 20,000 Loan from Stac 10,000
Inventory 40,000 Queen, Capital-30% 45,000
Plant Assets- net 70,000 Reed, capital- 50% 30,000
Loadn Reed 5,000 Stac , capital- 20% 15,000
150,00 150,00
Total Assets 0 Total Liabilities and Equity 0

The partners decide to liquidate the partnership. They estimate that the noncash assets,
other than loan to Reed, can be converted into P100,000 cash over the two-months
period ending December 31, 2015. Cash is to be distributed to the appropriate parties as
it becomes available during the liquidation process.

The partner most vulnerable to partnership losses on liquidation is:

a. Queen
b. Reed
c. Reed and Queen equally
d. Stac
10. Using the same information in No. 9, and P65,000 is available for first distribution, it
should be paid to:

Priority
Creditor
s Queen Reed Stac
P
a. P60,000 P5,000 P 0 0
b. 60,000 1,500 2,500 1,000
c. 50,000 5,000 0 10,000
d. 50,000 12,000 0 3,000

CHAPTER 6
Corporate Liquidation

THEORIES

1. The duties of the trustee include:


a. Appointing creditor’s committees in liquidation cases.
b. Approving all payments for debts incurred before the bankruptcy filing.
c. Examining claims and disallowing any that are improper
d. Calling a meeting of the debtor’s creditor.
2. When the business becomes insolvent, it generally has three possible courses of action.
Which of the following is not one of the three possible courses of action?
a. The debtors and its creditors may enter into a contractual agreement, outside of
formal bankruptcy proceedings
b. The debtors continue operating the business in the normal course of the day-to-
day operations.
c. The debtor or its creditors may file a bankruptcy petition, after which the debtor
is liquidated.
d. The debtor or its creditors may file a petition for reorganization.
3. Which of the statement is true?
a. Certain debts are not dischargeable
b. The goal of liquidation is to give the company a new start
c. All secured claims are paid in full
d. The expenses to administer the estate are paid last because they are unsecured.
4. On a balance sheet prepared for a company during its reorganization, at what balance
are liabilities reported?
a. At the expected amount of the allowed claims
b. At the present value of the expected future cash flows
c. At the expected amount of the settlement
d. At the amount of the anticipated final payment
5. An order for relief
a. Prohibits creditors from taking action to collect from an insolvent company
without court approval.
b. Calls for the immediate distribution of free assets to unsecured creditors.
c. Can be entered only in an involuntary bankruptcy proceedings
d. Gives an insolvent company time to file a voluntary bankruptcy petition.

PROBLEMS

Items 6-10 are based on the following data:

The Pukaki Company has decided to seek liquidation after previous restructuring and
quasi-reorganization attempts failed. The company has the following condensed balance sheet
as of May 1, 2015:
ASSET LIABILITIES AND STOCKHOLDERS' EQUITY
Cash P12,000 Accrued Payroll P40,000
Accounts Receivable (net) 280,000 Loans from Officer 50,000
Inventory 70,000 Accounts Payable 60,000
Prepaid expense 1,000 Equipment loan Payable 360,000
Plant Assets 300,000 Business loan payable 180,000
Goodwil
l 39,000 Common Stock 60,000
Deficit -48,000
TOTAL 702,000 TOTAL 702,000

The equipment loan payable is secured by specific plant asset having a book value of
P300,000 and a realizable value of P350,000. Of the accounts payable, P40,000 is secured by
inventory which has a cost of P40,000 and a liquidation value of P44,000. The balance of the
inventory has a realizable value of P32,000. Accounts Receivable with a book value and market
value of P100,000 and P80,000 respectively have been pledged as collateral on the business
loan payable. The balance of the accounts receivables have a realizable value of P150,000.

6. Assuming trustee expenses of P12,000 in addition to recorded liabilities , which of the


remaining unsecured creditors has the next highest order of priority?
a. Accrued payroll
b. Equipment loan payable
c. Loan from officer
d. Business loan payable
7. The realizable value of asset pledged with fully secured creditors is:
a. P459,000
b. 44,000
c. 40,000
d. 489,000
8. Of those creditors who are partially secured, their unsecured amounts are:
a. P430,000
b. 110,000
c. 540,000
d. 120,000
9. Estimated loss on asset disposition is:
a. P51,000
b. 89,000
c. 51,000
d. 90,000
10. Estimated amount paid to partially secured creditor is:
a. P70,000
b. 61,600
c. 20,000
d. 50,000

Mindanao State University


General Santos City

ANSWERS
Source : Mr Antonio Jaramillo Dayag, CPA, MBA
Prepared By: Ms. Angel D.
Pates
and Angelli
Lamique

CHAPTER 1: PARTNERSHIP FORMATION

THEORY

1.1 A partnership is formed by two individuals who were previously sole proprietors.
Property other than cash which is part of the initial investment in the partnership would
be recorded for financial accounting purposes at the:
a. Proprietors’ book values or the fair value of the property at the date of the investment,
whichever is higher

b. Proprietors’ book values or the fair value of the property at the date of the investment,
whichever is lower.

c. Proprietors’ book values of the property at the date of the investment.

d. Fair value of the property at the date of the investment.

1.2. When property other than cash is invested in a partnership, at what amount should
the non-cash property be credited to the contributing partner’s capital account?

a. Contributing partner’s tax basis.

b. Contributing partner’s original cost.

c. Assessed valuation for property tax purposes.

d. Fair value at the date of contribution.

1.3. An advantage of the partnership as a form of business organization would be

a. Partners do not pay income taxes on their share in partnership income.


b. A partnership is bound by the act of the partners
c. A partnership is created by mere agreements of the partners
d. A partnership may be terminated by the death or withdrawal of a partner.

1.4. When property other than cash i invested in a partnership, at what amount should
the noncash property be credited to the contributing partner’s capital account?

a. Fair value at the date of contribution


b. Contributing partner’s original cost
c. Assessed valuation for property tax purposes
d. Contributing partner’s tax basis

1.5. Partnership capital and drawings are similar to the corporate

a. Paid in capital, retained earnings and dividends accounts


b. Retained earnings account
c. Paid in capital and retained earnings accounts
d. Preferred and common stock accounts

PROBLEMS
1.6. Abena and Buendia establish a partnership to operate a used-furniture business
under the name of A and B Furniture. Abena contributes furniture that cost P60,000 and
has a fair value of P90,000. Buendia contributes P30,000 cash and delivery equipment
that cost P40,000 and has a fair value of P30,000. The partners agree to share profits
and losses 60% to Abena and 40% to Buendia.

The peso amount of gain (loss) that will result if the initial noncash contributions of the
partners are recorded at cost rather than fair market value will be

a. P30,000 and (P10,000) to Abena and Buendia, respectively


b. P12,000 and P8,000 to Abena and Buendia, respectively
c. (P18,000) and P18,000 to Abena and Buendia, respectively
d. P 18,000 and (P18,000) to Abena and Buendia, respectively

1.7. On April 30, 2003, Bautista, Jimenez and Laxamana formed a partnership by
combining their separate business proprietorships. Bautista contributed cash of
P100,000. Jimenez contributed property with a carrying amount of P72,000, original
cost of P80,000, and fair value of P160,000. The partnership accepted responsibility
for the P70,000 mortgage attached to the property. Laxamana contributed equipment
with a carrying amount of P60,000, original cost of P150,000, and fair value of 110,000.
The partnership agreement specifies that profits and losses are to be shared equally but
is silent regarding capital contributions.

Which partner has the largest capital account balance as of April 30, 2003?

a. Bautista c. Laxamana
b. Jimenez d. All capital account balances are equal

1.8. G. Macalino and W. Nolasco form a partnership and agree to divide initial capital
equally, even though Macalino contributed P100,000 and Nolasco gave P84,000 in
identifiable assets.

Under the bonus approach to adjust capital accounts, Nolasco’s unidentifiable assets
should be debited for

a . P8,000 c. P-0-

b . P16,000 d . P46,000

1.9. L. Molina and R. Nepomuceno enter into a partnership agreement in which Molina
is to have a 60% interest in capital and profits and Nepomuceno is to have a 40%
interest in capital and profits. Molina contributes the following:

Cost Fair Value

Land P 20,000 P 40,000

Building 200,000 120,000


Equipment 40,000 30,000

There is a P60,000 mortgage on the building that the partnership agrees to assume.
Nepomuceno contributes P100,000 cash to the partnership. Molina and Nepomuceno
agree that Nepomuceno’s capital account should equal Nepomuceno’s P100,000 cash
contribution and that goodwill should be recorded.

Goodwill should be recorded in the amount of

a. P20,000 c. P33,333

b. P30,000 d. P40,000

1.10. On March 1, 2003, Z Roxas and B. Poe decided to combine their business and
form a partnership. The balance sheet of Roxas and Poe on March 1, before adjustment
is presented below.

Roxas Poe

Cash P 9,000 P 3,750

Accounts Receivable 18,500 13,500

Inventories 30,000 19,500

Furniture and fixtures (net) 30,000 9,000

Office Equipment (net) 11,500 2,750

Prepaid Expenses 6,375 3,000

P105,375 P51,500

Accounts Payable P 45,750 P 18,000

Z. Roxas, Capital 59,625

B. Poe, Capital 33,500

P105,375 P 51,500

They agreed to provide 3% for doubtful accounts on their accounts receivable and
found Poe’s furniture to be under depreciated by P900.

If each partner’s share in equity is to be equal to the net assets invested, the capital
accounts of Roxas and Poe would be:
a. P58,170 and P33,095 respectively

b. P58,320 and P32,945 respectively

c. P59,070 and P32,195 respectively


d. P104,820 and P50,195 respectively

CHAPTER 2: PARTNERSHIP OPERATIONS

THEORY

2.1. Las Vegas retired from the partnership of Las Vegas, New York, and New Jersey.
Las Vegas’s cash settlement from the partnership was based on new goodwill
determined at the date of retirement plus the carrying amount of the other net assets.
As a consequence of the settlement, the capital accounts of New York and New Jersey
were decreased. In accounting for Las Vegas’s withdrawal, the partnership could have
used the

Bonus Method Goodwill Method

a. No Yes

b. No No

c. Yes Yes

d. Yes No

2.2. Partners Lovelle and Carlo share income and loss equally after each has been
credited in all circumstances with annual salary allowances of P15,000 and P12,000,
respectively. Under this arrangement, Lovelle will benefit by P3,000 more than Carlo in
which of the following circumstances?

a. Only is the partnership has earnings of P27,000 or more for the year.

b. Only if the partnership does not incur a loss for the year.

c. In all earnings or loss situations.

d. Only if the partnership has earnings of at least P3,000 for the year.

2.3. Partners Cora and Zon share income in a 2:1 ratio, respectively. Each partner
receives an annual salary allowance of P6,000. If the salaries are recorded in the
accounts of the partnership as an expense rather than treated as an allocation of
income, the total amount allocated to each partner for salaries and net income would be
a. Less for both Cora and Zon. b. More for Cora and less for Zon

b. Unchanged for both Cora and Zon. c. d. More for Zon and less for Cora.

2.4. The partnership agreement for the partnership of Brisbane and Ric provided for
salary allowances of P450,000 to Brisbane and P350,000 to Ric, and the residual profit
was allocated equally. During 2007, Brisbane and Ric each withdraw cash equal to 80
percent of their salary allowances. If during 2008, the partnership had profit in excess
of P1,000,000 without regard to salary allowances and withdrawals, Brisbane’s equity in
the partnership would

a. Increase more than Ric’s c. Increase the same as Ric’s

b. Decrease more than Ric’s d. Decrease the same as Ric’s

2.5.Partners C. Fontana and D. Guevarra share income and loss equally after each has
been credited in all circumstances with annual salary allowances of P30,000 and
P24,000, respectively. Under this arrangement, in which of the following circumstances
will Fontana benefit by P6,000 more than Guevarra?

a. Only if the partnership has earnings of P54,000 or more for the year
b. Only if the partnership does not incur a loss for the year
c. In all earnings or loss situation
d. Only if the partnership has earnings of at least P6,000 for the year

PROBLEMS

2.6. Shue, a partner in the Financial Brokers Partnership, has a 30 percent share in
partnership profits and losses. Shue's capital account had a net decrease of 100,000
during 2008. During 2008, Shue withdrew 240,000 as withdrawals and contributed
equipment valued at $50,000 to the partnership. What was the net income of the
Financial Brokers Partnership for 2008? 
A. 633,334
B. 466,666
C. 300,000
D. 190,000

2.7. The JPB partnership reported net income of 160,000 for the year ended December
31, 2008. According to the partnership agreement, partnership profits and losses are to
be distributed as follows:

   
How should partnership net income for 2008 be allocated to J, P, and B?
    
A. Option A
B. Option B
C. Option C
D. Option D

2.8. Xing Corporation owns 80 percent of the voting common shares of Adams
Corporation. Noncontrolling interest was assigned $24,000 of income in the 2009
consolidated income statement. What amount of net income did Adams Corporation
report for the year? 
A. $150,000
B. $96,000
C. $120,000
D. $30,000

2.9. The partnership’s 2003 income statement follows:

Revenues P192,900

Expenses (including salary, interest and bonus) 99,400

Net Income P 93,500

Ignoring income tax, what is the amount of bonus to Paredes?

a. P23, 376 c. P30, 000


b. P24, 000 d. none of the above

2.10. ABC’s partnership provided for the following distribution of profit and losses:

First, Alberto is to receive 10% of the net income up to P1,000,000 and 20% on the
amount in excess thereof;

Second, Bustamante and Cancio each are to receive 5% of the remaining income in
excess of P1, 500,000 after Alberto’s share as per above; and

The balance is to be divided equally among the partners.

For the year just ended, the partnership realized a net income of P2,500,000 before
distribution to partners.

How much is the share of Alberto in the income of the partnership?


a. P1,000,000 c. P1,080,000
b. P1,300,000 d. P1,100,000

CHAPTER 3: PARTNERSHIP DISSOLUTION: CHANGES IN OWNERSHIP

THEORY

3.1. Metcalf, Petersen, and Rusell are partners with capital balances of P50,000,
P30,000, P20,000, respectively. The partners share income and loss equally. For an
investment of P50,000 cash, Andersen is to be admitted as a partner with a one-fourth
interest in capital and income. Based on this information, the amount of Andersen’s
investment can best be justified by which of the following?

a. Andersen will receive a bonus from the other partners upon her admission to the partnership.

b. Assets if the partnership were overvalued immediately prior to Andersen’s investment.

c. The books value of the partnership’s net assets was less than their fair value immediately prior
to Andersen’s investment.

d. Andersen is apparently bringing goodwill into the partnership, and her capital account will be
credited for the appropriate amount.

3.2. If L is the total capital of a partnership before the admission of a new partner, I is
the total capital of the partnership after the investment of a new partner, M is the
amount of the new partner’s investment, B is the amount of the capital credit to the new
partner, then there is:

a. L bonus to the new partner if I = L + M and B < M.

b. Goodwill to the old partners if I > (L + M) and B = M.

c. Neither bonus nor goodwill if I = L – M and B > M.

d. Goodwill to the new partner if I > (L + M) and B < M.

3.3. In the AMS-ALMS partnership, Ams and Alms had a capital ratio of 3:1 and a profit
and loss ratio of 2:1, respectively. The bonus method was used to record Abs’
admittance as a new partner. What ratio should be used to allocate, to Ams and Alms,
the excess of Abs’ contribution over the amount credited to Abs’ capital account?

a. Ams and Alms’ new relative capital ratio. c. Ams and Alms’ old capital ratio.

b. Ams and Alms’ new relative profit and loss d. Ams and Alms’ old profit and loss ratio.
ratio.
3.4. When Nora retired from the partnership of Nora, Norman, and Norla, the final
settlement of Nora’s interest exceeded Nora’s capital balance. Under the bonus
method, the excess:

a. Was recorded as goodwill.

b. Was recorded as an expense.

c. Reduced the capital balances of Norman and Norla

d. Had no effect on the capital balances of Norman and Norla.

3.5. The partnership of Eugene, Alfred and Jericho shared profits and losses equally.
When Eugene withdrew from the partnership, the partners agreed that there was
unrecorded goodwill in the partnership. Under the bonus method, the capital balances
of Alfred and Jericho were

a. Not affected.

b. Each reduced by one-half of the total amount of the unrecorded goodwill.

c. Each reduced by one-third of the total amount of the unrecorded goodwill.

d. Each reduced by one-half of Eugene’s share of the total amount of the unrecorded goodwill.

PROBLEMS

3.6.Ric and Jason shares net income or losses 40% and 60%, respectively. On
January 2, 2007, Gen was admitted to the Ric- Gen Jason Partnership by the
investment of the net assets of her highly profitable single proprietorship. The partners
agreed to the following current fair values of the identifiable net assets of Gen’s single
proprietorship; Current assets P70,000, Plant assets P230,000, Liabilities P200,000.

The balance sheet of the Ric and Jason partnership on December 31, 2006, follows:

Ric and Jason Partnership

Condensed Balance Sheet

December 31,2006

Current Assests P100,000 Liabilities P300,000

Plant assets (net) 500,000 Ric, Capital 200,000

Jason, Capital 100,000

P600,000 P600,000
Gen’s capital account was credited for P120,000, the partners agreed further that the carrying
amounts of the net assets of the Ric and Jason partnership were equal to their current fair
values, and that the accounting records of the old partnership should be used for the new
partnership. The following income-sharing plan was adopted for the new partnership:

1. A bonus of 10% of net income after deduction of the bonus to Gen as managing
partner.

2. Remaining net income or loss as follows: 30% to Ric; 40% to Jason and
30% to Gen.

For the year ended December 31, 2007, the Ric-Gen Jason Partnership had net income
of P55,000 before the bonus Gen.

Compute: (1) the capital of Jason after Gen’s admission on January in the partnership, and
(2) share of Jason in the net income:

a. (1) P88,000; (2) P20,000 c. (1) P192,000; (2) P25,000

b. (1) P88,000; (2) P19,800 d. (1) P120,000; (2) P20,000

3.7. C and D wish to acquire the partnership interest of their partner E on July 10, 2007.
Partnership assets are to be used to acquire E’s partnership interest, the balance sheet
for the CDE Partnership on that date shows the following:

CDE Partnership

Balance Sheet

July 10, 2007

Cash P 74,000 Liabilities P 45,000

Receivables (net) 36,000 C, capital 120,000

Equipment (net) 135,000 D, capital 60,000

Goodwill 30,000 E, capital 50,000

P 275,000 P 275,000

C, D, and E share earning in the ratio of 3:2:1, respectively. E wants to retire from the
partnership. If E is paid P54,000 and bonus method is used, what is the capital account
balance of C and D:

C D C D
a. P117,000 P58,400 c. P117,600 P60,000

b. P120,000 P60,000 d. P122,400 P61,600

3.8. Partners A, B, C and D, who share profits 5:3:1:1 respectively, decide to dissolve.
Capital balances at this time are P60,000, P40,000, P30,000 and P10,000 respectively.
Before selling the firm’s assets, the partners agree to the following:

1. Partnership furniture and fixtures, with the book value of P12,000, is to be


taken over by partner A at a price of P15,000.

2. Partnership claims of P20,000 are to be paid off and the balance of cash on
hand, P30,000, is to be divided in a manner that will avoid the need for any
possible of cash from a partner.

How much the P30,000 cash be distributed to the partners?

A B C D

a. P0 P 0 P30,000 P 0

b. P(2,500) P11,500 P20,500 P500

c. P0 P20,000 P10,000 P 0

d. P0 P20,000 P20,000 P 0

3.9. Joy, Inna and Izza, sharing profits and losses 50%, 30%, and 20%, respectively,
have capital credit balances of P40,000, P30,000, and P20,000, respectively. They
decided to admit a new partner, Rita, to a 30% interest in the partnership upon Rita’s
investment of an amount equal to 5/6 of her capital credit with no assets adjustment
recognized. Immediately after the admission of Rita, the capital credit balance of Inna
will be:

a. P28,200 b. P30,000 c. P31,800 d. P33,000

3.10. J. Olson, L. Quizon and N. Dizon are partners with capital balances of P224,000,
P260,000 and P116,000 respectively, sharing profits and losses in the ratio of 3:2:1. D.
Sison is admitted as a new partner bringing with him expertise and reputation. He is to
invest cash for a 25% interest in the assets of the partnership which includes a credit of
P70,000 for goodwill that is recognized upon his admission.

How much cash should Sison contribute?

a. P130,000 c. P200,000
b. P185,000 d. P150,000

CHAPTER 4: PARTNERSHIP LUMP-SUM LIQUIDATION

THEORY

4.1. Prior to partnership liquidation, a schedule of possible losses is frequently prepared to


determine the amount of cash that may be safely distributed to the partners. The
schedule of possible losses

a. Consists of each partner’s capital account plus loan balance, divided by that
partner’s profit-and-loss sharing ratio.

b. Shows the successive losses necessary to eliminate the capital accounts of


partners (assuming no contribution of personal assets by partners).

c. Indicates the distribution of successive amounts of available cash to each


partner.

d. Assumes contribution of personal assets by partners unless there is a substantial


presumption of personal insolvency by the partners.

4.2. The final cash distribution to the partners in a partnership in liquidation should be made
in accordance with

a. Balances of the partners’ capital accounts.

b. Partners’ profit and loss sharing ratio.

c. Ratio of capital contributions made by the partners.

d. Ratio of capital contributions less withdrawals made by the partners.

4.3. In a partnership liquidation, the final cash distribution to the partners should be made in
accordance with the

a. Partners’ profit and loss sharing ratio.

b. Balances of the partners’ capital accounts.

c. Ratio of capital contributions made by the partners.

d. Ratio of capital contributions less withdrawals made by the partners.

For items 4.4 & 4.5.


Mandela, B. Clarion, C. Yamson, and D. Lobregat are partners sharing profits and
losses equally. The partnership is insolvent and is to be liquidated. The status of the
partnership and each partner is as follows:

Personal Personal

Assets Liabilities

Partnership (exclusive of (exclusive of

Capital partnership partnership

Balance interest) interest)

Mandela (P45,000) P300,000 P120,000

Clarion (30,000) 90,000 180,000

Yamson 60,000* 240,000 15,000

Lobregat 90,000* 3,000 84,000

*Deficit

4.4. The partnership creditors

a. must first seek recovery against Yamson because he is personally


solvent and has a negative capital balance,
b. will not be paid in full regardless of how they proceed legally
because the partnership assets are less than the partnership
liabilities.
c. Will have to share Clarion’s interesting the partnership on a pro rata
basis with Clarion’s personal creditors.
d. Have first claim to the partnership assets before any partner’s
personal creditors have rights to the partnership assets.

4.5. The partnership creditors may obtain recovery of their claims

a. in the amount of P18,750 from each partner


b. from the personal assets of either Mandela or Clarion
c. from the personal assets of either Yamson or Lobregat
d. from the personal assets of either Mandela or Yamson for all or
some of their claims.

PROBLEMS

4.6. Partners R, E, and H share net income and losses in a 5:3:2 ratio, respectively. At
the end of a very unprofitable year, they decided to liquidate the partnership. The
partners’ capital account balances on this date were as follows: R P22,000; E P24,900
and H P15,000. The liabilities in the balance sheet amounted to P30,000, including a
loan of P10,000 form R. The cash balance was P6,000.

The partners plan to sell the noncash assets on a piece meal basis and to distribute
cash as rapidly as it becomes available. All three partners are personally solvent.

If R received a total of P20,000 as a result of the liquidation, what was the total amount
realized by the partnership on the sale of the non-cash assets?

a. P61,900 b. . P85,900 c. P73,900 d. P24,000

4.7. Dennis, Brisbane and Ric form a partnership on January 1, 2007 investing P15,000,
P10,000 and P10,000 respectively; profits are to be shared in the ratio of 2:1:1
respectively. It is agreed that 6% (1/2 of 1% per month) is to be charged on withdrawals
that decrease capitals below the original investments. On March 1, Dennis withdraws
P5,000. Business is unsatisfactory and it is decided to dissolve the partnership.
Partnership assets realize P5,000 and the accountant distributes this cash to the proper
parties on November 1, 2007. All parties are solvent and proper settlement is made
among partners the same day.

How much will Dennis contribute to the partnership for the final settlement?

a. P2,500 b. P2,600 c. P2,700 d. . P2,800

4.8. On December 31, the partnership accounts of I. Gabon, J. Hipolito and K. Imperial
who share profits and losses in the ratio of 5:3:2 follow:

I. Gabon, drawing – debit P12,000

K. Imperial, drawing – credit 4,800

Accounts receivable – Gabon 7,200

Loans payable – Hipolito 14,400

I.Gabon, capital 59,400

J.Hipolito, Capital 44,400

K. Imperial, capital 39,000

Total partnership assets on this day stands at P211,200, including cash of P64,200. The
partnership is liquidated and imperial ultimately receives P33,000 in final liquidation.

How much is the total loss on realization of the partnership?

a. P64,200 c.P54,000
b.P31,200 d.P10,800

For items 4.9.& 4.10.

As of December 31, 2003, the books of GTB Partnership showed capital balances of
Gueco – P40,000; Tiongco – P25,000; Barcelona – P5,000. The partners’ profit and
loss ratio was 3:2:1, respectively. The partners decided to dissolve and liquidate. They
sold all the noncash assets for P37,000 cash. After settlement of all liabilities amounting
to P12,000, they still have P28,000 cash cash left for distribution.

4.9.The loss on realization of the noncash assets was:

a. P42,000 c.P45,000

b.P40,000 d.28,000

4.10. Assuming that any debit balance of partners’ capital is uncollectible, the share of
Gueco on P28,000 cash for distribution was

a. P19,000 c. P18,000

b.P17,800 d.P28,000

CHAPTER 5: PARTNERSHIP INSTALLMENT LIQUIDATION

THEORY

5.1. In a partnership liquidation, the final cash distribution to the partners should be
made in accordance with the:

a. Partners’ profit and loss sharing ratio.

b. Balances of the partners’ loan and capital accounts.

c. Ratio of the capital contribution by the partners.

d. Ratio of the capital contributions less withdrawals by the partners.

5.2. In accounting for the liquidation of a partnership, cash payments to partners after all
non-partner creditors’ claims have been satisfied , but before the final cash distribution,
should be according to:

a. The partners’ relative profit and loss sharing ratios.

b. The final balances in partner capital accounts.


c. The partners’ relative share of the gain or loss on liquidations.

d. Safe payments computations.

5.3. In accounting for the liquidation of a partnership, cash payments to partners after all
non partner creditors’ claims have been satisfied, but before the final cash distribution,
should be according to

a. the partners’ relative profit and loss sharing ratio


b. the final balances in partner capital accounts
c. the partners’ relative share of the gain or loss on liquidations
d. safe payments computations

5.4. In a partnership liquidation, the final cash distribution to the partners should be
made in accordance with the

a. partner profit and loss sharing ratios


b. balances of partner capital accounts
c. ratio of the capital contributions by partners
d. safe payment computations

5.5. The doctrine of marshalling of assets

a. is applicable only if the partnership is insolvent


b. allows partners to first contribute personal assets to unsatisfied partnership
creditors
c. is applicable if either the partnership is insolvent or individual partners are
insolvent.
d. Amount owed to personal creditors and to the partnership for debit capital
balances are shared proportionately from the personal assets of the partners.

PROBLEMS

5.6. A balance sheet for the partnership of J, K, and L who share profits 2:1:1
respectively, shows the following balances just before liquidation:

Cash Other Liabilities J, Cap. K, Cap. L, Cap.


assets

P48,000 P238,000 P80,000 P88,000 P62,000 P56,000

In the first month of liquidation, P128,000 was received on the sale of certain
assets. Liquidation expenses of P4,000 were paid, and additional liquidation
expenses of P3,200 are anticipated before liquidation is completed. Creditors were
paid P22,400. Available cash distributed to the partners.

The cash to be received by each partner based on the above data:


J K L

a. P56,000 P28,300 P28,300

b. P86,000 P61,000 P55,000

c. P29,400 P32,700 P26,700

d. P88,000 P62,000 P56,000

5.7. Partners R. Romero, S. Segundo, and T. Tenorio, who share income and loss in
the ratio of 3:5:2, respectively, have decided to liquidate their partnership. At the time of
liquidation, the balance sheet of the partnership consisted of the following:

Assets Liabilities and Capital

Cash P120,000 Accounts payable P93,000

Other assets 360,000 Loan from Segundo 30,000

R, Romero, capital 108,000

S. Segundo, capital 120,000

T. Tenorio, capital 129,000

Total assets P480,000 Total liabilities and capital 480,000

The partners desire to prepare an installment distribution schedule showing how cash
would be distributed to partners as assets are realized.

In the schedule of maximum absorbable loss, the maximum absorbable loss for each
partner would be

a. Romero P360,000; Segundo, P240,000; Tenorio, P645,000


b. Romero P300,000; Segundo, P600,000; Tenorio, P225,000
c. Romero P450,000; Segundo, P525,000; Tenorio, P375,000
d. Romero P360,000; Segundo, P300,000; Tenorio, P645,000

For items 5.8 to 5.10

W. Aguila, R. Balingit, and J. Corpuz are partners. On January 3, 2003, their capital
balances and profit and loss ratio are as follows:

Capital Profit & loss Ratio


W. Aguila P25,000 60%

R. Balingit 50,000 25%

J. Corpuz 60,000 15%

Corpuz withdrew P10,000 during the year. Net loss on December 31, 2003
totaled P20,000. Hence, the partners decided to liquidate the partnership. It is uncertain
how much of the assets will ultimately yield but favorable realization is expected. It is,
therefore, agreed to distribute cash as it becomes available. There are unpaid liabilities
of P5,000 and cash on hand of P700.

5.8. The amount of noncash assets before liquidation is:

a. P110,000 c. P109,300

b. P104,300 d. P105,000

5.9. The amount to be realized by the partnership on the sale of its assets so that Aguila
will receive a total of P19,000 in the final settlement of his interest is

a. P103,300 c. P119,300

b. P 9,300 d. P 6,000

5.10. If Corpuz received a total ofP33,000, the amount that Balingit would have received
at this point is:

a. none c.P5,000

b. P2,000 d.P21,667

CHAPTER 6: CORPORATE LIQUIDATION

THEORY

6.1. Which of the following best illustrates the insolvency of a firm?

a. The filing of bankruptcy proceedings against the firm.


b. A deficit in the firm’s retained earnings.
c. The firm has more liabilities than assets.
d. The firm has negative working capital.

6.2. If the value of the pledged property is lesser than the obligation, what is the
treatment of the liability?

a. Partially secured
b. Fully secured
c. Collateralized
d. Unsecured

6.3. The primary difference between a balance sheet and an accounting statement of
affairs is that:

a. A balance sheet reflects book values, while the statement of affairs emphasizes
realization values.
b. Assets are arranged in a different sequence.
c. Liabilities are arranged in a different sequence.
d. Owners’ equity is not considered in the statement of affairs.

6.4. An accounting statement of affairs of a corporation in financial difficulty indicates


that unsecured creditors would receive P0.40 on the peso. Which one of the following
assets is most likely to realize the smallest percentage of its book value?

a. Accounts receivable
b. Inventories
c. Plant and equipment
d. Goodwill

6.5. If the dividend of 80% is allocable to Class 7 unsecured creditors based on an


accounting statement of affairs, it correctly may be concluded that

a. All unsecured claims will receive the same percentage of return.


b. All unsecured claims will be paid in full.
c. Class 1 through 6 claims will be paid in full.
d. Stockholders will receive 20% of their equity.

PROBLEMS

6.6. In may 2015, it was determined that it is necessary to complete the work in process
of Wild West Corp. To complete the work in progress, P10000 book value of raw
materials and supplies and P10000 conversion cost will be required. When completed,
these goods will probably sell for approximately P50000. The raw materials, which have
a book value of P40000, have an estimated total realizable value of P20000. What is
the estimated amount that will become available for unsecured creditors as a result of
the realization of the work in progress?

a. 50000
b. 35000
c. 30000
d. 0

6.7. The accountant of Radi Corp. prepared a statement of affairs. Assets which are no
claims or liens are expected to produce 700000. Unsecured claims of all classes
totalled 1,050,000. The following data are claims deemed outstanding:

1. Accrued salaries, 15,000


2. Unrecorded note for 10,000, on which 600 of interest has accrued held by Telen
Co.
3. A note for 30,000 secured by 40,000 receivable, estimated to be 60% collectible
held by Biboso Co.
4. A 15000 note, on which 300 interest has accrued held by Fernandez Pty.
Property with a book value of 10000, and a market value of 18000 is pledged to
guarantee payment of principal and interest.
5. Unpaid income taxes of 35,000.

What is the amount realized by partially secured creditors?

a. 10,600
b. 19,500
c. 24,900
d. 27,900

6.8. Silva Corp. was forced into bankruptcy and is in the process of liquidating assets
and paying claims. Unsecured claims will be paid at the rate of 0.40 on the peso.
Fernandez holds a 30000 noninterest bearing note receivable from Silva collaterized by
an asset with a book value of 35000 and a liquidation value of 5000. The amount to be
realized by Fernandez on this note is

a. 5,000
b. 12,000
c. 15,000
d. 17,000

For items 6.9. & 6.10.

The following selected account balances were taken from the balance sheet of Quitting
Corp. as of December 31, 2014, immediately before the take over of the trustee:

Marketable securities 300,000

Inventories 110,000

Land 150,000

Building 400,000

Additional information:

 Marketable securities have present market value of 320,000. These securities


have been pledged to secure notes payable of 280,000.
 The estimated worth of inventories is 70,000. However, inventories with book
value of 50,000 have been pledged to secure notes payable of 60,000. The
realizable value of the inventories pledged is estimated to be 40,000.
 Land and building are estimated to have a total realizable value of 450,000. This
property is pledged to secure the mortgage payable of 250,000.

1. What is the estimated amount available for preferred claims and unsecured
creditors out of assets pledged with fully secured creditors?
a. 840,000
b. 810,000
c. 770,000
d. 240,000
2. What is the total amount of net free assets?
a. 810,000
b. 770,000
c. 270,000
d. 240,000

SOLUTIONS:

1.1. The answer is letter D

1.2. The answer is letter D

1.3. The answer is letter C

1.4. The answer is letter A

1.5. The answer is letter A

1.6. The answer is letter D

Abena Buendia Total

FMV 90000 60000 150000

Cost 60000 70000 130000

18000 (18000)

1.7. The answer is letter C

B J L

100000 160000 110000

(70000)

90000

1.8. The answer is letter C


M N Total

100000 84000 184000

No goodwill/ intangible assets are recorded.

1.9. The answer is letter B

60 40

M N

40000 100000

120000 X

30000

(60000) _____

130000 130000

X= 30000

1.10. The answer is letter C

K S

59625 33500

(900)

(555) (405)

59070 32195

2.1. The answer is letter D

2.2. The answer is letter C

2.3.The answer is letter B

2.4. The answer is letter A

2.5. The answer is letter C

2.6. The answer is letter C

Shue 30%

(100,000)
50,000

(240,000)

90,000/0.3= 300,000

2.7. The answer is letter C

J P B Total

80,000 60,000 30,000 140,000

16,000 8,000 16,000 40,000

(6,000) (8,000) (6,000) (20,000)

60,000 60,000 40,000

2.8. The answer is letter C

240,000/0.2= 120,000

2.9. The answer is letter D

2.10. The answer is letter C

A B C

100000 30000 30000

300000

2040000 (equally) 680000 680000 680000

1080000

3.1. The answer is letter C

3.2. The answer is letter B

3.3. The answer is letter D

3.4. The answer is letter C

3.5. The answer is letter D

3.6.The answer is letter A

TCC TAC

R 200 (8)
J 100 (12) 88,000

G 100 20 120

400 0 400

Bonus to Gen = 5,000

Share of J in the residual profit ( 50,000 x 40 % ) = 20,000

3.7. The answer is letter D

Equity balance of E 50,000

Amount Paid 46,000

Bonus to C & D 4,000

C 120,000 + ( 3/5 x 4,000) 122,400

D 60,000 + ( 2/5 x 4,000) 61,600

3.8. The answer is letter B

A B C D Total

60 40 30 10 140

(15) (15)

1.5 .900 .300 .300 3

46,500 40.900 30,300 10,300 128,000

(49,000) (29,400) (9,800) (9,800) (98,000)

ANS  ( 2,500) 11,500 20,500 500 30,000

2,500 ( 1,500) (.500) (500)

0 10T 20 0 30T
3.9. The answer is letter A

TCC TAC

J 40 ( 3 ) 37,000

In 30 ( 1.8 ) 28,200

IZ 20 ( 1.2 ) 18,800

R 30 6 36

120 0 120 [ 90/ (1 – 5/6 x 30 ) ]

3.10. The answer is letter A

3 2 1

J L N Total

224,000 260,000 116,000 600,000

600,000/75%= 800,000- 600,000= 200,000

200,000

(70,000)

130,000

4.1. The answer is letter B

4.2. The answer is letter A

4.3. The answer is letter B

4.4. The answer is letter D

4.5. The answer is letter D

4.6. The answer is letter A

R E H Total

Equity 32,000 24,900 15,000 71,900


Loss (12,000) ( ) ( ) ( 24,000)

CAFD 20,000 _________ _________ 47,900

Proceeds = 47,900 + 20,000 – 6,000 = 61,900

4.7. The answer is letter B

Dennis Brisbane Ric Total

15,000 10,000 10,000 35,000

(5,000) (5,000)

( 200 ) ( 200 )

100 50 50 200

9,900 10,050 10,050 30,000

(12,500) ( 6,250) (6,250) (25,000)

( 2,600 ) _________ _________ 5,0000

4.8. The answer is letter C

5 3 2

I J K

44400 43800

40200 14400

58800

33000

15800/20%= 54000
4.9. The answer is letter A

3 2 1

Cash N/C G T B Liabilities

3000 79000= 40000 25000 5000 12000

37000

42000 Loss

4.10. The answer is letter B

3 2

S T B

40000 25000 5000

21000 14000 7000

19000 11000 (2000)

(1200) (800) 2000

17800

5.1. The answer is letter B

5.2. The answer is letter D

5.3. The answer is letter D

5.4. The answer is letter A

5.5. The answer is letter C

5.6. The answer is letter C

J K L Total

88,000 62,000 56,000 206,000

58.6 ( 29,300) ( 29,300) ( 117.2 )

29,400 32,700 26,700 88,800


5.7. The answer is letter A

R S T

108000 120000 129000

÷ 30% 50% 20%

360000 240000 645000

5.8. The answer is letter C

Total Capital 135000

10000

20000

105000

Liabilities 5000

100000

Cost of Asset 700

Noncash Assets 109300

5.9. The answer is letter A

25000

19000

6000 LOSS

(10000) Total Loss

109300-10000= 99300

5.10. The answer is letter D

33000

(50000)

17000/15%= 133333
133333x25%= 28333

(28333)

50000

21667

6.1. The answer is letter C

6.2. The answer is letter A

6.3. The answer is letter A

6.4. The answer is letter D

6.5. The answer is letter C

6.6. The answer is letter B

Estimated upon completion

Less cost to complete: 50000

Raw materials [P10000X 20000/40000)]5000

Conversion cost 10000 15000

Amount expected to realized upon completion 35000

6.7. The answer is letter D

Total free assets 700000

Less unsecured claims with priority:

Accrued salaries 15000

Unpaid income taxes 35000 50000

Net free assets 650000

Divide by total unsecured claims without priority:

Total unsecured claims of all classes

(with or without priority) 1050000

Less unsecured claims with priority

(accrued salaries and unpaid taxes) 50000 1000000


% of recovery 65%

Realizable amount of AR (60% X 40000) 24000

Add: unsecured portion [(30000-24000) x 65%] 3900

Total amount realized by partially secured creditors 27900

6.8. The answer is letter C

Cash received in full as secured creditor 5,000

Cash received as unsecured creditor [(30000-5000)x0.40] 10,000

Total amount realized from the note 15000

6.9. The answer is letter D

Marketable securities (320,000-280,000) 40,000

Land and building (450,000- 250,000) 200,000

Estimated amount available out of assets pledged

With fully secured creditors 240,000

6.10. The answer is letter C

Marketable securities (320,000-280,000) 40,000

Land and building (450,000- 250,000) 200,000

Unpledged portion of inventories (70,000- 40,000) 30,000

Total amount of net free assets 270000


P
A R
T N
E R
S H
IP FORMATION
1. Which of the following is not a characteristic of a partnership?
a. The partnership itself pays no income taxes
b. It is easy to form a partnership
c. Any partner can be held personally liable for all debts of the business
d. A partnership requires written Article of Partnership

2. The partnership form of business is:


a. An economic entity
b. A taxable entity
c. A fiscal entity
d. A separate legal entity, just as a corporation is a legal entity

3. Which of the following is not an advantage of partnership over a corporation?


a. Ease of formation
b. Unlimited liability
c. The elimination of taxes at the entity level
d. All of the above

4. A partner’s withdrawal of assets from a limited liability partnership that is considered a


permanent reduction of in that partner’s equity is debited to the partner’s:
a. Drawing account
b. Retained earnings account
c. Capital account
d. Loan receivable account

5. For financial accounting purposes, assets of an individual partner contributed to partnership are
recorded by the partnership at:
a. Historical cost
b. Book value
c. Fair market value
d. Lower of cost or market

6. On December 1, 2009, DD and EE formed a partnership with each contributing the following
asset at fair market values:
DD EE
Cash 9,000 18,000
Machinery and Equipment 13,500
Land 90,000
Building 27,000
Office Furniture 13,500
The land and building are subject to a mortgage loan of P54,000 that the partnership will
assume. The partnership agreement provides that DD and EE share profits and losses, 40% and
60%, respectively and partners agreed to bring their capital balances in proportion to the profit
and loss ratio and using the capital balance of EE as the basis. The additional cash investment
made by DD should be:
a. 18,000
b. 85,500
c. 134,100
d. 166, 250
DD, Capital= 9+13.5+13.5=36
EE, Capital= 18+90+27-54=81
81/.60=135
135*.40=56-36=18 A

7. JJ and KK are joining their separate business to form a partnership. Cash and noncash asset are
to be contributed for a total capital of 300,000. The noncash assets to be contributed and
liabilities to be assumed are:
JJ KK
Book Value Fair Value Book Value Fair Value
Accounts Receivable 22,500 22,500
Inventories 22,500 33,750 60,000 67,500
Equipment 37,500 30,000 67,500 71,250
Accounts Payable 11,250 11,250 7,500 7,500
The partner’s capital are to be equal after all contributions of assets and assumptions of
liabilities. The total assets of the partnership.
a. 318,750
b. 300,000
c. 281,250
d. 225,000
Equity=Assets-Liabilities
300,000=X-(11,250+7,500)
Assets=X=318,750 A

8. Refer to number 8, the amount of cash that each partner must contribute.
a. JJ=75,000; KK=18750
b. JJ=75,000; KK=11,250
c. JJ=161,250; KK= 157,500
d. JJ= 127,500; KK= 11,250
For JJ; 150,000=Cash to be contrubuted+22,500+33,750+30,000+(-11250)
Cash to be contributed=75,000
For KK; 150,000=Cash to be contributed+67,500+71,250+(-7500)
Cash to be contributed=18,750 A

9. Jones and Smith formed a partnership with each partner contributing the following items:

  
Assume that for tax purposes Jones and Smith agree to share equally in the liabilities assumed
by the Jones and Smith partnership. Refer to the above information. What is the balance in each
partner’s capital account for financial accounting purposes?


Jones Smith
Assets at fair value
Jones: 80,000+400,000 480,000
Smith: 40,000+280,000 320,000
Less: Liabilities assumed 120,000 60,000
Capital 360,000 260,000
10. MM, NN, and OO are partners with capital balances on December 31, 2012 of P300,000,
P300,000 and P200,000, respectively. Profits are shared equally. OO wishes to withdraw and it is
agreed that OO is taken certain equipment with second-hand value of P50,000 and a note for
the balance of OO’s interest. The equipment are carried on the books at P65,000. Brand new
equipment may cost P80,000. Compute for: (1) OO’s acquisition of the second-hand equipment
will result to reduction in capital; (2) the value of the note that will OO get from the
partnership’s liquidation,
a. (1) 15,000 each for MM and NN (2) 150,000
b. (1) 5,000 each for MM, NN, and OO (2) 145,000
c. (1) 5,000 each for MM, NN, and OO (2) 195,000
d. (1) 7,500 each for MM and NN (2) 145,000

B
1. Reduction in Capital:
Equipment at carrying value 65,000
Equipment at secondhand value (fair value) 50,000

Decrease in equipment 15,000


Multiply by: Profit & Loss Ratio of MM, NN,and OO 1/3
Reduction in capital 5,000

2. Notes Payable to OO
Unadjusted Capital of OO 200,000
Less: Share in the decrease of equipment 5,000
Adjusted capital of OO 195,000
Less: Equipment receive at secondhand value 50,000
Value of notes payable 145,000

Incidentally, the juournal entry would be:


OO, Capital 200,000
NN, Capital 5,000
MM, Capital 5,000
Eqipment, carrying value 65,000

---Don’t do nothing because you feel you can only do little, do what you can---
PARTNERSHIP OPERATIONS
1. Partnership drawings are:
a. always maintained in a separate account from the partner’s capital account
b. equal to partner’s salaries
c. usually maintained in a separate draw account with any excess draws being debited directly to the
capital account
d. not discussed in the specific contract provisions of the partnership

2. Drawings are:
a. are advances to a partnership
b. are loans to a partnership
c. are a function of interest on partnership average capital
d. are the same nature as withdrawals

3. In a partnership, interest on capital investment is accounted for as a(n)


a. return of investment
b. expense
c. allocation of net income
d. reduction of capital

4. What is the underlying purpose of the interest on capital balances component of allocating
partnership profits and losses?
a. Compensate partners who contribute economic resources to the partnership
b. Reward labor and expertise contribution
c. Reward for special responsibilities undertaken
d. None of the above

5. What is the underlying purpose of the salary component of allocating partnership profits and
losses?
a. Compensate partners who contribute economic resources to the partnership
b. Reward labor and expertise contribution
c. Reward for special responsibilities undertaken
d. None of the above

6. X, Y, and Z, a partnership formed on January 1, 2012 had the following initial investment:

XX 100,000
YY 150,000
ZZ 225,000
The partnership agreement states that the profits and losses are to be shared equally by the
partners after consideration is made for the following:

 Salaries allowed to partners: P60,000 for X, P48,000 for Y, and P36,000 for Z.
 Average partners’ capital balances during the year shall be allowed by 10%.
Additional Information

On June 30, 2012, X invested an additional P60,000


ZZ withdrew P70,000 from the partnership on September 30, 2012
Share the remaining partnership partnership profit was P5,000 for each partner.

Interest on average capital balances of the partners totalled:

a. 48,750
b. 53,750
c. 57,625
d. 60,625

A
Average Capital:
X: 100,000*6=600,000
160,000*6=960,000 1,560,000/12=130,000
Y: (same with beginning capital) 150,000
Z: 255,000*9= 2,025,000
155,000*3= 465,000 2,490,000/12= 207,500
487,500
X: interest rate 10%

7. Refer to number 6, partnership net profit at December 31, 2012 before salaries. Interests, and
partners share on the remainder was:
a. 199,750
b. 207,750
c. 211,625
d. 222,750

X Y Z Total
Salaries 60,000 48,000 36,000 144,000
Interest-refer above 13,000 15,000 20,750 48,750
Balance (P5,000 each) 5,000 5,000 5,000 15,000
207,750

8. XX and YY formed a partnership on January 2, 2009 and agreed to share profits and losses in the
ratio of 90% and 10% respectively. XX contributed capital of P25,000. YY contributed no capital
but has a specialized expertise and manages the firm full time. There were no withdrawals
during the year. The partnership agreement provides for the following:
 Capital accounts are to be credited annually with interest ay 5% of the beginning capital.
 YY is to be paid a salary of P1,000 a month.
 YY is to receive a bonus of 20% of net income calculated before deducting his salary and
interest on both capital accounts
 Bonus, interest, and YY’s salaries are to be considered as partnership expense.

The partnership’s income statement for 2009 follows:

Revenues 96,450
Less: Expenses (including salary, interest, and bonus) 49,700
Net Income 46,750

What is YY’s 2009 bonus?

a. 11,688
b. 12,000
c. 15,000
d. 15,738

Corrected Expense=49,700-1250-12,000=36,450

Corrected Net Income= 96,450-36,450= 60,000

B=.20(60,000-B)
B=15,000

9. The AA, BB, and CC Partnership was formed on January 2, 2009. The original cash investment
were as follows:

AA 96,000
BB 144,000
CC 216,000
According to the general partnership contract, the partners were to be remunerated as follows:

Salaries of 14,400 for AA, 12,000 for BB, and 13,600 for CC
Interest at 12% on the average capital account balances during the year
Remainder divided 40% ,30%, 30% respectively

Income before partner’s salaries for the year ended December 31, 2008, was P92,080. AA invested
an additional P24,000, in the partnership on July 1; CC withdrew 36,000 from the partnership on
October 1; and, as authorized by the partnership contract, AA,BB and CC each withdrew P750
monthly against their shares of net income for the year.

The capital balance of partner CC on December 31, 2008

a. 217,540
b. 208,540
c. 200,224
d. 198,624

10. Refer to #9, if the salaries partners’ are to be recognized as operating expenses by the
partnership, the share of partner BB in the net income?
a. 36,832
b. 28,380
c. 16,380
d. 15,624

9. B 10. C

AA BB CC Total
1/1/2008 96,000 144,000 216,000 456,000
9/1/2008 24,000
10/1/2008 ______ ______ -36,000 ______
12/31/2008 120,000 144,000 180,000 444,000

Average 108,000 144,000 207,000 459,000

Salaries 14,400 12,000 13,600 40,000


Interest 12,960 17,280 24,840 55,080
Balance(4:3:3) (1,200) (900) (900) (3,000)
26,160 28,380 37,540 92,080

Beginning Bal. 96,000 144,000 216,000 456,000


Additional Inv 24,000 24,000

---Example is not the main thing in influencing others. It is the only thing---
PARTNERSHIP WITHOUT LIQUIDATION
1. Transferable interest of the partner includes all of the following except:
a. the partner’s share of the profits and losses of the partnership
b. the right to receive distributions
c. the right to receive any liquidating distribution
d. the authority to transact any of the partnership’s business operation
2. The dissolution of the partnership occurs
a. only when the partnership sells its assets and permanently closes its books
b. only when the partner leaves the partnership
c. at the end of each year, when income is allocated to the partners
d. only when a new partner is admitted to the partnership
e. when there is any change in the individuals who make up the partnership
3. When a new partner joins a partnership by investing assets into the partnership, what method
may be used to record the admission of the new partner?
a. revaluation of the existing assets
b. recognition of goodwill
c. application of the bonus method
d. any of the three or a combination may be applied
4. Which of the following is not a criterion for recognizing a bonus to existing partners when a new
partner joins the partnership?
a. only cash assets were contributed to the partnership by the new partner
b. the existing partners desire to not recognize goodwill on the balance sheet
c. the article of partnership indicate that the bonus method will be used to admit new partners
d. the partner invests more into the partnership that his/her share of total partnership capital after the
investment is made
5. What amount of goodwill can be recognized at the date a partner withdraws form the
partnership?
a. the withdrawing partner’s portion of goodwill
b. the continuing partner’s portion of goodwill
c. goodwill may not be recognized at the date the partner withdraws
d. either the withdrawing partner’s portion of goodwill or the goodwill attributable to the entire
partnership

6. Kravitz and Rowe are partners in an excavating business known as K and R Excavating. The
partners are considering a number of options regarding the partnership including the admission
of a new partner and a potential sale of the partnership. The following information has been
prepared as a basis for evaluating various alternatives:
Items Book Value Fair Value Tax Basis
Cash and Cash Equivalence 20,000 20,000 20,000
Accounts Receivable 85,000 72,000 92,000
Inventory 42,000 30,000 50,000
Prepaid and Other Current Ass 18,000 15,000 18,000
Property, Plant, and Equipmen 358,000 300,000 320,000
Total Assets 523,000 437,000 500,000

Accounts Payable 54,000 54,000 54,000


Other Current Liabilities 29,000 35,000 29,000
Notes/Loan Payable 240,000 240,000 240,000
Kravitz, Capital 120,000
Rowe, Capital 80,000
Total Liabilities and Capital 523,000

The partners currently share profits and losses 60% and 40% respectively for Kravitz and Rowe.
Given the stated fair values, if Rowe were to sell ½ of her interest in capital to someone outside
the partnership, what would be the suggested asking price?
a. 21,600
b. 22,800
c. 35,400
d. 40,000

7. Refer to 6, given the stated fair value, if a third party were to convey assets to the partnership in
exchange for a 40% interest in the partnership, what would the value of those assets have to
be?
a. 72,000
b. 118,000
c. 133,333
d. 291,333
6. A 7. A
Rowe, Capital: 80,000-(92,000*.40)= 43,200
X: interest Sold 1/2
Suggested Existing Price 21,600

Book Value Fair Value Adjustments


Assets (given) 523,000 437,000
Liabilities 323,000 329,000
Capital 200,000 108,000 -92,000
54000+29,000+240,000=323,000
54,000+35,0000+240,000=329.000

CC AC
Old (fair Value) 108,000 108,000
New 72,000 72,000
180,000 180,000

8. RR and XX formed a partnership and agreed to divide initial capital equally, even though RR
contributed P25,000 and XX contributed P21,000 in identifiable assets. Under bonus approach
to adjust capital accounts. XX’s unidentifiable assets should be debited for:
a. 11,500
b. 4,000
c. 2,000
d. Some Other Answer (Please Specify P____)
D. The answer is P0. There would be no unidentifiable assets recognized by the
creation of this new partnership.
9. In the AD partnership, Allen's capital is P140,000 and Daniel's is P40,000 and they share income
in a 3:1 ratio, respectively. They decide to admit David to the partnership. Allen and Daniel agree
that some of the inventory is obsolete. The inventory account is decreased before David is
admitted. David invests P40,000 for a one-fifth interest. What is the amount of inventory written
down? 
A. P4,000
B. P20,000
C. P15,000
D. P10,000
B
Total Agreed Capital after admission of David:
(40,000*5) 200,000
Less: Contribution/Investment of David 40,000
Capital Balances of AD before admission of David 160,000
Capital Contribution (140,000+40,000) 180,000
Reduction of inventory 20,000

10. Roy and Gil are partners sharing profits and losses in the ratio of 1:2 respectively. On July 1,
2011, they decided to form the R and G Corporation by transferring the assets and liabilities
from the partnership to the Corporation in exchange of its shares. The following is the post-
closing trial balance of the partnership.
Debit Credit
Cash 45,000
Accounts Receivable 60,000
Inventory 90,000
Fixed Assets (net) 174,000
Liabilities 60,000
Roy, Capital 94,800
Gil, Capital 214,200
369,000 369000

It was agreed that adjustments be made to the following assets to be transferred to the
corporation:

Accounts Receivable 40,000


Iinventory 68,000
Fixed Assets 180,600
The R&G Corporation was authorized to issue P100 par preference shares and P10 par ordinary
share. Roy and Gil agreed to receive for their equity in the partnership 720 ordinary share each,
plus even multiples of 10 shares for their remaining interest.
The total number of shares of preference and ordinary share issued by the Corporation in
exchange of assets and liabilities of the partnership are:
a. PS= 2540 shares OS=1500 shares
b. PS= 2592 shares OS=1440 shares
c. PS= 2642 shares OS=1440 shares
d. PS= 2642 shares OS=1550 shares 10. B
Total
Capital before Adjustment 309,000
Less: Net Adjustment 35,400
Capital after Adjustment 273,600
Less: Portion Covered by Ordinary
Share, par P10 (720 share to each
partner) 14,400
Portion to be covered by Preferrence
Share, par P100 259,200

Share to be issued:
---Great achievement are not done by strength but by perseverance---
LUMP SUM LIQUIDATION
1. In partnership liquidation, how are partner salary allocations treated?
a. Salary allocations take precedence over creditor payments.
b. Salary allocations take precedence over amounts due to partners with respect to their capital
interests, but not profits.
c. Salary allocations take precedence over amounts due to partners with respect to their capital profits,
but not capital interests.
d. Salary allocations are disregarded.

2. If cash payments to the partners of a limited partnership in liquidation are delayed until all
noncash assets have realized, any cash remaining after all partnership creditors have been paid is
distributed:
a. according the liquidator’s best judgment
b. in the ratio for sharing net income and losses
c. in amounts equal to the partners’ loan and capital account balances
d. in some other manner

3. In the liquidation of a limited partnership, a loan payable to a partner by the partnership is:
a. paid immediately after all outside creditors have been paid in full
b. liabilities still to be paid
c. considered to be a liability of a partnership
d. disregarded

4. Which item is not shown on the schedule of partnership liquidation?


a. current cash balances
b. property owned by the partnership
c. liabilities still to be paid
d. personal assets of the partners

5. The following is the priority sequence in which liquidation proceeds will be distributed for
partnership:
a. partnership drawings, partnership liabilities, partnership loans, partnership capital balances
b. partnership liabilities, partnership loans, partnership capital balances
c. partnership liabilities, partnership loans, partnership drawings, partnership capital balances
d. partnership liabilities, partnership capital balances, partnership loans

6. The following account balances were available for the Perry, Quincy and Renquist partnership
just before it entered liquidation:
Cash 90,000 Liabilities 170,000
Noncash Assets 300,000 Perry, capital 70,000
Quincy, capital 50,000
Renquist, capital 100,000

Perry, Quincy, and Renquist had shared profits and losses in a ratio of 2:4:4. Liquidation
expenses were expected to be P8,000. All partners were solvent. What would be the minimum
amount for which the noncash asset must have been sold for, in order for Quincy to receive
some cash from the liquidation?
a. Any amount in excess of 175,000
b. Any amount in excess of 117,000
c. Any amount in excess of 183,000
d. Any amount in excess of 198,667

Quincy capital before liquidation 50,000


Less: Share in Liquidation Expense 3,200
Quincy capital before realization of noncash assets 46,800
Less: Cash received by Quincy (minimum) 0
Share in the loss on realization 46,800
Divided by P and L 40%
Loss on realization 117,000
Less: NonCash Assets 300,000
Proceeds from Sale 183,000

7. A local partnership is considering possible liquidation because one of the partners (Bell) is
insolvent. Capital balances at the current time are as follows. Profit and loss are divided on a
4:3:2:1 basis respectively.

Bell, Capital 50,000


Hardy, Capital 56,000
Dennard, Capital 14,000
Suddath, Capital 80,000
Bell’s creditors have filed a P21,000 claim against the partnership’s assets. The partnership
currently holds asset reported at P300,000 and liabilities of P100,000. If the assets can be sold for
P190,000. What is the minimum amount that Bell’s creditor would receive?

a. 0
b. 2,000
c. 2,800
d. 6,000
B Cash NonCash Liabilities Bell Hardy Dennard Suddath
Balances 0 300,000 100,000 50,000 56,000 14,000 80,000
Realization 190,000 -300,000 -44,000 -33,000 -22,000 -11,000
Balances 190000 0 100,000 6,000 23,000 -8,000 69,000
Absorption _________________ ________ -4,000 -3,000 8000 -1,000
Balances 190000 0 100,000 2,000 20,000 0 68,000

8. Fleming, Durano, and Mart are partners in a wholesale business. On January 1, 2009, the total
capital was P60,000 and drawings presented as follows:

Capitals Drawings
Fleming 12,500 7,500
Durano 10,000 5,000
Mart 37,500 2,500

Partners agree that profit and loss ratio are shared equally. Because of the failure of some
debtors to pay their outstanding accounts, the partnership loses heavily and is compelled to
liquidate. After exhausting the partnership assets, including those arising from an operating
profit of P9,000 in 2009, they still owe P10,500 to creditors on December 31, 2009. Fleming has
no personal assets but the others are well off.
The partnership liquidation loss:
a. None
b. 20,000
c. 55,500
d. 64,500

9. Refer to #4, the amount to be received by Mart as a result of the liquidation:


a. 1,637.50
b. 9,750
c. 14,250
d. 19,500

3. D 4. B
Balance
Distribution of los
(10,500+54,000)
Balance
10. The Keaton, Lewis, and Meador partnership had the following balance sheet just before entering
liquidation:

Keaton, Lewis, and Meador share profits and losses in a ratio of 2:4:4. Noncash assets were sold for
P180,000. Liquidation expenses were P10,000.

Assume that Keaton was personally insolvent with assets of P8,000 and liabilities of P60,000. Lewis and
Meador were both solvent and able to cover deficits in their capital accounts, if any. What amount of
cash could Keaton's personal creditors have expected to receive from partnership assets?
A) P30,000.
B) P0.
C) P52,000
D) P26,000
E) P34,000
Answer: E

Keaton Lewis Meador Total


Balances before liquidation 60,000 40,000 80,000 180,000
Liquidation Expense (2:4:4) -2,000 -4,000 -4,000 -10,000
Loss on Realization-2:4:4 (180,000-
300,000) -24,000 -48,000 -48,000 -120,000
Balances 34,000 -12,000 28,000 50,000
Additional Investment _______ 12,000 _______ 12,000
Payment to Partners 34,000 0 28,000 62,000

---The secret of llife is not just to live, but to have something worthwhile to live for---
INSTALLMENT LIQUIDATION
1. Which of the following statements is true concerning the distribution of safe payments?
a. The distribution of safe payments assumes that any capital deficit balances will prove to be a total loss
to partnership.
b. Safe payments are equal to the recorded capital balances of partners with positive capital balances.
c. The distribution of safe payments may only be made after all liabilities have been paid.
d. In computing safe payments, partners with positive capital balances are assumed to absorb an equal
share of any deficit balance(s)

2. An advance cash distribution plan is prepared


a. each time cash is distributed to partners in an installment liquidation.
b. each time a partnership asset is sold in an installment liquidation.
c. to determine the order and amount of cash each partner will receive as it becomes available for
distribution
d. none of these

3. In a partnership liquidation, the final cash distribution to the partners should be made in
accordance with the:
a. partners’ profit and loss – sharing ratio.
b. balances of the partners’ capital accounts.
c. ratio of the capital contributions by the partners.
d. ratio of capital distributions less withdrawals by the partners.

4. The first step in preparing an advance cash distribution plan is to


a. determine the order in which partners are to participate in cash distributions.
b. compute the amount of cash each partner is to receive as it becomes available for distribution.
c. allocate any gains (losses) to the partners in their profit – sharing ratio.
d. determine the net capital interest of each partner

5. In an advance plan installment distributions of cash to partners of liquidating, each partner’s


loss absorption potential is computed by
a. dividing each partner’s capital account balance by the percentage of that partner’s capital account
balance to total partners’ capital.
b. multiplying each partner’s capital account balance by the percentage of that partner’s capital account
balance to total partners’ capital.
c. dividing the total of each partner’s capital account less receivables from the partner plus payables to
the partner by the partner’s profit and loss percentage.
d. some other method
AA, BB, and CC are partners sharing profits and loss in the ratio of 4:3:3, respectively. On January 1,
2009, they decided to liquidate the partnership and the balance sheet were prepared as follows:

Assets Liabilities and Capital


Cash 2,000 Liabilities 6,000
Other Assets 46,000 BB, loan 5,000
CC, loan 2,500
AA, capital 14,450
BB, capital 12,550
________ CC, capital 7,500
Total Assets 48,000 Total Liab. & Cap. 48,000

The following transactions as a result of liquidation were as follows:

Book
Value of Payment Payment
Assets Proceeds Liquidation to Cash
Sold from Sale Expense Creditors Withheld
January 12,000 10,500 500 6,000 2,000
February 7,000 6,000 750 1,000
March 15,000 10,000 1,000 2,500
April 12,000 5,000 5,000 0

1. The amount to be received by Partner CC for the month of February?


a. 0
b. 475
c. 1875
d. 2500
2. The amount to be received by Partner BB for the month of April?
a. 0
b. 750
c. 1,000
d. 1250
3. The second payment to any partner(s) under a program of priorities shall be made thus:
a. To BB=6,712.50
b. To AA=1,116.70
c. To CC=6,712.50
d. To AA=1,116.70 and BB=837.50

1.B 2. B 3.D
January AA BB CC Total
Total January 14,450 17,550 10,000 42,000
Possible Loss -15,200 -11,400 -11,400 -38,000
Balance -750 6,150 -1,400 4,000
Loss Absorption 750 -2150 1,400 0
Balance-Payment 0 4,000 0 4,000

February
Total Interest 14,450 13,550 10,000 38,000
Possible Loss -12,700 -9,525 -9,525 -31,750
Balance-Payment 1,750 4,025 475 6,250

March
Total Interest 12,700 9,525 9,525 31,750
Possible Loss -9,700 -7,275 -7,275 -24,250
Balance-Payment 3,000 2,250 2250 7,500

April
Total Interest 9,700 7,275 7,275 24,250
Possible Loss -8,700 -6,525 -6,525 -21,750
Balance-Payment 1,000 750 750 2,500

Under Cash Priority AA BB CC AA BB CC

4. The partnership of Urich, Volks, and Wales liquidated. The partners have shared profits and
losses in the ratio 3:4:3. Prior to liquidation, the capital balances were the following:
Urich -60,000
Volks 120,000
Wales -10,000

Cash and other assets totalled 200,00 with liabilities amounting to 150,000. Liquidation left the
partnership with 120,000, not enough to pay liabilities. Each partner can contribute 20,000 (but
not more) from personal assets. Determine how much cash Urich receives or pays-in as a result
of the liquidation.
Receives(collects) Pays-in
a. 0 20,000
b. 0 84,000
c. 10,000 0
d. 24,000 60,000
Urich Volks Wales Total
Balance before Liquidation -60,000 120,000 -10,000 50,000
Loss on Liquidation (120,000-
200,000) -24,000 -32,000 -24,000 -80,000
Balances -84,000 88,000 -34,000 -30,000
Additional Investment 20,000 0 20,000 40,000
Balances -64,000 88,000 -14,000 10,000
Additional loss (64,000+14,000)
64,000 -78,000 14,000 0
Cash Reveived 0 10,000 0 10,000

5. Gardo and Gordo formed a partnership on July 1, 2009 to operate two stores to be managed by
each of them. They invested 30,000 and 20,000 and agreed to share earnings 60% and 40%,
respectively. All their transactions were for cash, and all their subsequent transactions were
handled through their respective bank accounts as summarized below:
Gardo Gordo
Cash Receipts 79,100 65,245
Cahs Disbursements 62,275 70,695

On October 31, 2009, all remaining noncash assets in the two-stores were sold for cash 60,000.
The partnership was dissolved, and cash settlement was affected. In the distribution of the
60,000 cash, Gardo received:
a. 24,000
b. 26,000
c. 34,000
d. 36,000

B Gardo (40%) Gordo (40%) Total


Initial Investment 30,000 20,000 50,000
Investment (personal disbursement) 62,275 70,695 132,970
Withdrawals (personal withdrawals) -79,100 -65,245 -144,345
Balance before liquidation 13,175 25,450 38,625
Gain on Realization (60,000-38,625) 12,825 8,550 21,375
Balance before payment to partners 26,000 34,000 60,000
Payment to partners -26,000 -34,000 -60,000

---Achievement comes from the person who dares---


Corporate Liquidation
1. What is the inherent limitation of the Statement of Financial Affairs?
a. Many of the amounts reported are only estimations that might prove to be inaccurate.
b. The statement is applicable only to bankruptcy.
c. the statement covers only a short time whereas a bankruptcy may last much longer
d. the figures on the statement vary as to a voluntary and an involuntary bankruptcy

2. Which of the following does not described the accounting statement of affairs?
a. the emphasis is asset net realizable value, not historical cost
b. the Statement of Affairs is concerned only with the assets of the debtor organization, not the claims
c. the statement can also be used in a reorganization
d. the Statement of Affairs is based on estimated values;
Actual realized values maybe different

1. The following information was available on 3-31-2008 for Bankrupt Corporation which they
cannot pay their liabilities when they are due:
Carrying Amount
Cash 16,000
Trade Accounts Receivable (net): Current fair value equal to carrying
amount 184,000
Inventories: NRV, 72,000; pledged on 84,000 of note payable 156,000
Plant assets: Current fair value, P269,600; pledged on mortgage note
payable 536,000
Accumulated Depreciation of plant assets 108,000
Supplies: Current fair value, P6,000 8,000
Wages Payable, all earned during March 23,200
Property Taxes Payable 4,800
Trade Accounts Payable 240,000
Notes Payable, 84,000 secured by inventories 160,000
Mortgage payable, including accrued interest of P1,600 201,600
Common Stock 400,000

The estimated gains on realization of assets:

a. 84,000
b. 158,400
c. 244,400
d. Some other Answer (Please Specify P___)
2. Refer to 1, the expected recovery percentage of unsecured creditors:
a. 75%
b. 78%
c. 76%
d. 77%
3. The estimated deficiency to unsecured creditors:
a. 86,000
b. 82,000
c. 70,000
d. 54,000
1. A 2. A 3. B
Estimated Gain on Realization: None/0

Assets Pledged to Fully Secured Creditors (269,000-201,600) 68,000


Free Assets
Cash 16,000
Accounts Receivable 184,000
Supplies 6,000 206,000
Total Free Assets 274,00
Less: Unsecured Creditors with priority
Wages 23,200
Taxes 4,800 28,000
Net Free Assets 246,000
Less: Unsecured Creditors
Partially Secured Creditors
Inventories 12,000
Unsecured Creditors
Trade Accounts Payable 240,000
Notes Payable (160,000-84,000) 4,800 328,000
Estimated Deficiency to Unsecured Creditos -82,000
4. The Liquid Company had a very unstable financial condition caused by a deficiency of liquid
assets. On 2-4-2008. The following information was available:
Cash
Assets Not Realized: 112,000
Accounts Receivable 80,000
Merchandise Inventory 160,000
Investmeny in common stock 26,400
Land 100,000
Building 60,000
Machinery and Equipment 48,000
Liabilities Not Liquidated:
Note Payable 244,000
Accounts Payable 288,000
Salaries and Wages 40,000
Taxes Payable 8,000
Bank Loan 180,000
Estate Deficit -173,600

During the six-month period ending 7-31-2008, the trustee sold the Investment in Common
Stock for 26,000, realized 84,000 for the accounts receivable, sold the merchandise for 152,000,
and paid-off 26,000of the bank loan and all liabilities with priorities (salaries, and wages payable,
taxes payable) as well as 7,440 for estate administration expenses.

The estate deficit, ending (July 31, 2008) should be:

a. 161,760
b. 178,000
c. 185,440
d. 189,440
5. Refer to 4, the net (gain) loss on realization and liquidation:
a. 11,840 loss
b. 11,840 gain
c. 15,840 loss
d. 4,400 loss
4. C 5. A

Estate Deficit Beginning 173,600


Assets Realized
Sales Price Fair Value (Gain) Loss
Investment 26,000 26,400 400
Accounts Receivable 84,000 80,000 -4,000
Merchandise Inventory 152,000 160,000 8,000 4,400
Liquidated:
PSCreditors-Bank Loan 26,000
Unsecured Creditors w/
Priority
Salaries and Wages 40,000
Taxes 8,000
Administrative Expense 7,440
Estate Deficit- Ending 185,440

Statement of Realization and Liquidation


Assets to be realized Assets Realized
AR 80,000 Investment in C/S 26,000
MI 160,000 AR 84,000
Investment in C/s 26,400 MI 152,000
Land 100,000
Building 60,000 Assets Not Realized
Machinery and Equip. 48,000 Land 100,000
Assets Acquired 0 Building 60,000
Machinery & Equip. 48,000
Liabilities Liquidated Liabilities to be liquidated
Bank Loan 26,000 Notes Payable 244,000
Salaries and Wages 40,000 Accounts Payable 288,000
Taxes 8,000 Salaries and Wages 40,000
Taxes Payable 8,000
Bank Loans 180,000
Liabilities not liquidated Liabilities Incurred/Assumed
Notes Payable 244,000
Accounts Payable 288,000
Bank Loan (180-26) 154,000
Supplementary Debit(s) Supplementary Credits
Administrative Expense 7,440 ________
Totals ### ###
Net Loss 11,840

---Life can only be understood backwards, but it must be lived forwards---

Chapter 1 – Partnership Formation


Theories

1. Cat and Dog formed a partnership, each contributing assets to the business. Cat contributed
inventory with a current market value in excess of its carrying amount. Dog contributed real estate
with a carrying amount in excess of its current market value. At what amount should the
partnership record each of the following assets?
Inventory Real estate
a. Market value Market value
b. Market value Carrying amount
c. Carrying amount Market value
d. Carrying amount Carrying amount

2. Recording of Cash Investment

a. Face Value b. Agreed value c. memorandum entry d. none of these

3. Recording of Property Investment

a. Face Value b. Agreed value c. memorandum entry d. none of these

4. Recording of the investment(industry)

a. Face Value b. Agreed value c. memorandum entry d. none of these

Problems

1. On May 1, 2015, Cat and Meow formed a partnership and agreed to share profits and losses in
the ratio of 3:7, respectively. Cat contributed a parcel of land that cost her P10,000. Meow
contributed P40,000 cash. The land has a fair value of P15,000. Cat insisted that the value of the
land should be P18,000. The partners agreed to value the land at P18,000. What amount should
be recorded in Cat’s capital account on formation of the new partnership?
a. P18,000 b. P17,400 c. P15,000 d. P10,000
2. On July 1, Manny and Floyd formed a partnership, agreeing to the profit and loss in the ratio of
4:6, respectively. Manny contributed a parcel of land that cost him P25,000. Floyd contributed
P50,000 cash. The land was sold for P50,000 on July 1, for hours after formation of the
partnership. How much should be recorded in Manny’s capital account on the partnership
formation?
a. P10,000 b. P20,000 c. P25,000 d. P50,000
3. Bill and Ken enter into a partnership agreement in which Bill is to have a 60% interest in capital
and profits and Ken is to have a 40% interest in capital and profits. Bill contributes the ff:
Cost Fair Value

Land P10,000 P20,000


Building P100,000 P60,000
Equipment P20,000 P15,000

There is a P30,000 mortgage on the building that the partnership agrees to assume. Ken
contributes P50,000 cash to the partnership. Bill and Ken agree that Ken’s capital account should
equal Ken’s P50,000 cash contribution and that goodwill should be recorded. Goodwill should be
recorded in the amount of:

a. P10,000 b. P15,000 c. P16,667 d. P20,000

Solution:
Cash contribution of Ken P50,000
Divided by Ken capital interest ÷ 40%
Total agreed capital P125,000
Less: Bill’s Contribution 65,000
Ken’s agreed capital P 60,000
Less: Ken’s contribution 50,000
Goodwill P 10,000

For 4 and 5

Cat admits Dog as partner in business. Accounts in the ledger for Cat on November 30, 2015, just before
the admission of Dog, show the following balances:
Cash P6,800
Accounts Receivable P14,200
Merchandise Inventory P20,000
Accounts Payable P8,000
Cat, capital P33,000

It is agreed that or the purposes of establishing Cat’s interest the following adjustments shall be made:
a. An allowance for doubtful accounts of 3% of accounts receivable is to be established
b. The merchandise inventory is to be valued at P23,000
c. Prepaid salary expenses of P600 and accrued rent expense of P800 are to be recognized.

4. Dog is to invest sufficient cash to obtain a 1/3 interest in the partnership. Cat’s adjusted capital
before the admission of Dog
a. P28,174 b. P35,347 c. P35,374 d. P36,374
5. The amount of cash investment by Dog
a. P11,971 b. P35,347 c. P17,687 d. P18,790

Solution:
Cat, capital P33,000 Cat’s capital contribution P35,347
Less: Allowance for Divided by Cat’s capital interest ÷ 2/3
doubtful accounts 426 Total agreed capital P53,061
Accrued rent Multiply by Dog’s capital interest x 1/3
expense 800 Dog’s cash contribution P17,687
Total P 31,774
Add: Inventory 3,000
Prepaid rent 600
Cat’s adjusted capital P 35,374

Chapter 2 – Partnership Operations

Theories

1. Which of the following is an expense of a partnership?


a. Interest on partners’ capital account balances
b. Interest on loans from partners to the partnership
c. Both a and b
d. Neither a and b
2. The allocation of an error should be based o the profit and loss ratio in effect when:
a. The error was made
b. The error was corrected
c. The error was discovered
d. The allocation should always be made equally
3. Partners Manny and Floyd share profits in a 2:1 ratio, respectively. Each partner receives an
annual salary allowance of P60,000. If the salaries are recorded in the accounts as a partnership
expense rather that treated as a division of net income, the total amount allocated to each
partner for salaries and net income would be:
a. Less for both Manny and Floyd
b. Unchanged for both Manny and Floyd
c. More for Manny and less for Floyd
d. More for Floyd and less for Manny
4. A partners’ withdrawal of assets from a partnership that is considered a permanent reduction in
that patners’ equity is debited to the partners’:
a. Drawing accounts
b. Retained earnings account
c. Capital account
d. Loan receivable account
5. The most common of allocating profit or loss
a. Equally
b. Arbitrary ratio
c. Ratio of partner’s capital
d. A or b

Problems

1. Sison, Torres and Velasco are partners in an accounting firm. Their capital amount balances at year-
end were: Sison, P50,000; Torres, P110,000; Velasco, P50,000. They share profits and losses on a 4:4:2
ratios, after the following terms.

 Partnership Velasco is to receive a bonus of 10% of net profit after bonus.


 Interest of 10% shall be paid on that portion of a partner’s capital in excess of P100,000
 Salaries of P10,000 and P12,000 shall be paid to partners Sison and Velasco, respectively

Assuming a net profits of P44,000 for the year, the total profit share of Sison was:

a. P7,800 b. P16,800 c. P19,400 d. 19,800

Solution:
Sison Torres Velasco Total
(4/10) (4/10) (2/10)
Bonus P 4,000 P 4,000
Interest in excess of P100,000 P 1,000 1,000
Salaries P 10,000 12,000 22,000
Remainder 6,800 6,800 3,400 17,000
Total P 16,800 P 7,800 P 19,400 P 44,000

2. Using the information in # 22 and assuming a net profit of P22,000 for the year and that the partners
agreed on the above order of profit sharing provision, the total profit share of Velasco was:

a. P4,400 b. P13,400 c. P24,600 d. P12,364

Solution:

Sison Torres Velasco Total


(4/10) (4/10) (2/10)
Bonus P 2,000 P 2,000
Interest in excess of P100,000 P 1,000 1,000
Salaries P 10,000 12,000 22,000
Remainder (1,200) (1,200) (600) (3,000)
Total P 8,800 P 800 P 13,400 P 22,000

3. Ramos, Campos, and Ocampo are partners with average capital balances in 2012 of P240,000,
P120,000 and P P80,000, respectively. Partners received 10% interest on their average capital balances.
After deducting salaries of P60,000 to Ramos and P40,000 to Campos, the residual profit or loss is
divided equally. In 2012, the partnership sustained a P66,000 loss before interest and salaries to
partners. By what amount should Ocampo’s capital account change?

a. P14,000 increase
b. P62,000 decrease
c. P70,000 decrease
d. P84,000 decrease

Solution:

Ramos Campos Ocampo Total


Interest on average capital P24,000 P12,000 P8,000 P44,000
Salaries 60,000 40,000 100,000
Remainder (70,000) (70,000) (70,000) (210,000)
Total P14,000 (P18,000) (P62,000) (P66,000)
Items 4 and 5
In the first year of the operation, Alba and Company, a partnership, made a net income of P20,000,
before providing for a salaries of P5,000 and P3,000 per annum for Alba and Bana, respectively,
as stipulated in the partnership agreement. Capital contributions and profit-sharing are as
follows:
Capital Profit share
Alba P30,000 40%
Bana P20,000 30%
Cada P10,000 30%
P60,000 100%

4. How much profit share would Cada be entitled to?


a. P6,000 b. P4,500 c. P3,600 d. None of the above
Solution:

Alba(40%) Bana(30%) Cada(30%) Total


Salaries P5,000 P3,000 P8,000
Remainder P4,800 P3,600 P3,600 P12,000
Total P9,800 P6,600 P3,600 P20,000

5. Assuming no profit and loss ratio provided in the partnership agreement and that there has been
no change in the capital contribution during the year, how much profit share would Alba be
entitled to receive?
a. P6,000 b. P4,500 c. P3,600 d. None of the above

Solution:
Alba(3/6) Bana(2/6) Cada(1/6) Total
Salaries P5,000 P3,000 P8,000
Remainder P6,000 P4,000 P2,000 P12,000
Total P11,000 P7,000 P2,000 P20,000
Chapter 3 – Partnership Dissolution: Changes in Ownership

Theories

1. The goodwill and bonus methods are two means of adjusting for differences between the net book
value and the fair value of partnerships when new partners are admitted. Which of the following
statement about these methods is correct?
a. The bonus method does not revalue assets to market values.
b. The bonus method revalues assets to market values.
c. Both methods result in the same balances in partner capital accounts.
d. Both methods result in the same total value of partner capital accounts, but the individual
capital accounts vary.

2. In the Ell-Emm partnership, Ell and Emm had a capital ratio of 3:1 and a profit and loss ratio of 2:1,
respectively. The bonus method was used to record Enn’s admittance as a new partner. What ratio
would be used to allocate, to Ell and Emm, the excess of Enn’s contribution over the amount
credited to Colter’s capital account?
a. Ell and Emm’s new relative capital ratio.
b. Ell and Emm’s new relative profit and loss ratio.
c. Ell and Emm’s old capital ratio.
d. Ell and Emm‘s old profit and loss ratio.

3. Assume that C has a P50,000 equity in the partnership of “A, B, and C.” Partner C arranges to sell his
entire interest to D for P80,000 Cash. Partners A and B agree to the admission of D. At what
amount will the equity of the incoming partner, D, be shown in the balance sheet?
a. at P50,000.

b. at P50,000 and the P30,000 will be divided equally among the original partners.
c. at P80,000
d. at P80,000 and the P30,000 will represent Goodwill which will be apportioned between the
existing equities of A and B.

4. When A retired from the partnership of A, B, and C, the final settlement of A’s interest exceeded A’s
capital balance. Under the bonus method, the excess
a. Was recorded as goodwill.
b. Was recorded as an expense.
c. Reduced the capital balances of B and C.
d. Had no effect on the capital balances of B and C.

5. Which of the following conditions constitutes a legal dissolution of a partnership?


a. Death of partner
b. Retirement of a partner
c. Admission of a partner
d. All of the above
Problems

1. Partners Alba, Basco and Castro share profits and losses 50:30:20, respectively. The statement
of financial position at April 30, 2014 follows:
Cash P 40,000 Accounts Payable P 100,000
Other assets 360,000 Alba, capital 74,000
Basco, capital 130,000
Castro, capital 96,000
Total P 400,000 Total P 400,000
The assets and liabilities are recorded and presented at their respective fair values Jocson is to
be admitted as a new partner with a 20% capital interest and a 20% share
of profits and losses in exchange for a cash contribution. No asset revaluation or bonus is to be
recorded. How much cash should Jocson contribute?
a. P60,000 b. P72,000 c. P75,000 d. P80,000

Solution:
Contributed capital of old partners P300,000
Divided by total new capital interest of old partners ÷ 80%
TAC 375,000
Multiply by capital interest of new partner x 20%
Cash Contribution of new partner P75,000

2. The following is the condensed statement of financial position of the partnership Jo, Li and Bi
who share profits and losses in the ratio of 4:3:3

Cash P 180,000 Accounts Payable P 420,000


Other assets 1,660,000 Bi, Loan 60,000
Jo, receivable 40,000 Jo, capital 620,000
Li, capital 400,000
Bi, capital 380,000
Total P1,880,000 Total P1,880,000

Assume that the assets and liabilities are fairly valued on the balance sheet and the partnership
decides to admit Mac as a new partner, with 20% interest. No asset revaluation or bonus is to be
recorded. How much Mac should contribute in cash or other assets?

a. P 350,000 b. P 280,000 c. P 355,000 d. P 284,000

Solution:

Contributed capital of old partners P1,400,000


Divided by total new capital interest of old partners ÷ 80%
TAC 1,750,000
Multiply by capital interest of new partner x 20%
Contribution of new partner P350,000

3. Carlos and Deo are partners who share profits and losses in the ratio of 7:3, respectively. On
Octuber 5, 2011, their respective accounts were as follows:
Carlos P35,000
Deo P30,000

On that date they agreed to admit Sotto as a partner with a one-third interest in the capital and
profits and losses, and upon his investment of P25,000. The new partnership will begin with total
capital of P90,000. Immedietly after Sotto’s admission, what are the capital balances of Carlos, Deo
and Sotto, respectively?

a. P30,000 P30,000 P30,000


b. P31,500 P28,500 P30,000
c. P31,667 P28,333 P30,000
d. P35,000 P30,000 P25,000

Solution:

TAC P90,000 Sotto’s contributed capital P25,000


TCC 90,000 Sotto’s agreed capital 30,000
Difference P 0 Bonus to new partner P 5,000

Carlos, capital [P35,000 – (5,000 x 7/10)] P31,500


Deo, capital [P30,000 – (5,000 x 3/10)] P28,500

4. Ell and Emm are partners sharing profits 60% and 40%, respectively. On January 1, Ell and Emm
decided to admit Enn as a new partner upon his investment of P8,000. On this date, their interest in the
partnership are as follows: Ell, P11,500; Emm, P9,300.

Assuming that the new partner is given a 1/3 interest in the firm, with bonus being
allowed the partner, the new capital balances of Ell, Emm and Enn, respectively would be:

a. P11,500 P9,300 P8,000


b. P12,480 P8,320 P8,000
c. P11,520 P7,680 P9,600
d. P10,540 P8,660 P9,600

Solution:

TAC=TCC

Enn’s contributed capital P8,000 Ell,capital [11,500 – (1,600 x .6)] P10,540


Enn’s agreed capital (28,800 x 1/3) 9,600 Emm, capital [9,300 – (1,600 x .4)] P 8,660
Bonus to new partner P1,600

5. Partners Chito and Ditas share profits in the ratio of 6:4 respectively. On December 31, 2011 their
respective capital balances were Chito, P120,000 and Ditas, P100,000. On that date Meng was admitted
as partner with a one-third interest in capital and profits for an investment of P80,000. The new
partnership began in 2011 with total capital of P300,000. Immediately after Meng’s admission, Chito’s
admission should be:

a. P120,000 b. P108,000 c. P100,000 d. P160,000

Solution:

TAC P300,000 Chito, capital [120,000 – (20,000 x .6)] P108,000


TCC 300,000
Difference 0

Meng’s contributed capital P80,000


Meng’s agreed capital 100,000
Bonus to new partner P 20,000

Chapter 4 – Lump-sum liquidation

Theories

1. Offsetting a partner’s loan balance against his debit capital balance is referred to as the:
a. Marshalling of assets
b. Right of offset
c. Allocation of assets
d. Liquidation of assets
2. The first step in the liquidation process is to
a. Convert noncash assets into cash
b. Pay partnership creditors
c. Compute any net income up to the date of dissolution
d. Allocate any gains or losses to the partners
3. If a partnership is liquidated, how is the final allocation of business assets made to the partners?
a. Equally
b. According to the profit and loss ratio
c. According to the final capital account balances
d. According to the initial investment made by each of the partners
4. Which item is not shown on the schedule of partnership liquidation?
a. Current cash balances
b. Property owned by partnership
c. Liabilities still to be paid
d. Personal assets of the partners
5. In which order are partnership assets distributed to partners under the Partnership law?
a. Capital balances, Loans, profit
b. Loans, profits, Capital balances
c. Loans, Capital balances, profits
d. Profits, Capital balances, loans

Problems

For 1-3:

The statement of financial position of BCD partnership, just before liquidation, is as follows:

Cash P 40,000 Liabilities P 70,000


Non-cash assets 140,000 Bird, Capital(60%) 50,000
Cat, capital (20%) 50,000
Dog, capital (20%) 10,000
Total P180,000 P180,000

1. If noncash assets are sold for P150,000 and the liabilities are paid, the remaining cash should be
distributed to the partners as follows:
Bird Cat Dog
a. P50,000 P50,000 P10,000
b. P44,000 P48,000 P 8,000
c. P72,000 P24,000 P24,000
d. P56,000 P52,000 P12,000
2. If noncash assets are sold for P100,000 and the liabilities are paid, the remaining cash should be
distributed to the partners as follows:
Bird Cat Dog
a. P50,000 P50,000 P10,000
b. P26,000 P42,000 P 2,000
c. P20,000 P40,000 P 0
d. P42,000 P14,000 P14,000

3. If noncash assets are sold for P 70,000 and the liabilities are paid, the remaining cash should be
distributed to the partners as follows:
Bird Cat Dog
a. P 8,000 P 8,000 P 0
b. P 5,000 P35,000 P 0
c. P 5,600 P35,200 P 0
d. P24,000 P 8,000 P 8,000

For 4-5

The statement of financial position of partnership of Tweet,Meow and Aw, just before liquidation whose
shares of profits and losses are 40%,50% and 10%, is as follows:

Cash P 50,000 Liabilities P 150,000


Non-cash assets 360,000 Tweet, Capital 160,000
Meow, capital 45,000
Aw, capital 55,000
Total P410,000 P410,000

4. If noncash assets are sold for P 300,000, How much should Tweet receive upon liquidation of
the partnership?
a. P 48,000
b. P 100,000
c. P 136,000
d. P 160,000
5. If noncash assets are sold for P 180,000, How much should Aw receive upon liquidation of the
partnership?
a. P 28,000
b. P 32,500
c. P 37,000
d. P 55,000
Chapter 5 – Installment Liquidation

Theories

1. If all partners are included in the first installment of an installment liquidation, then in future
installments
a. cash will be distributed according to the residual profit and loss sharing ratio.
b. cash should not be distributed until all non-cash assets are converted into cash.
c. a safe payments schedule must be prepared before each cash distribution to avoid excessive
payments to partners.
d. a cash distribution plan must be prepared so that partners will know when they will be included in
cash distributions.
2. In a schedule of assumed loss absorptions
a. the partner with lowest loss absorption is eliminated last.
b. it is necessary to have a cash distribution plan first.
c. the least vulnerable partner is eliminated first.
d. the most vulnerable partner is eliminated first.
3. The rank order is for claims against a bankrupt partner of:
I. Those owing to partners by way of contribution
II.Those owing to separate creditors
III.Those owing to partnership creditors
a. II first; I second and III third.
b. III first; II second and I third.
c. I first; III second and II third.
d. II first; III second and I third.
4. In partnership liquidation, the final cash distribution to the partners should be made in accordance
with the:
a. Partner’s profit and loss sharing ratio
b. Balances of the partners’ capital accounts
c. Ratio of the capital contribution by the partners
d. Ratio of capital contributions less withdrawals by the partners
5. A schedule prepared each time cash is to be distributed is called:
a. Advance cash distribution schedule
b. Marshaling of assets schedule
c. Loss absorption potential schedule
d. Safe payment schedule

Problems

1. A balance sheet for the partnership KK, LL, MM, who share profits 2:1:1 respectively, show the
following balances just before liquidation:

Cash Other Asset Liabilities KK, Capital LL, Capital MM, Capital

P48,000 P238,000 P80, 000 P88, 000 P62,000 P56,000

In the first month of liquidation, P128,000 was received on the sale of certain assets. Liquidation
expenses of P4,000 were paid, and additional liquidation expenses of P3,200 are anticipated
liquidation is completed. Creditors were paid P22,400. The available cash was distributed to the
partners. The cash to be received by each partner based on the above data:

KK LL MM KK LL MM

a. P56,600 P28,300 P28,300 c. P29,400 P32,700 P26,700


b. P86,600 P61,000 P55,000 d. P88,000 P62,000 P56,000

Solution:

KK (2/4) LL (1/4) MM (1/4)


Balances before payment to partners P88,000 P62,000 P56,000
Less: Liquidation Expense (2,000) (1,000) (1,000)
Total 86,000 61,000 55,000
Restricted interest for possible losses:
Unrealized non-cash assets.............. P110,000
Anticipated liquidation expense....... 3,200 113,200 (56,600) (28,300) (28,300)
Payment to partners P29,400 P32,700 P26,700

2. When Mikki and Mylene, partners who share earnings equally, were incapacitated in an airplane
accident, a liquidator was appointed to wind up their business. The account showed cash,
P35,00; Other assets, P110,000; Liabilities, P20,000; Mikki, capital, P71,000; Mylene, capital, P
54,000. Because of highly specialized nature of the non-cash assets, the liquidator anticipated
that considerable item would be required to disposed them. The expenses of liquidating the
business (advertising, rent, travel, etc.) are estimated at 10,000.

How much cash can be distributed safely to each partner at this point?
a. P5,000 to Mikki and P0 to Mylene
b. P5,000 to Mikki and P500 to Mylene
c. P3,000 to Mikki and P0 to Mylene
d. P5,000 to Mikki and P1,000 to Mylene

Solution:

Mikki Mylene
Balance before payment to partners P71,000 P54,000
Less: Liquidation expense (P5,000) (P5,000)
Total 66,000 49,000
Restricted interest for possible losses:
Unrealized non-cash assets.............. P110,000 (55,000) (55,000)
11,000 (6,000)
Restricted for possible insolvency of Mylene (6,000) 6,000
Payment to Partner P5,000

3. The year-end balance sheet and residual profit and loss sharing percentages for the Lang, Maas, and
Neal partnership on December 31, 2005, are as follows:
Cash P 30,000 Accounts payable P 200,000
Loan to Lang 40,000 Loan from Maas 50,000
Other assets 480,000 Lang, capital (25%) 70,000
Maas, capital (25%) 80,000
Neal, capital (50%) 150,000
Total assets P 550,000 Total liab. and equity P 550,000

The partners agree to liquidate the business and distribute cash when it becomes available. A cash
distribution plan for the Lang, Maas, and Neal partnership will show that cash available, after outside
creditors are paid, will initially go to:
a. Lang in the amount of P20,000.
b. Maas in the amount of P45,000.
c. Maas in the amount of P55,000.
d. Neal in the amount of P90,000.
Solution:

Vulnerability ranks:
Lang equity (P70,000 - P40,000)/.25 = P120,000 = 1
Maas equity (P80,000 + P50,0000/.25 = P520,000 = 3
Neal equity (P150,000/.5) = P300,000 = 2

Assumed loss absorption:


25% 25% 50%
Lang Maas Neal Total
Equities p 30,000 P 130,000 P 150,000 P 310,000
Loss to eliminate
Lang ( 30,000 ) ( 30,000 ) ( 60,000 ) ( 120,000 )
Subtotals 0 P 100,000 P 90,000 P 190,000

Loss to eliminate
Neal ( 45,000 ) ( 90,000 ) ( 135,000 )
Subtotals P 55,000 P 0 P 55,000

4. Jade, Kahl, and Lane are in the process of liquidating their partnership. Lane has agreed to accept the
inventory, which has a fair value of P60,000, as part of her settlement. A balance sheet and the residual
profit and loss sharing percentages are as follows:
Cash P 198,000 Accounts payable P 149,000
Inventory 80,000 Jade, capital (40%) 79,000
Plant assets 230,000 Kahl, capital (40%) 140,000
Lane, capital (20%) 140,000
Total assets P 508,000 Total liab./equity P 508,000

If the partners then distribute the available cash, Lane will receive:
a. P23,000.
b. P29,000
c. P30,000.
d. P34,000.
Solution:
40% 40% 20%
Jade Kahl Lane
Equities P 79,000 P 140,000 P 140,000
Distribute inventory to Lane and: ( 60,000 )
recognize P20,000 loss ( 8,000 ) ( 8,000 ) ( 4,000 )
Possible losses on plant ( 92,000 ) ( 92,000 ) ( 46,000 )
Subtotal P( 21,000 ) P 40,000 P 30,000
Eliminate Jade’s debit
balance to Kahl & Lane 21,000 ( 14,000 ) ( 7,000 )
Balance P 0 P 26,000 P 23,000

5. Hara, Ives, and Jack are in the process of liquidating their partnership. Since it may take several
months to convert the other assets into cash, the partners agree to distribute all available cash
immediately, except for P10,000 that is set aside for contingent expenses. The balance sheet and
residual profit and loss sharing percentages are as follows:

Cash P 400,000 Accounts payable P 200,000


Other assets 200,000 Hara, capital (40%) 135,000
Ives, capital (30%) 216,000
Jack, capital (30%) 49,000

Total assets P 600,000 Total liab./equity P 600,000

How much cash should Ives receive in the first distribution?


a. P146,000.
b. P147,000.
c. P153,000.
d. P156,000
Solution:
Losses 40% 30% 30%
Hara Ives Jack
Equities P 135,000 P 216,000 P 49,000
Possible loss on
remaining assets P 200,000 ( 80,000 ) ( 60,000 ) ( 60,000 )
Contingencies 10,000 ( 4,000 ) ( 3,000 ) ( 3,000 )
Subtotals P 51,000 P 153,000 P( 14,000 )

Eliminate Jack’s
debit balance ( 8,000 ) ( 6,000 ) 14,000

Safe payments P 43,000 P 147,000 P 0

Chapter 6 – Corporate liquidation

Theories

1. In a Statement of Affairs, assets pledged for partially secured creditors are:


a. Included with assets pledged for fully-secured creditors
b. Offset against partially-secured liabilities
c. Included with free assets]
d. Disregarded

2. The preferred sequence of listing (1) fully secured liabilities,(2) partially secured liabilities,(3)
unsecured liabilities with priority and(4) unsecured liabilities without priority in the liabilities
and stockholders’ equity section of a statement of affair is:
a. 1,2,3,and 4 c. 1,3,2 and 4
b. 3,1,2 and 4 d. 1,3,4 and 2

3. The estimated amount available for Free assets in Statement of Affairs for a business enterprise
undergoing bankruptcy liquidation is equal to the assets:
a. Carrying amounts less current fair values
b. Carrying amounts plus gain or less loss on realization
c. Carrying amounts plus loss or less gain on realization
d. Current fair values less carryiong amounts
4. In corporate liquidation, creditors having priority are what type of creditors?
Secured Creditors Unsecured Creditors
a. Yes Yes
b. Yes No
c. No Yes
d. No No
5. Which of the following statement is true?
a. Certain debts are not dischargeable
b. The goal of liquidation is to give the company a new start
c. All secured claims are paid in full
d. All of these are true

Problems

For items 1-4


The following data were taken from the statement of affairs of RG corp.:

Assets pledged for fully secured liabilities(current fair value,P75,000) P 90,000


Assets pledged for partially secured liabilities(current fair value,P52,000) 74,000
Free assets (current fair value, P40,000) 70,000
Unsecured liabilities with priority 7,000
Fully secured liabilities 30,000
Partially secured liabilities 60,000
Unsecured liabilities without priority 112,000

1. The amount that will be paid to creditors with priority is:


a. P7,000 b. P6,000 c. P7,500 d. P6,200

2. The amount to be paid to fully secured creditors is:


a. P30,000 b. P32,000 c. P20,000 d. P35,000

3. The amount to be paid to unsecured creditors is:


a. P78,200 b. P70,800 c. P72,000 d. P72,800

4. The amount to be paid to partially secured creditors is:


a. P52,700 b. P57,200 c. P56,200 d. P57,000

5. The statement affair for Vzell Corp. shows that approximately P0.78 on the peso probably will be paid

to unsecured creditors without priority. The corp. owe Boy Co. P23,000 on Promissory note, plus
accrued interest of P940. Inventories with current fair value of P19,200 collateral the Note payable.

Compute the amount that the Boy should receive from Vzell assuming that the actual payments to
unsecured creditors without priority consist of 78% of total claims. Round all amounts the nearest
peso:
a. P19,200 b. P22,897 c. P33,987 d. 52,200

Partnership Operations

Red and White formed a partnership in 2014. The partnership agreement provides for annual salary
allowances of 55,000 for Red and 45,000 for White. The partners share profits equally and losses in a
60/40 ratio. The partnership had earnings of 80,000 for 2014 before any allowance to partners.

1. What amount of these earnings should be credited to each partner’s capital account?

Red White
A. 40,000 40,000
B. 43,000 37,000
C. 44,000 36,000
D. 45,000 35,000
Answer: ( B )

Philogyny and Love formed a partnership on January 2, 2014. Philogyny and Love contributed capital of
350,000 and 50,000 respectively. They agreed to share profits and losses 80% and 20%, respectively.
Love is given a monthly salary of 5,000 and a 15% bonus based on income before salaries, interest and
bonus. Both partners are given an interest of 5% of the beginning capital. The income statement for the
year ended prepared by the company’s bookkeeper is shown below:

Net Sales 1,750,000


Cost of Sales (1,400,000)
Gross Profit 350,000
Expenses (including salary, interest, and bonus) (286,000)
Net Income 64,000

1. What is the amount of Bonus to Love in 2014?


A. 25,412
B. 0
C. 2,087
D. 18,782.61
Answer: ( A )

Aubrey and Lauren are partners operating a small chain of convenience stores. Their business has grown
substantially over the last six years and they amended their partnership agreement to provide the
following distribution of profits and losses:

Aubrey Lauren
Salaries 49,000 None
Commission on Gross Sales None 4%
Interest on average capital balances 7% 9%

Bonus to Aubrey is 10% of net income after salary, commissions, interest and bonus.

Remainder 40% 60%

Gross Sales for 2014 was 3,000,000. Income after deducting salaries, commissions, and interest was
132,000. Average capital balances were 720,000 and 540,000 for Aubrey and Lauren.

1. How much profit share will Aubrey receive?


A. 158,920
B. 159,400
C. 152,800
D. 152,500
Answer: ( B )

A, B and C are engaged in a merchandising business. Their capital accounts in the ABC Partnership in
2015 are as follows:
A B C
January 1 100,000 150,000 120,000
April 1, withdrawal 5,000
May 1, investment 10,000 15,000
August 1, investment 15,000
December 1, withdrawal 6,500 30,000

The partners agreed on the following terms: 8% interest on their average capital balances. Quarterly
salaries of 75,000; 55,000; 68,000 for them respectively. Bonus is given to B, 5% of income before tax
after his interest, his salary, and bonus. For the year ended, the partnership has a credit balance in the
income summary account of 550,000. The relevant tax rate during the year is 20%. The remainder will
be divided in the ratio 2:3:4.

1. How much is the share of partner B in the net income?


A. 158,251
B. 155,971
C. 151,286
D. 153,532
Answer: ( B )

2. How much is the capital balance of Partner A at December 31, 2015?


A. 352,743
B. 353,824
C. 350,732
D. 351,867
Answer: ( A )

Distribution of Income

1. If the partnership agreement does not specify how income is to be allocated, profits should be
allocated

a. Equally.

b. In proportion to the weighted-average of capital invested during the period.

c. Equitably so that partners are compensated for the time and effort expended on behalf of the
partnership

d. In accordance with an established ratio.

2. Which of the following is not a characteristic of the proprietary theory that infkuences accountung
for partnerships?

a. partners' salaries are viewed as a distributionof income rather than a componenr of net income.

b. a partnership is not viewed as a separate, distinct, taxable entity.

c. a partnership is characterized by limited liability

d. changes in the ownership structure of a partnership result in the dissolution of the partnership.

answer: C

3. which of the following woukd be least likely would be leasr likely to be used as means of allocating
profits among partners who are active in rye management of the partnersip?

a. salaries

b. bonus as a percentage of net income before the bonus


c. bonus as a percentage of sales in excess of a targeted amount

d. interesr on ave. capital balances

answer: D

4. which of the following best describes the use of interest on invested capital as a means of allocating
profits?

a. if interest on invested capital is used, if must be used for all partners

b. interesr is allocated only if there is partnership net profit

c. invested capital balances are never affected by drawings of the partnerships

d. use of beginning or ending measures of invested capital may be subject to manipulation that distorts
the measure of invested capital.

answer: D

5. a partneship agreement calls for allocation of profits and losses by salary allocations, a bonus
allocation, interest on capital, with any remainder to be allocated by presenr ratios. if a partnershop has
a loss to allocate, generally which of the ff. procedures would be applied?

a. any loss would be allocated equally to all partners

b. any salary allocation criteria would not be used

c the bonus criteria would not be used

d. the loss would be allocated using the profit and loss ratios only

answer: C

Partnership Lump-sum Dissolution

After all noncash assets have been converted into cash in the liquidation of the AA and JJ partnership,
the ledge contains the following account balances:

Debit Credit
Cash 34,000
Accounts payable 25,000
Loan payable to AA 9,000
AA, capital 8,000
JJ, capital 8,000
1. Available cash should be distributed: 25,000 to accounts payable and:

A. 9,000 loan payable to AA


B. 4,500 each to AA and JJ
C. 1,000 to AA and 8,000 to JJ
D. 8,000 to AA and 1,000 to JJ
Answer: ( C )

Partners A, B and C are partners of ABC Partnership and decided to liquidate the business. Below is their
condense statement of financial position dated December 31, 2012.

ASSETS LIABILITIES AND EQUITY


Cash 15,000 Liabilities 75,000
Non-cash 110,000 A, capital (35%) 5,000
B, capital (45%) 15,000
C, capital (20%) 30,000

Total assets 125,000 Total liabilities&equity 125,000

The personal assets and liabilities of the partners on this date apart from their equities in the
partnership are as follows:

Partners Personal Assets Personal Liabilities


A 100,000 25,000
B 50,000 50,000
C 5,000 60,000

Assume that the non-cash assets are sold for 40,000 and liquidation expenses of 13,500 are incurred and
paid.
1. How much is the total cash paid to partners?
A. 14,366
B. 22,575
C. 5,091
D. 8,209
Answer: (C )
Due to financial difficulty partners G, H na I decided to liquidate. The following balances are before
liquidation: Capital balances of G, H and I are 10,000; 25,000; and 20,000 respectively. Loan from G is
25,000; Cash 25,000; Profit and loss ratio 30:35:35 respectively.

Partner H received 10,650 upon liquidation and the share of the liquidation expenses of partner G is
1,800. Meanwhile, the cash available after realizing the non-cash asset and paying the liquidation
expenses is 69,000.

1. How much is the book value of the non-cash asset sold?


A. 85,000
B. 80,000
C. 86,000
D. 85,500
Answer: ( A )

2. How much is paid to the outside creditors?


A. 35,000
B. 45,000
C. 30,000
D. 40,000
Answer: ( C )

The accounts of the partnership of Larry, Ralph, and Bert at the end of its fiscal year on September
30,2014 are as follows:

Cash 36,000 Loan from Bert 18,000


Other assets 225,000 Larry, capital (30%) 81,000
Loan to Ralph 9,000 Ralph, capital (50%) 54,000
Liabilities 90,000 Bert, capital (20%) 27,000

Bert received 16,200 on the first distribution of cash.


1. What was the cash realized from the initial sale of assets?
A. 18,000
B. 180,000
C. 108,000
D. 120,000
Answer: ( C )

Theories

1. Prior to partnership liquidation, a schedule of possible losses is frequently prepared to determine the amount
of cash that may be safely distributed to the partners. The schedule of possible losses

a. Consists of each partner’s capital account plus loan balance, divided by that partner’s profit-and-loss
sharing ratio.

b. Shows the successive losses necessary to eliminate the capital accounts of partners (assuming no
contribution of personal assets by partners).

c. Indicates the distribution of successive amounts of available cash to each partner.

d. Assumes contribution of personal assets by partners unless there is a substantial presumption of personal
insolvency by the partners.
2. The final cash distribution to the partners in a partnership in liquidation should be made in accordance with

a. Balances of the partners’ capital accounts.

b. Partners’ profit and loss sharing ratio.

c. Ratio of capital contributions made by the partners.

d. Ratio of capital contributions less withdrawals made by the partners.

3. In a partnership liquidation, the final cash distribution to the partners should be made in accordance with the

a. Partners’ profit and loss sharing ratio.

b. Balances of the partners’ capital accounts.

c. Ratio of capital contributions made by the partners.

d. Ratio of capital contributions less withdrawals made by the partners.

4. The following is the priority sequence in which liquidatoon proceeds will be distributed to the
partnership

a. patnership: drawings, liabilities, loans, capital balances

b. partneship: liabilities, loans, capital balances

c. partnership: liabilities, loans, drwaings, capital balances

d. partnership: liabilities, capital balances, koans.

answer: B

5. The Doctrine of marshalling assets

a. is applicable only if the partnership is insolvent

b. allows the partners to first contribute personal assets to unsatisfied partnership creditors

c. is applicable if either the partnership is insolvent or individual partners are insolvent.

d. provides that when the uniform partnership act is adopted, amounts owed to personal creditors and
to the partnership for debit capital balances are shared proportionately from the personal assets of the
partners.
answer: C

Partnership Installment Liquidation

The balance sheet of the partnership of Hypnopompic, Wakefulness and Semiconscious are shown
below:
HWS Partnership
Balance Sheet
December 31, 2013
Cash 50,000 Liabilities 80,000
Non-cash Assets 250,000 Hypnopompic, capital (50%) 100,000
Wakefulness, capital (25%) 75,000
Semiconscious, capital (25%) 45,000
TOTAL 300,000 TOTAL 300,000

On January 2014, certain non-cash assets were sold for a certain amount. Liquidation expenses and
liabilities of 4,000 and 25,000 were paid. Future liquidation expenses of 5,000 are anticipated.
Wakefulness received 42,750 from the first distribution of available cash.

1. How much is the cash received from realization?


A. 120,000
B. 140,000
C. 130,000
D. 75,000
Answer: ( C )

2. Assuming that on February 2014, the remaining non-cash assets were sold for 75,000 and
liquidation expenses of 5,000 are paid, how much is the total cash received by Hypnopompic
from the two distributions of cash?
A. 37,500
B. 73,000
C. 75,000
D. 74,000
Answer: ( B )

Capital balances of partners Q, R, S are the following before liquidation: 87,000; 95,500; 106,250
respectively. The partnership has a loan from partner Q in the amount of 8,000; loan to partner R in the
amount of 4,500; advances to partner S in the amount of 6,500. The partner’s profit and loss ratio is
25:40:35 respectively.

1. If in the first installment, the total cash paid to partners is 57,000, how much did partner S
receive?
A. 0
B. 19,396
C. 13,854
D. 20,125
Answer: ( B )

2. If partner Q received 20,000 in the first installment and partners received 12,396 in the second
installment, how much is received by partner Q as of the second installment?
A. 12,604
B. 8,854
C. 23,750
D. 32,604

Answer: ( D )

3. How much is the total cash paid to partners in the second installment?
A. 25,000
B. 30,000
C. 35,000
D. 40,000

Answer: ( A )

1. The Doctrine of marshalling assets

a. is applicable only if the partnership is insolvent

b. allows the partners to first contribute personal assets to unsatisfied partnership creditors

c. is applicable if either the partnership is insolvent or individual partners are insolvent.

d. provides that when the uniform partnership act is adopted, amounts owed to personal creditors and
to the partnership for debit capital balances are shared proportionately from the personal assets of the
partners.

answer: C

2. The final cash distribution to the partners in a partnership in liquidation should be made in accordance with

a. Balances of the partners’ capital accounts.

b. Partners’ profit and loss sharing ratio.

c. Ratio of capital contributions made by the partners.

d. Ratio of capital contributions less withdrawals made by the partners.

3. In a partnership liquidation, the final cash distribution to the partners should be made in accordance with the
a. Partners’ profit and loss sharing ratio.

b. Balances of the partners’ capital accounts.

c. Ratio of capital contributions made by the partners.

d. Ratio of capital contributions less withdrawals made by the partners.

4. under the Doctrine of Marshalling of assets, unsatisfied partnership creditors

a. must first proceed against the partner with the largest capital balance

b. may attach to the assets of an individual partner before individual creditors have been satisfied

c. may proceed against any personally solvent partner

d. may proceed against any personally solvent partner but only to the extent of their capital balance in
the partnership.

answer: C

5 Prior to partnership liquidation, a schedule of possible losses is frequently prepared to determine the amount of
cash that may be safely distributed to the partners. The schedule of possible losses

a. Consists of each partner’s capital account plus loan balance, divided by that partner’s profit-and-loss
sharing ratio.

b. Shows the successive losses necessary to eliminate the capital accounts of partners (assuming no
contribution of personal assets by partners).

c. Indicates the distribution of successive amounts of available cash to each partner.

d. Assumes contribution of personal assets by partners unless there is a substantial presumption of personal
insolvency by the partners.

Corporate Liquidation

On December 31,2014, the Statement of Affairs of Tarry Company, which is in bankruptcy liquidation,
included the following:

Assets pledged for fully secured liabilities 100,000


Assets pledged for partially secured liabilities 40,000
Free assets 120,000
Fully secured liabilities 80,000
Partially secured liabilities 50,000
Unsecured liabilities with priority 60,000
Unsecured liabilities without priority 90,000

1. How much is the estimated amount to be paid to partially secured liabilities?


A. 50,000
B. 48,000
C. 47,500
D. 45,000
Answer: ( B )

2. How much is the estimated amount to be paid to all creditors?


A. 80,000
B. 140,000
C. 250,000
D. 260,000
Answer: ( D )

Rocket Bunny Corp. is experiencing financial difficulty and is in the process of liquidation. In its
statement of financial position, the mmortgage payable of 110,000 is secured by the land with carrying
amount of 100,000 and fair market value of 105,000. Accrued expenses total 15,000 of which 10,000
represents salaries of employees and the remainder is secured by the inventory with carrying amount of
20,000 and fair market value of 10,000. Total liabilities reflected in the statement of financial position
amount to 132,000. Estimated payment to partial creditors is 108,250. The amount of total assets is
135,000.

1. How much is the estimated recovery percentage?


A. 60%
B. 65%
C. 70%
D. 75%
Answer: ( B )

2. How much is the estimated deficiency?


A. 4,200
B. 3,600
C. 3,000
D. 4,800
Answer: ( A )
3. How much is the payment to unsecured creditors without priority?
A. 4,900
B. 5,250
C. 4,200
D. 4,550
Answer: ( D )

Thories
1. How is a straight bankruptcy proceeding commenced?

a. by discharging all debt.

b. by selling the property of the estate.

c. by filing an order for relief.

d. by filing a petition in bankruptcy.

answer: c

2. If Deroy’s creditors force him into bankruptcy proceedings, what is this called?

a. an armed proceeding.

b. involuntary bankruptcy.

c. concealed bankruptcy.

4. an automatic bankruptcy.

answer: B

3 When a corporation reorganizes, which of the following things must the


reorganization plan NOT do?

a. designate a class of claims and interest.

b. favor a group within a class of secured creditors.

c. provide adequate means for execution.

d. be fair and equitable.

answer: B

4 which of the following might be a reason for denying the discharge of the debtor, as
opposed to the debt, in a bankruptcy proceeding?

a. the debtor acted in good faith.

b. the debtor refused to sign a reaffirmation agreement.


c. thedebtor fraudulently concealed or destroyed financial records.

d. the debtor relinquished all property.

answet: C

5. Normally, in order to receive a portion of a debtor’s estate, what must each creditor
file?

a. petition in bankruptcy.

b. proof of claim.

c. notice of exemption.

d. bill of attainder.

answer: B

Partnership Formation

On January 1, 2015, Ernie and Bert both sole proprietors decided to form a partnership to expand both
of their businesses. According to their agreement, they will split profits and losses 75:25 and their initial
capital will also reflect that ratio.

The following are Ernie and Bert’s Statement of Financial Position:

Ernie Proprietor
Statement of Financial Position
December 31, 2014
ASSETS LIABILITIES AND EQUITY

Cash 50,000 Accounts payable 65,000


Accounts Receivable 100,000 Accrued expenses 55,000
Inventories 75,000 Notes payable 80,000
Equipment 250,000 Ernie, capital 90,000
Accumulated depreciation- Equipment (185,000)
TOTAL ASSETS 290,000 TOTAL LIABILITIES&EQUITY 290,000

Bert Proprietor
Statement of Financial Position
December 31, 2014
ASSETS LIABILITIES AND EQUITY
Cash 30,000 Accounts Payable 75,000
Accounts receivable 110,000 Accrued expenses 90,000
Inventories 85,000 Notes Payable 100,000
Equipment 300,000 Bert, Capital 160,000
Accumulated Depreciation- Equipment (100,000)
TOTAL ASSETS 425,000 TOTAL LIABILITIES&EQUITY 425,000

The values reflected in the Statement of Financial Position are already at fair values except fo the
following accounts:

Ernie’s Accounts Receivable is now 20,000 less than what is stated in his Statement of Financial Position.
Both inventories of Ernie and Bert are now 90,000 and 70,000 respectively. Equipment for Bert has an
assessed value of 275,000, appraised value of 250,000 and book value of 200,000. Additional accrued
expenses are to be established in the amount of 10,000 for Bert only while additional accounts payable
in the amount of 5,000 for Ernie. It is also agreed that all liabilities will be assumed by the partnership,
except for the notes payable of Bert which will be personally paid by him.

1. How much is the adjusted capital balance of Bert upon formation?


A. 91,250
B. 185,000
C. 285,000
D. 310,000
Answer: ( C )

2. How much is the capital credit to Ernie upon formation?


A. 80,000
B. 273,750
C. 292,000
D. 255,500
Answer: ( B )

3. How much should Ernie invest as additional cash to be in conformity with their initial capital
agreement?
A. 193,750
B. 212,000
C. 175,500
D. 205,000
Answer: ( A )

Bonnie and Clyde enters into a partnership agreement in which Bonnie is to have 55% interest in the
partnership and 35% in the profits and losses, while Clyde will have 45% interest in the partnership and
65% in the profits and losses. Bonnie contributed the following:

Cost Fair value


Building 235,000 255,000
Equipment 168,000 156,000
Land 500,000 525,000
The building and the equipment has a mortgage of 50,000 and 35,000 respectively. Clyde is to
contribute 150,000 cash and equipment. The partners agreed that only the building mortgage will be
assumed by the partnership.

1. How much is the fair market value of the equipment which Clyde contributed?
A. 615,818
B. 989,143
C. 546,273
D. 574,909
Answer: ( D )

2. How much is the total asset of the partnership upon formation?


A. 1,892,143
B. 1,701,818
C. 1,660,909
D. 1,632,273
Answer: ( C )

Theories (letter of answer is underlined)

1 The partnership agreement is an express contract among the partners (the owners of the business).
Such an agreement generally does not include
a. A limitation on a partner’s liability to creditors.
b. The rights and duties of the partners.
c. The allocation of income between the partners.
d. The rights and duties of the partners in the event of partnership dissolution.

2. A partnership records a partner’s investment of assets in the business at


a. The market value of the assets invested.
b. A special value set by the partners.
c. The partner’s book value of the assets invested.
d. Any of the above, depending upon the partnership agreement.

?3. When property other than cash is invested in a partnership, at what amount should the noncash
property be credited to the contributing partner’s capital account?
a. Fair value at the date of recognition.
b. Contributing partner’s original cost.
c. Assessed valuation for property tax purposes.
d. Contributing partner’s tax basis.
?4. When property other than cash is invested in a partnership, at what amount should the noncash
property be credited to the contributing partner’s capital account?
a. Fair value at the date of contribution.
b. Contributing partner’s original cost.
c. Assessed valuation for property tax purposes.
d. Contributing partner’s tax basis.

5. Four individuals who were previously sole proprietors form a partnership. Each partner contributes
inventory and equipment for use by the partnership. What basis should the partnership use to record
the contributed assets?
a. Inventory at the lower of FIFO cost or market.
b. Inventory at the lower of weighted-average cost or market.
c. Equipment at each proprietor’s carrying amount.
d. Equipment at fair value.

Partnership Dissolution: Changes in Ownership

CC and DD are partners who share profits and losses in the ratio of 7:3, respectively. On October 21,
2014, their respective capital accounts were as follows:

CC 35,000
DD 30,000
65,000
1. On that date they agreed to admit EE as a partner with a one-third interest in the capital and
profits and losses, and upon his investment of 25,000. The new partnership will begin with a total capital
of 90,000. Immediately after EE’s admission, what are the capital balances of CC, DD and EE,
respectively?

A. 30,000; 30,000; 30,000


B. 31,500; 28,500; 30,000
C. 31,667; 28,333; 30,000
D. 35,000; 30,000; 25,000
Answer: ( B )

Partner’s P and Q has capital balances of 358,500 and 300,000 respectively before admitting R, P and Q
share profits and losses in the ratio 6:4. R paid 225,000 in exchange for 30% interest in the partnership
as well as the profits and losses.

1. How much is the capital of Partner P after admission of R?


A. 250,950
B. 250,590
C. 279, 480
D. 269,580
Answer: ( A )
2. How much is debited from the capital of partner Q upon R’s admission?
A. 120,000
B. 90,000
C. 79,020
D. 105,360
Answer: ( B )

Halina, Barbara, and Maricar are partners sharing profits and losses in the ratio of 2:2:1. On January 2,
2014, their capital balances are as follows:

Halina 87,500
Barbara 62,500
Maricar 50,000

The partners agree to admit Lolita on the following agreement:


> Lolita is to pay Halina and Maricar a total 70,000 for ¼ of each of their interest.
> Lolita is also to invest 60,000 in the partnership.
> The total capital of the partnership is to be 300,000, of which Lolita’s interest is to be 30%.

1. What is the capital balance of Maricar after the admission of Lolita?


A. 46,375
B. 45,500
C. 58,000
D. 80,250
Answer: ( A )

Jack Sanchez, a partner in a law firm, decided to withdraw from the partnership. Sanchez’s share in the
partnership profits and losses was 20%. Upon withdrawal from the partnership he was paid cash in final
settlement for his interest. The total of the partners’ capital accounts before the recognition of the
revaluation prior to Sanchez’s withdrawal was 315,000. After his withdrawal, the remaining partners’
capital accounts, excluding their share of revaluation, totaled 240,000, but including their share of
revaluation, totaled 384,000.

1. What is the total amount of cash paid to Jack Sanchez?


A. 111,000
B. 96,000
C. 99,000
D. 87,000
Answer: ( A )

Theories

Goodwill Method
Admission of New Partners
?1. The goodwill and bonus methods are two means of adjusting for differences between the net book
value and the fair value of partnerships when new partners are admitted. Which of the following
statement about these methods is correct?
a. The bonus method does not revalue assets to market values.
b. The bonus method revalues assets to market values.
c. Both methods result in the same balances in partner capital accounts.
d. Both methods result in the same total value of partner capital accounts, but the individual
capital accounts vary.

Purchase Method
2. Assume that C has a P50,000 equity in the partnership of “A, B, and C.” Partner C arranges to sell his
entire interest to D for P80,000 Cash. Partners A and B agree to the admission of D. At what amount
will the equity of the incoming partner, D, be shown in the balance sheet?
a. at P50,000.
b. at P50,000 and the P30,000 will be divided equally among the original partners.
c. at P80,000
d. at P80,000 and the P30,000 will represent Goodwill which will be apportioned between the
existing equities of A and B.

Bonus Method
?3. In the Adel-Brick partnership, Adel and Brick had a capital ratio of 3:1 and a profit and loss ratio of
2:1, respectively. The bonus method was used to record Colter’s admittance as a new partner. What
ratio would be used to allocate, to Adel and Brick, the excess of Colter’s contribution over the amount
credited to Colter’s capital account?
a. Adel and Brick’s new relative capital ratio.
b. Adel and Brick’s new relative profit and loss ratio.
c. Adel and Brick’s old capital ratio.
d. Adel and Brick’s old profit and loss ratio.

4. When Mill retired from the partnership of Mill, Yale, and Lear, the final settlement of Mill’s interest
exceeded Mill’s capital balance. Under the bonus method, the excess
a. Was recorded as goodwill.
b. Was recorded as an expense.
c. Reduced the capital balances of Yale and Lear.
d. Had no effect on the capital balances of Yale and Lear.

You might also like